Vous êtes sur la page 1sur 300

.

PROBLEMAS SOBRE GEOMETRÍA


Relaciones métricas en la circunferencia. Lugares geométricos

MARÍA DEL CARMEN RODRÍGUEZ MARÍN

Máster en Matemáticas

Universidad de Granada. 2014


Problemas sobre Geometría
Relaciones métricas en la circunferencia. Lugares geométricos

MARÍA DEL CARMEN RODRÍGUEZ MARÍN

Trabajo dirigido por el prof. Pascual Jara


Departamento de Álgebra

Máster en Matemáticas
Universidad de Granada. 2014
.

Dedicatoria.
Este trabajo se lo quiero dedicar a “Sapillo”, mi sobrina Leire, por la motivación que me ha transmitido
a lo largo de este proyecto.
.

Septiembre, 2014 Curso 20132014.


Agradecimientos

Este trabajo no hubiese sido posible sin los conocimientos adquiridos durante mi licenciatura en la
Universidad de Granada y los proporcionamos por este máster.
Además, agradecer a cada uno de los profesores que me han ayudado y en especial al profesor
Pascual Jara Martínez, tutor de este proyecto, por su dedicación durante estos meses.
Finalmente, quiero agradecer a mis padres, que me han permitido cursar estos estudios y han con-
fiado siempre en mí, a mi hermano por apoyarme en todo momento y mi compañera de piso que
siempre ha estado dándome ánimos cuando más lo necesitaba.
ii

Septiembre, 2014 Curso 20132014.


Introducción

El presente Trabajo Fin de Máster tiene como objetivo principal el desarrollo del pensamiento y
habilidades matemáticas tanto para aquellos que quieran aprender sobre los temas tratados, como
aquellos que quieran prepararse para el reto de las olimpiadas matemáticas. Este material puede
permitir fortalecer el potencial de los alumnos y ayudarlos en sus debilidades, generando estrategias
de aprendizaje.
Las olimpiadas matemáticas, además de ser un concurso, permiten acercar éstas a aquellos jóvenes
inquietos en su educación. Su finalidad es estimular el estudio de la Matemática y el desarrollo de
jóvenes talentos en esta Ciencia.
Hemos considerado en la olimpíada tres fases: local, nacional e internacional. Para resolver los pro-
blemas de cada fase el alumno debe conocer conocimientos específicos a un determinado nivel, sin
olvidar que la finalidad última de la resolución de los problemas es desarrollar capacidades y habi-
lidades que le permitan enfrentarse a otras situaciones.
En este trabajo se ha realizado el estudio de dos temas fundamentales de la Geometría: relaciones
métricas en la circunferencia y lugares geométricos, y se han enfocado para ayudar al alumno en su
preparación para las olimpiadas de matemáticas en sus diferentes fases.
Como parte teórica se incluyen dos temas en los que se han recopilado los resultados que han si-
do necesarios en la resolución de los problemas tratados, por esta razón puede no ser exhaustivo
su contenido. Esta introducción teórica se acompaña de ejemplos que ilustran sobre las nociones
tratadas.
El último epígrafe, trata sobre la resolución y análisis de problemas en los tres niveles antes mencio-
nados. Los más sencillos se organizan bajo el rótulo de “Fase Local”, y pueden servir de ejercicios de
introducción a los temas tratados. Como siempre, su dificultad depende de los conocimientos pre-
vios, y de ahí la importancia de las nociones incluidas en los primeros temas. Más elaborados, en su
resolución, son los problemas que aparecen bajo el rótulo de “Fase Nacional”; se trata de problemas
en los que el proceso de resolución requiere varios pasos, lo que incrementa su dificultad. Finalmente
en la “Fase Internacional” hemos incluido los problemas de una mayor complejidad, tanto en su re-
solución como en las nociones que involucran. La ubicación en uno u otro apartado es simplemente
cuestión del bagaje del lector, y el redactor, por lo que la clasificación que aquí aparece es solamente
una forma de estructurar la materia tratada.
Con la aparición de nuevas herramientas de computación, y el uso de sistemas gráficos los problemas
en Geometría pueden ser estudiados de una forma más rápida a como se hacía en el pasado; además
el uso de estas herramientas permite la simulación y por lo tanto la detección de invariantes en
determinadas construcciones. Hemos puesto a prueba uno de estos sistemas y comprobado que el
iv

tratar los problemas planteados en competiciones nacionales o internacionales se pueden abordar de


forma más elegante tras su análisis en este sistema de cálculo simbólico; lo que reduce la complejidad
de los mismos, permite trabajar nuevas soluciones y realizar variaciones, una más sencillas, y otras
más complejas. El sistema utilizado ha sido Geogebra, que además permite exportar el código de los
dibujos a para ser incluidos en LaTeX, y realizar construcciones dinámicas, mostrando en particular
el lugar geométrico de determinados elementos, una de los temas de estudio de este trabajo.
Geogebra es un software de geometría dinámica y cálculo simbólico de uso libre, que permite enseñar
y aprender matemáticas desde Primaria a Universidad, con muchos recursos y materiales libres, y
mantenido por una amplia comunidad.

Septiembre, 2014 Curso 20132014.


Índice general

Agradecimientos I

Introducción III

I Relaciones métricas en la circunferencia 1


1 Conceptos fundamentales . . . . . . . . . . . . . . . . . . . . . . . . . . . . . . . . . . . 1

II Lugares geométricos 17
2 Lugares geométricos . . . . . . . . . . . . . . . . . . . . . . . . . . . . . . . . . . . . . . 17

III Problemas y otros desarrollos 25


3 Problemas de Olimpiadas. Fase Local . . . . . . . . . . . . . . . . . . . . . . . . . . . . 25
4 Problemas de Olimpiadas. Fase Nacional . . . . . . . . . . . . . . . . . . . . . . . . . . 41
5 Problemas de Olimpiadas. Fase Internacional . . . . . . . . . . . . . . . . . . . . . . . 65

Bibliografía 87

Bibliografía. Referencias Web 89

Índice alfabético 91
.
Capítulo I

Relaciones métricas en la circunferencia

1. Conceptos fundamentales

Circunferencia
Una circunferencia se define como el lugar geométrico (Ver página 17) de los puntos del plano
equidistantes de otro, llamado centro de la circunferencia.
También podemos decir que la circunferencia es la línea formada por todos los puntos que están a
la misma distancia de otro punto, llamado centro.
Una circunferencia queda determinada cuando conocemos:
Tres puntos de la misma.
El centro y el radio.
El centro y un punto en ella.
El centro y una recta tangente a la circunferencia.

En una circunferencia podemos distinguir los siguientes elementos:

(I) Centro: punto (interior) equidistante de todos los puntos de la circunferencia.


(II) Radio: segmento que une el centro de la circunferencia con un punto cualquiera de la misma.
El radio mide la mitad del diámetro y es igual a la longitud de la circunferencia dividida entre
2π.
(III) Diámetro: segmento que une dos puntos de la circunferencia y pasa por el centro. El diámetro
mide el doble del radio y es igual a la longitud de la circunferencia dividida entre π.
(IV) Cuerda: segmento que une dos puntos de la circunferencia. El diámetro es la cuerda de longi-
tud máxima.
(V) Recta secante: recta que corta a la circunferencia en dos puntos.
(VI) Recta tangente: recta que toca a la circunferencia en un sólo punto.
(VII) Punto de tangencia: punto de contacto de la recta tangente con la circunferencia.
(VIII) Arco: cada una de las partes en que una cuerda divide a la circunferencia.
(IX) Semicircunferencia: cada uno de los dos arcos delimitados por los extremos de un diámetro.
2 CAP. I. RELACIONES MÉTRICAS EN LA CIRCUNFERENCIA

tangente secante

arco

cuerda
diámetro

radio centro

punto de tangencia

Una circunferencia se puede representar, mediante su ecuación ordinaria o su ecuación general.

Ecuación ordinaria
Para cualquier punto, P(x, y), de una circunferencia cuyo centro es el punto C(a, b) y radio r,
la ecuación ordinaria es:
(x − a)2 + ( y − b)2 = r 2 . (I.1)
En el caso particular en el que el centro de la circunferencia coincide con el origen de coorde-
nadas, (0, 0), la ecuación queda reducida a:
x 2 + y 2 = r 2. (I.2)
Ecuación general
Una vez que se tiene la ecuación ordinaria de una circunferencia podemos obtener su ecuación
general. Para ello eliminamos los paréntesis desarrollando los binomios, pasamos todos los
términos al primer miembro e igualamos a cero.
x 2 − 2ax + a2 + y 2 − 2b y + b2 − r 2 = 0.
Ordenando dicha ecuación obtenemos:
x 2 + y 2 − 2ax − 2b y + a2 + b2 − r 2 = 0.
Para tener una ecuación más sintetizada hacemos las siguientes asignaciones:
−2a = D,
−2b = E,
a2 + b2 − r 2 = F.

Septiembre, 2014 Curso 20132014.


SEC. 1. CONCEPTOS FUNDAMENTALES 3

Por tanto la ecuación general de la circunferencia quedaría expresada de la siguiente manera:

x 2 + y 2 + Dx + E y + F = 0. (I.3)

Ésta debe cumplir una serie de condiciones:

No existe término en x y.

Los coeficientes de x 2 e y 2 son iguales.


−D
Si D = −2a entonces a = .
2
−E
Si E = −2b entonces b = .
2
p
Si F = a2 + b2 − r 2 entonces r = a2 + b2 − F .

Otra condición necesaria para que una ecuación dada represente una circunferencia es que:

a2 + b2 − F > 0.

Potencia de un punto respecto a una circunferencia


Dada una circunferencia C y un punto P cualquiera del plano, cada recta que pase por P y corte a C
en dos puntos A y B verifica la siguiente propiedad:

PA · P B = PA0 · P B 0 = PA00 · P B 00 = K,

A00
A0

B 00

B0
B

TFM: Problemas sobre Geometría M.C. Rodríguez


4 CAP. I. RELACIONES MÉTRICAS EN LA CIRCUNFERENCIA

siendo K la potencia de P respecto a C.


Es decir, el valor del producto es independiente de la recta considerada. La constante K sólo depende
de P y C.
Dependiendo de la posición del punto respecto a la circunferencia, la potencia de éste varía, es decir,
si el punto es exterior a la circunferencia entonces tendrá potencia positiva, si es interior su potencia
será negativa y si está en la circunferencia entonces es nula.
En relación con lo anterior, existen una serie de resultados que caben destacar:

Teorema. 1.1. (Teorema de las secantes)


Sean PA y P B dos rectas secantes, entonces:

PA · P D = P B · P C.

O C

Septiembre, 2014 Curso 20132014.


SEC. 1. CONCEPTOS FUNDAMENTALES 5

Teorema. 1.2. (Teorema de la tangente y la secante)


Sean PA una recta tangente y P C una recta secante, entonces:

(PA)2 = P C · P D.

Teorema. 1.3. (Teorema de las tangentes)


Sean PA y P C dos rectas tangentes, entonces:

PA = P C.

TFM: Problemas sobre Geometría M.C. Rodríguez


6 CAP. I. RELACIONES MÉTRICAS EN LA CIRCUNFERENCIA

Teorema. 1.4. (Teorema de las cuerdas)


Sean AB y C D dos cuerdas, entonces:

AP · P B = C P · P D.

A D

P
O

Eje radical de dos circunferencias


El eje radical de dos circunferencias es el lugar geométrico (Ver página17) de los puntos que tienen
igual potencia con respecto a ambas circunferencias.
Sean las circunferencias C de centro O y radio r y C 0 de centro O0 y radio r 0 . Llamamos d y d 0 a las
distancias de P a O y O0 respectivamente. Entonces un punto P tendrá igual potencia respecto de C
y C 0 si y sólo si:
d 2 − r 2 = d 02 − r 02 ⇔ d 2 − d 02 = r 2 − r 02 = Cte.
El lugar geométrico de puntos cuya diferencia de cuadrados de distancias a los puntos C y C 0 es
constante es una recta perpendicular a la recta que une los puntos O y O0 .
En cualquier caso, para construir el eje radical de dos circunferencias bastaría con hallar un punto
de igual potencia respecto de ambas circunferencias y por él trazar la recta perpendicular a la línea
que une los centros.

Septiembre, 2014 Curso 20132014.


SEC. 1. CONCEPTOS FUNDAMENTALES 7

Construcción del eje radical

Si las circunferencias son secantes, los puntos de corte tienen potencia nula con respecto a ambas
circunferencias y el eje radical será la recta que pasa por esos dos puntos.

Eje radical

O O0

Si las circunferencias son tangentes, el eje radical será la tangente común.

O O0 O0 O

Eje radical
Eje radical

Si las circunferencias son exteriores podemos trazar una tangente común y el punto medio será del
eje radical. Trazando la perpendicular a la línea que une los centros por este punto obtendremos el
eje radical.

TFM: Problemas sobre Geometría M.C. Rodríguez


8 CAP. I. RELACIONES MÉTRICAS EN LA CIRCUNFERENCIA

Eje radical

O O0

Si las circunferencias son interiores, hay que trazar una circunferencia auxiliar que corte a ambas,
obtener los ejes radicales e y e0 (no paralelos) de la circunferencia auxiliar con cada una de las
dadas. La intersección de ambos ejes radicales nos proporciona un punto del eje que buscamos. Para
terminar basta trazar la recta que pase por ese punto y sea perpendicular a la que une los centros de
las circunferencias dadas.

Eje radical e
e0

f O O0

Septiembre, 2014 Curso 20132014.


SEC. 1. CONCEPTOS FUNDAMENTALES 9

Si las circunferencias son concéntricas no existe eje radical.

Centro radical de tres circunferencias

Si partimos de tres circunferencias, intentamos buscar todos los puntos que tengan la misma potencia
respecto de las tres circunferencias.
Basta hallar los ejes radicales de dos de ellas y buscar su intersección, por ese punto ha de pasar el
tercer eje y es el único punto que cumple la condición pedida. Se llama centro radical de las tres
circunferencias.

Eje radical de C y C 00

Eje radical de C y C 0

C
C0

Eje radical de C 0 y C 00

O O0

Centro radical

C 00
00
O

TFM: Problemas sobre Geometría M.C. Rodríguez


10 CAP. I. RELACIONES MÉTRICAS EN LA CIRCUNFERENCIA

Circunferencias ortogonales
Dos circunferencias, C de centro O y radio r y C 0 de centro O0 y radio r 0 , son ortogonales si se cortan
bajo un ángulo de 90o .

r r0

d O’
O

Las siguientes afirmaciones son equivalentes:


(1) C y C 0 son ortogonales.
(2) Los radios de ambas circunferencias en los puntos de intersección son perpendiculares.
(3) Si llamamos d a la distancia entre los centros se cumple:

d 2 = r 2 + r 02 .

(4) La potencia del centro de cada circunferencia respecto de la otra es su propio radio al cuadrado.

Arco capaz
Un arco capaz es el lugar geométrico (Ver página17) de los puntos desde los que un segmento AB
se ve con el mismo ángulo, es decir, el lugar geométrico de los vértices de los ángulos que tienen la
misma amplitud y abarcan un mismo segmento.
El arco capaz de un segmento AB, de ángulo λ, es un par de arcos de circunferencia, simétricos a
cada lado del segmento AB que contiene los vértices de ángulo λ, unidos por los puntos A y B. El
ángulo que subtiende el segmento AB visto desde el centro del círculo es 2λ.
El más utilizado es el arco capaz con ángulo λ = 90o . Este caso se corresponde con el segundo
teorema de Tales1 , de tal modo que el arco capaz es la circunferencia cuyo diámetro es el segmento
AB.
1
El segundo teorema de Tales afirma que si los tres vértices de un triángulo están sobre una circunferencia dada,
siendo uno de sus lados el diámetro de la circunferencia, entonces el ángulo opuesto a este lado es un ángulo recto.

Septiembre, 2014 Curso 20132014.


SEC. 1. CONCEPTOS FUNDAMENTALES 11

Construcción del arco capaz: Para construir el arco capaz, de ángulo λ, del segmento AB es posible
seguir varios métodos:

(I) Primer método

Se traza un triángulo AP B, tal que un lado es AB y su ángulo opuesto de amplitud λ


(primero dibujamos el ángulo λ). Después trazamos el segmento AB: sus extremos son
dos puntos de los lados del ángulo.
Se trazan las mediatrices del dicho triángulo.
Estas mediatrices se cortan en el punto O, que es el centro del arco capaz buscado.
Bastará con dibujar con el compás un arco de centro O y radio OA.
El punto O es el circuncentro: el centro de la circunferencia circunscrita. Equidista del
vértice y de los puntos A y B.

A B

(II) Segundo método

Se parte únicamente del segmento AB.


Se traza la mediatriz m de dicho segmento.
A continuación se traza la recta r que forme un ángulo λ con el segmento AB, con vértice
en A.
Desde A, se dibujará una segunda recta s perpendicular a la recta r.
El punto de corte O entre la recta s y la mediatriz m es el centro del arco capaz buscado.
Bastará con dibujar con el compás un arco de centro O y radio OA.

TFM: Problemas sobre Geometría M.C. Rodríguez


12 CAP. I. RELACIONES MÉTRICAS EN LA CIRCUNFERENCIA

A B
λ

Por semejanza de triángulos, se deduce que:

El ángulo formado por la recta s y la mediatriz m mide igual que el ángulo λ;

Por tanto, el ángulo con centro en O, conformado por la recta s y la recta simétrica a s, respecto
de la mediatriz m, medirá el doble que el ángulo λ, es decir, AOB medirá 2λ.

Ángulos de la circunferencia
(I) Ángulo inscrito es aquel que tiene su vértice en la circunferencia y sus lados son dos rectas
secantes.
Mide la mitad del arco que lo abarca.

1_
∠AOB = AB.
2

(II) Ángulo semiinscrito: es aquel que tiene su vértice en la circunferencia, un lado secante y el
otro tangente.
Mide la mitad del arco que lo abarca.

1_
∠AOB = AB.
2

(III) Ángulo interior: es aquel que tiene su vértice en un punto interior de la circunferencia y los
lados son dos secantes.

Septiembre, 2014 Curso 20132014.


SEC. 1. CONCEPTOS FUNDAMENTALES 13

Mide la mitad de la suma de las medidas de los arcos que abarcan sus lados y las prolongaciones
de sus lados.
1 _ _
∠AOB = (AB + CD).
2
(IV) Ángulo central: es aquel que tiene su vértice en el centro de la circunferencia y sus lados lo
forman dos radios.
La medida de un arco es la de su ángulo central correspondiente.
_
∠AOB = AB.

(V) Ángulo exterior: es aquel que tiene su vértice fuera de la circunferencia y los lados son dos
secantes o dos tangentes de la circunferencia o uno tangente y otro secante.

1 _ _
∠AOB = (AB − CD).
2

Ángulo inscrito Ángulo semiinscrito Ángulo central


A
O A

B B

O
A
Ángulo interior Ángulo exterior Ángulo exterior
Ángulo exterior O O
A A =D O
D A=D
A

C C
B

B=C

B B

TFM: Problemas sobre Geometría M.C. Rodríguez


14 CAP. I. RELACIONES MÉTRICAS EN LA CIRCUNFERENCIA

Cuadrilátero
Un cuadrilátero está circunscrito a una circunferencia cuando sus lados son tangentes a ella. Desde
otro punto de vista la circunferencia quedará inscrita en el cuadrilátero cuando en cada lado existe
un punto y sólo uno que pertenece a la circunferencia.
Un cuadrilátero está inscrito en una circunferencia cuando sus vértices son puntos de la circunfe-
rencia.

Proposición. 1.5.
En un cuadrilátero inscrito en una circunferencia, los ángulos opuestos son suplementarios, esto es,
(α + γ = β + δ = 180o ).

δ
C

β
B

α
A

DEMOSTRACIÓN. Sean D
byB
b dos ángulos opuestos. Entonces:

b +B
D
1 _ _
b = (CDA + ABC),
2
es decir, la suma de estos dos ángulos opuestos es la mitad de la circunferencia o lo que es lo mismo,
la suma es 180o .
Un razonamiento análogo se puede aplicar para cualquier par de ángulos opuestos del cuadrilátero.
ƒ

Septiembre, 2014 Curso 20132014.


SEC. 1. CONCEPTOS FUNDAMENTALES 15

Teorema. 1.6. (Teorema de Pitot)


En todo cuadrilátero circunscrito a una circunferencia, se cumple que la suma de las longitudes de
dos lados opuestos es igual a la suma de las longitudes de los otros dos lados: AB + C D = BC + AD.

C
B y
x
x y

n
m

A m D
n

DEMOSTRACIÓN. Definimos los segmentos AB, BC, C D, AD:

AB = x + n,
BC = x + y,
C D = y + m,
AD = n + m.

Sumando:
AB + C D = x + n + y + m = x + y + n + m = BC + AD.
ƒ
Se llama trapecio a un cuadrilátero que tiene dos lados paralelos y otros dos que no lo son. En
concreto, un trapecio isósceles es el que tiene los lados no paralelos de igual longitud, dos ángulos
internos agudos y dos obtusos, iguales entre sí, las diagonales son congruentes y la suma de los
ángulos opuestos es 180o .

TFM: Problemas sobre Geometría M.C. Rodríguez


16 CAP. I. RELACIONES MÉTRICAS EN LA CIRCUNFERENCIA

Dos o más puntos son colineales cuando todos están sobre la misma recta, i.e., existe una recta que
pasa por todos los puntos. Son no colineales si al menos uno de los puntos se encuentra fuera de la
recta que definen dos de los restantes.

A
D

B E

C F

Puntos colineales

Puntos no colineales

Proposición. 1.7.
Un polígono convexo es una figura en la que:
1. todos los ángulos interiores miden menos de 180o o π radianes y todas sus diagonales son
interiores.
2. cualquier recta que pase por un lado del polígono convexo deja a todo el polígono completa-
mente en uno de los semiplanos definidos por la recta.
3. cualquier segmento entre dos puntos que estén dentro del mismo está dentro, es decir, el
segmento no corta los lados.
4. todos los vértices apuntan hacia el exterior del polígono.

Todos los triángulos son polígonos convexos. Todos los polígonos regulares son convexos.

Septiembre, 2014 Curso 20132014.


Capítulo II

Lugares geométricos

2. Lugares geométricos
Se denomina lugar geométrico al conjunto de puntos que verifican una determinada propiedad.
(I) En el plano.
Ejemplos de lugares geométricos en el plano.

El lugar geométrico de los puntos que equidistan a otros dos puntos fijos A y B es una
recta o eje de simetría de dichos dos puntos. Si los dos puntos son los extremos de un
segmento AB, dicha recta o lugar geométrico, se llamada mediatriz y es la recta que se
interseca perpendicularmente a AB en su punto medio.
La bisectriz es también un lugar geométrico. Dado un ángulo, la bisectriz cumple la
propiedad de que todos sus puntos equidistan a los lados de dicho ángulo, convirtiéndose
la bisectriz en un caso particular del lugar geométrico que sigue a continuación.
Generalizando la propiedad de equidistancia a dos rectas, obtenemos que la paralela me-
dia es el lugar geométrico de los puntos que equidistan de dos rectas paralelas. Se observa
que, bajo el punto de vista de que las rectas paralelas se cortan en el infinito (se elimina,
pues, la noción de paralelismo), pasa a ser un sinónimo de la bisectriz, donde el ángulo
ha tomado valor nulo. Si, por el contrario, se diferencia el concepto de paralelismo, la
bisectriz vuelve a ser, como se ha dicho antes, un caso particular de esta definición y el
caso de rectas paralelas, con ángulo 0, es disjunto al de las bisectrices (ángulo no nulo).

Secciones cónicas.
Las secciones cónicas pueden ser descritas mediante sus lugares geométricos:

La circunferencia es el lugar geométrico de los puntos cuya distancia a un punto deter-


minado, el centro, es un valor dado (el radio).
La elipse es el lugar geométrico de los puntos tales que la suma de las distancias a dos
puntos fijos, los focos, es una constante dada (equivalente a la longitud del semieje
mayor de la elipse).
18 CAP. II. LUGARES GEOMÉTRICOS

La parábola es el lugar geométrico de los puntos cuya distancia a un foco equivale a su


distancia a una recta llamada directriz.
La hipérbola es el lugar geométrico de los puntos tales que el valor absoluto de la diferen-
cia entre sus distancias a dos puntos fijos, los focos, es igual a una constante (positiva),
que equivale a la distancia entre los vértices.
(II) En el espacio. Figuras geométricas muy complejas pueden ser descritas mediante el lugar geo-
métrico generado por los ceros de una función o de un polinomio. Por ejemplo, las cuádricas
están definidas como el lugar geométrico de los ceros de polinomios cuadráticos.
En general, los lugares geométricos generados por los ceros del conjunto de polinomios reci-
ben el nombre de conjunto algebraico, las propiedades de dichas variedades se estudian en
Geometría algebraica.

2.1. Técnicas habituales para hallar un lugar geométrico


Muchos de los problemas de lugares geoḿetricos salen por más de un método, pero siempre hay que
saber elegir el que resulte más interesante y rápido para revolver un problema concreto.

2.1.1. Método paramétrico

Consite en expresar analíticamente las condiciones expuestas en el enunciado para determinar las
coordenadas (x, y) de un punto genérico del lugar en función de un parámetro que expresa la va-
riación del punto en el lugar.
En este método es importante la elección de la posición de los ejes y la del parámetro.

2.1.2. Método de transformaciones geométricas

Este método consiste en identificar el lugar pedido como la imagen de algún lugar conocido mediante
una transformación geométrica también conocida.
Las transformaciones más usuales son traslaciones, giros, simetrías, homotecias e inversiones.
Para establecer el método se intenta ligar los puntos pedidos con otros que pertenezcan a un lugar
geométrico conocido mediante alguna relación que exprese una transformación de las anteriores o
una sucesión (producto) de ellas.

2.1.3. Método analítico directo

Dicho método consiste en obtener la ecuación implícita del lugar pedido imponiendo a un punto
genérico P(x, y) la condición definitoria expresada en el enunciado.
De nuevo resulta crítica la elección de los ejes.

Septiembre, 2014 Curso 20132014.


SEC. 2. LUGARES GEOMÉTRICOS 19

2.2. Ejemplos

Ejemplo. 2.1.
Un segmento de recta de 10 cm de longitud se mueve apoyando sus extremos en los ejes de coorde-
nadas. Determinar el lugar geométrico descrito por un punto P(x, y) situado sobre el segmento AB
a 4 cm del extremo que se apoya sobre el eje OX , como se muestra en la figura siguiente:
Ver: Referencia Web.

P(x,y)
ϕ
C

X
ϕ
O
D B

SOLUCIÓN. A partir de la figura podemos obtener las funciones trigonométricas:


x
cos ϕ = ,
6
y
sen ϕ = .
4
Estas son las ecuaciones paramétricas del lugar geométrico que estamos buscando, pero necesitamos
transformarlas para que podemos identificar que las dos ecuaciones anteriores representan una sola
curva.
Elevamos al cuadrado las dos ecuaciones:
x2
cos2 ϕ = ,
36

TFM: Problemas sobre Geometría M.C. Rodríguez


20 CAP. II. LUGARES GEOMÉTRICOS

y2
sen2 ϕ = .
16

Sumando miembro a miembro:

x2 y2
+ = sen2 ϕ + cos2 ϕ.
36 16
Por la propiedad de que sen2 ϕ + cos2 ϕ = 1, sustiyendo tenemos:

x2 y2
+ = 1.
36 16
Por tanto, el lugar geométrico descrito por P es una elipse horizontal, con centro en el origen y cuyos
semiejes miden 6 y 4.

Y
A

A P

P P

X
F2 O F1 B
B B

Septiembre, 2014 Curso 20132014.


SEC. 2. LUGARES GEOMÉTRICOS 21

Ejemplo. 2.2.
En una circunferencia C se considera un punto fijo A y uno variable B. Se traza la tangente t a C
en A y se construyen los rombos ABC D que tienen la diagonal AC contenida en t. Hallar el lugar
geométrico del vértice D al variar B en C .
Ver: Referencia Web.

SOLUCIÓN. La recta t, tangente a la circunferencia C en A, contiene a la diagonal AC de los rombos


ABC D. Por tanto, los puntos D son simétricos a los puntos B con respecto a la recta t.
Así, al variar B en C , los puntos D son simétricos de puntos de la circunferencia C, con respecto a t.
Por tanto, el lugar geométrico de D es la circunferencia simétrica, circunferencia lila, de C respecto
a la recta t.

D
B

B D

B D
C

B D

TFM: Problemas sobre Geometría M.C. Rodríguez


22 CAP. II. LUGARES GEOMÉTRICOS

Ejemplo. 2.3.
Determinar el lugar geométrico de los puntos del plano cuyo cociente de distancias a los puntos
PM
M (6, 0) y N (−2, 0) es 3 (es decir, = 3).
PN
Ver: Referencia Web.

SOLUCIÓN. Si P(x, y) es un punto del lugar geométrico, entonces se cumple que:

(x − 6)2 + y 2
p
PM Æ ”Æ —
=3⇒ p =3⇒ (x − 6)2 + y 2 = 3 (x + 2)2 + y 2 .
PN (x + 2)2 + y 2

Elevamos al cuadrado la igualdad, para que las raíces desaparezcan:

(x − 6)2 + y 2 = 9 (x + 2)2 + y 2 ⇒ x 2 − 12x + 36 + y 2 = 9 x 2 + 4x + 4 + y 2


   

⇒ x 2 − 12x + 36 + y 2 = 9x 2 + 36x + 36 + 9 y 2 ⇒ 8x 2 + 8 y 2 + 48x = 0.

Simplificando:
x 2 + y 2 + 6x = 0.

Esta es la ecuacion general de una circunferencia, que es de la forma(I.3). Tenemos que en este caso
concreto D = 6, E = 0 y F = 0. Entonces:

−D −6
a= ⇒= = −3,
2 2

−E 0
b= ⇒= = 0,
2 2
Æ p
r= (a2 + b2 − F ) ⇒ r = 9 + 0 − 0 = 3.

Por tanto, el lugar geométrico de los puntos del plano cuyo cociente de distancias a los puntos M (6, 0)
y N (−2, 0) es 3, es una circunferencia de centro (−3, 0) y radio 3.

Septiembre, 2014 Curso 20132014.


SEC. 2. LUGARES GEOMÉTRICOS 23

-3

TFM: Problemas sobre Geometría M.C. Rodríguez


24 CAP. II. LUGARES GEOMÉTRICOS

Septiembre, 2014 Curso 20132014.


Capítulo III

Problemas y otros desarrollos

3. Problemas de Olimpiadas. Fase Local

Ejercicio. 3.1. (2007-2008, Ver 30 en las Referencias Web)


Sea P una familia de puntos en el plano tales que por cada cuatro puntos de P pasa una circunferen-
cia. ¿Se puede afirmar que necesariamente todos los puntos de P están en la misma circunferencia?
Justifica la respuesta.


SOLUCIÓN. Sea T = x 1 , x 2 , x 3 , x 4 un subconjunto de P con cuatro elementos. Por hipótesis existe
una circunferencia α que pasa por estos cuatro puntos. Supongamos que existe un punto x ∈ P
tal que x ∈ / α. Por la condición del enunciado existe una circunferencia β que pasa por los puntos
x, x 2 , x 3 y x 4 . Entonces las circunferencias α y β tienen tres puntos comunes, lo que implica que
deben coincidir, puesto que como se vio en teoría, una circunferencia queda determinada cuando
conocemos tres puntos de esta (Ver página 1) y si dos circunferencias tienen tres puntos comunes,
estas son la misma. Por tanto, se puede afirmar que necesariamente todos los puntos de P están en
la misma circunferencia.

x1
α

x4

O
x
x2

x3
26 CAP. III. PROBLEMAS Y OTROS DESARROLLOS

Ejercicio. 3.2. (2002-2003, Ver 30 en las Referencias Web)


Dibuja un semicircunferencia con centro en O y diámetro AB y, en su interior, otra, con diámetro OA.
Traza por un punto C de OA una recta perpendicular a dicho radio OA, que cortará a la semicircunfe-
rencia pequeña en D y a la grande en E y, finalmente, la recta AD que cortará a la semicircunferencia
grande en F . Demuestra que el circunferencia circunscrita al triángulo DE F es tangente a la cuerda
AE en E.

SOLUCIÓN. Sea AEB un triángulo rectángulo, ya que el segundo teorema de Tales afirma que si
los tres vértices de un triángulo están sobre una circunferencia dada, siendo uno de sus lados el
diámetro de la circunferencia, entonces el ángulo opuesto a este lado es un ángulo recto. Además la
semicircunferencia de diámetro OA coincide con el arco de capaz de ángulo 90o , entonces E b = 90o .
Por el Teorema del Cateto:1
AE 2 = AC · AB (III.1)

El cuadrilátero BC DF es inscriptible en una circunferencia ya que sus ángulos opuestos son su-
plementarios, es decir, su suma es 180o . En este caso C y F son rectos. Por tanto, su suma es
90o + 90o = 180o . Luego los otros dos ángulos del cuadrilátero sumarán 180o , y por tanto, son
suplementarios, ya que la suma de todos los ángulos de un cuadrilátero es 360o .

1
Teorema del cateto para triángulos se enuncia de la siguiente forma:
El cuadrado de un cateto es igual al producto de la hipotenusa por la proyección del cateto sobre la hipotenusa, es
decir, b2 = na y c 2 = ma.

b c
h

A n m B
a

Septiembre, 2014 Curso 20132014.


SEC. 3. PROBLEMAS DE OLIMPIADAS. FASE LOCAL 27

F
E

A O B
C

Así, las rectas AC B y ADF son secantes a la circunferencia que lo circunscribe.


La potencia del punto A respecto de esa circunferencia nos da:

AC · AB = AD · AF .
(III.1)
Por tanto, AE 2 = AD · AF .
Y esto quiere decir, por potencia de A respecto a la circunferencia que circunscribe al triángulo DE F ,
que la recta AE es tangente a dicha circunferencia en E. ƒ

TFM: Problemas sobre Geometría M.C. Rodríguez


28 CAP. III. PROBLEMAS Y OTROS DESARROLLOS

Ejercicio. 3.3. ([2, página 257])


Dada una circunferencia C y una recta r, hallar todas las circunferencias cuyo eje radical con C sea
r.

SOLUCIÓN. En este ejercicio lo que tenemos que encontrar es el conjunto de circunferencias que
cumplan lo que nos pide el enunciado. A tal conjunto le llamaremos haz de circunferencias y su
posición geométrica depende de la posición relativa de C y r.
Estudiamos entonces cada uno de los casos.

r y C secantes. Toda circunferencia que pase por los puntos de corte de r y C cumple la
condición pedida, luego es del haz.
Si llamamos d a la mediatriz del segmento definido por los puntos de corte de r y C, todas
las circunferencias pedidas tienen centro en la recta d y todo punto de d es centro de una
circunferencia del haz.
Por cada punto M del plano no perteneciente a r pasa una y sólo una circunferencia del haz,
la determinada por M y los dos puntos de corte de r y C.

C M

r y C tangentes. Este caso puede considerarse el caso límite del anterior cuando los dos puntos
de corte de r y C tienden a confundirse.
Como antes, todas las circunferencias del haz son tangentes a r en el mismo punto.
Todas ellas tienen centro en la recta d perpendicular a r por el punto de contacto.
Por cualquier punto M del plano no perteneciente a r pasa una y sólo una circunferencia del
haz cuyo centro se determina por la intersección con d de la mediatriz del segmento que une
M con el punto de contacto.

Septiembre, 2014 Curso 20132014.


SEC. 3. PROBLEMAS DE OLIMPIADAS. FASE LOCAL 29

C M

r exterior a C. En este caso para obtener las circunferencias del haz trazaremos una circunfe-
rencia auxiliar C 0 con centro en P (intersección de r y d) y ortogonal a C.

Cualquier circunferencia con centro en d y ortogonal a C 0 es del haz ya que P tendrá la misma
potencia respecto de C y de ella (el cuadrado del radio de C 0 ).

Todo punto de d exterior al segmento AB es centro de una circunferencia del haz.

A y B se llaman polos del haz.

C’
r

P
A d B

TFM: Problemas sobre Geometría M.C. Rodríguez


30 CAP. III. PROBLEMAS Y OTROS DESARROLLOS

Ejercicio. 3.4. ([2, página 326, Ejercicio 1])


Hallar el lugar geométrico del punto medio de un segmento de longitud a que se apoya continua-
mente en los ejes.

SOLUCIÓN. Tomamos los ejes como se muestra en la siguiente figura:

P(x,y)

α X

O A

Llamamos α al parámetro que representa al ángulo formado por el segmento y el eje OX .


Tenemos que:
OA
cos α = ⇒ OA = a cos α,
a

OB
sen α = ⇒ OB = a sen α.
a
Entonces, las coordenadas de los extremos, y las del punto medio P son, respectivamente:

A(a cos α, 0), B(0, a sen α),

 a cos α
 x =
 2

 y = a sen α

2
Las igualdades anteriores representan las ecuaciones paramétricas del lugar pedido. Para encontrar
la ecuación cartesiana basta eliminar el parámetro.

Septiembre, 2014 Curso 20132014.


SEC. 3. PROBLEMAS DE OLIMPIADAS. FASE LOCAL 31

Elevando al cuadrado, sumando y teniendo en cuenta que sen2 α + cos2 α = 1, nos queda:

a2
x2 + y2 = ,
4
ecuación de una circunferencia de centro el origen y radio la mitad de la longitud a del segmento
dado.

B
P(x,y) P(x,y)

P(x,y) P(x,y)

A A A A X
O

P(x,y) P(x,y)

P(x,y) P(x,y)
B

TFM: Problemas sobre Geometría M.C. Rodríguez


32 CAP. III. PROBLEMAS Y OTROS DESARROLLOS

Ejercicio. 3.5. ([2, página 329, Ejercicio 3])


Dados tres puntos alineados A, B, C se toma un punto variable D sobre la mediatriz de AB. Se traza
la circunferencia que pasa por A, B y D. Se traza la recta C D que corta a la circunferencia en un
segundo punto P. Hallar el lugar geométrico de P al variar D en la mediatriz.

SOLUCIÓN. Tomamos los ejes de modo que el eje OY coincida con la recta dada que contiene a A,
B, C. El eje OX coincide con la mediatriz del segmento AB.
Todas las circunferencias pasan por A y B, luego forman un haz con el eje radical común, que es la
recta definida por A y B.
Como C está en el eje radical común, la potencia de C respecto de cualquier circunferencia es la
misma y por ello:
C P · C D = C t e.
Dicho de otro modo, P es el inverso de D en un inversión de polo C y potencia el valor de la constante.
Como D recorre una recta, el lugar geométrico pedido es la figura inversa a esa recta, que, al no pasar
por el polo, es un circunferencia que pasa por el polo.

Y
C

P P

P
P
B
P P
P
P

O X
D D D D D D D D

Septiembre, 2014 Curso 20132014.


SEC. 3. PROBLEMAS DE OLIMPIADAS. FASE LOCAL 33

Ejercicio. 3.6. ([2, página 331, Ejercicio 4])


Dada una circunferencia y una recta tangente a ella, hallar el lugar geométrico de los centros de las
circunferencias tangentes a ambas.

SOLUCIÓN. En primer lugar tomamos los ejes como se muestra en la figura, coincidiendo la recta
tangente a la circunferencia con el eje OX .

r P(x,y)
y

X
O x

Denotamos x e y a las coordenadas del centro P de una circunferencia variable cualquiera. Es obvio,
que el radio de esta circunferencia es y.
Si llamamos r al radio de la circunferencia dada, se cumple (teorema de Pitágoras):

(r + y)2 = x 2 + (r − y)2 ⇔ x 2 = 4r y.

Esta es la ecuación de una parábola2 de vértice (0, 0), foco F (0, r), ya que en nuestro caso 2p =
2r
4r ⇒ p = 2r, y directriz la recta de ecuación y = − = −r.
2
2
La ecuación general de la parábola es:
(x − a)2 = 2p( y − b).
p p
Tenemos V (a, b) el vértice, F (a, b + ) el foco e y = b − la recta directriz.
2 2

TFM: Problemas sobre Geometría M.C. Rodríguez


p1

34 CAP. III. PROBLEMAS Y OTROS DESARROLLOS

P(x,y)

Recta directriz

Septiembre, 2014 Curso 20132014.


SEC. 3. PROBLEMAS DE OLIMPIADAS. FASE LOCAL 35

Ejercicio. 3.7. (1998-1999, Ver 30 en las Referencias Web)


Sea ABC un triángulo con baricentro G.
(a) Prueba que para cualquier punto del plano M se verifica:
2 2 2 2 2 2
M A + M B + M C ≥ GA + GB + GC ,

obteniéndose la igualdad si y solamente si M = G.


2 2 2
(b) Fijado un número k > GA + GB + GC , halla el lugar geométrico de los puntos M tales que
2 2 2
M A + M B + M C = k.

SOLUCIÓN.
A

M G

B C

2 2 2
En el primer apartado, se trata de minimizar la expresión: f (M ) = M A + M B + M C .
Sean A = (a1 , a2 ), B = (b1 , b2 ) y C = (c1 , c2 ) los vértices del triángulo dado. Para un punto genérico
M = (x, y) se obtiene:

f (x, y) = (x − a1 )2 + ( y − a2 )2 + (x − b1 )2 + ( y − b2 )2 + (x − c1 )2 + ( y − c2 )2

= 3x 2 + 3 y 2 − 2x(a1 + b1 + c1 ) − 2 y(a2 + b2 + c2 ) + a1 2 + a2 2 + b1 2 + b2 2 + c1 2 + c2 2
‹2 ‹2 
a 1 + b 1 + c1 a + b 2 + c2
 
=3 x− + y− 2 + g,
3 3
(III.2)
donde g es una determinada constante real.

TFM: Problemas sobre Geometría M.C. Rodríguez


36 CAP. III. PROBLEMAS Y OTROS DESARROLLOS

Por lo tanto, esta cantidad resultará mínima


 si y sólo si los cuadrados
‹ que aparecen en la expresión
a1 + b1 + c1 a2 + b2 + c2
son cero, es decir, cuando M = (x, y) = , , que corresponde precisamente
3 3
al baricentro G.
Para resolver el segundo apartado es necesario encontrar el lugar geométrico de los puntos M tales
que f (M ) = k. Para ello determinaremos en primer lugar la constante g. Tomando M = G en (III.2)
2 2 2
se obtiene g = f (G) = GA + GB + GC . Por tanto,

f (M ) = k
a 1 + b 1 + c1 2 a 2 + b 2 + c2 2
 ‹  ‹
⇔3 x − + y− +g=k
3 3
‹2 ‹2
a1 + b1 + c1 a + b 2 + c2 k−g
 
⇔ x− + y− 2 = ,
3 3 3
v
tk − g
que es exactamente la ecuación de la circunferencia de centro G y radio r = .
3

M
M

B
C

Septiembre, 2014 Curso 20132014.


SEC. 3. PROBLEMAS DE OLIMPIADAS. FASE LOCAL 37

Ejercicio. 3.8. (2012-2013, Ver 30 en las Referencias Web)


Deslizamos un cuadrado de 10 cm de lado por el plano OX Y de forma que los vértices de uno de
sus lados estén siempre en contacto con los ejes de coordenadas, uno con el eje OX y otro con el eje
OY . Determina el lugar geométrico que en ese movimiento describen:
(a) El punto medio del lado de contacto con los ejes.
(b) El centro del cuadrado.
(c) Los vértices del lado de contacto y del lado opuesto en el primer cuadrante.

SOLUCIÓN. Sean PQRS el cuadrado de lado 10 cm, PQ el lado de apoyo, M (m1 , m2 ) el punto medio
de dicho lado y C(c1 , c2 ) el centro del cuadrado, como se muestra en la figura, además señalamos
los puntos A, B, D y E.

A D
M
X
O B P F
E

(a) Caso del punto medio M .


Tenemos que
1
OM = P M = PQ = 5,
2

TFM: Problemas sobre Geometría M.C. Rodríguez


38 CAP. III. PROBLEMAS Y OTROS DESARROLLOS

luego m1 2 + m2 2 = 25.
Además, sabemos que m1 , m2 ≥ 0, entonces el lugar geométrico sería el cuarto de circunferencia
situado en el primer cuadrante.

R R

S
Q

S
Q
M

M
Q
M

X
O
P P P

(b) Caso del centro del cuadrado C.


Los triángulos AQM , AOM , BM O y DM C son congruentes3

AM = OB = DC,

AQ = OA = M D = BM ,
OM = MQ = M C = 5.
Así, resulta que las coordenadas del centro del cuadrado, en su deslizamiento, son iguales

c1 = OE = OB + BE = m1 + M D = m1 + m2 ,

c2 = EC = E D + DC = OA + AM = m2 + m1 .
Luego, el centro del cuadrado se mueve, en este primer cuadrante, sobre un segmento de la
línea bisectriz. Las posiciones extremas se dan cuando el lado PQ se apoya sobre alguno de los
ejes,pC(5,p
5), y cuando forma una escuadra, esto es, un triángulo rectángulo isósceles, con ellos,
C(5 2, 5 2).
3
Dos figuras son congruentes si tienen la misma forma y tamaño, aunque su posición u orientación sean distintas.

Septiembre, 2014 Curso 20132014.


SEC. 3. PROBLEMAS DE OLIMPIADAS. FASE LOCAL 39

Aplicando un razonamiento análogo en los demás cuadrantes podemos afirmar que el centro
del cuadrado recorre el segmento de sus bisectrices que viene dado por la expresión
p
C(c1 , c2 ) = (±5λ, ±5λ), λ ∈ [1, 2].

R
R

R
S
S
S

C S
Q C
C
C
Q

Q M

X
O= Q
P P P P

(c) Caso de los vértices del cuadrado del lado de contacto: P y Q.


Los vértices P y Q se mueven sobre segmentos de los ejes coordenados, esto es, de las líneas
x = 0 y y = 0.
Los casos extremos se dan cuando el lado de contacto descansa sobre los ejes.
p 
Así, si las coordenadas de uno son (0, λ), las del otro ± 100 − λ2 , 0 y si las coordenadas de
p 
uno son (λ, 0), las del otro son 0, ± 100 − λ2 con λ ∈ [−10, 10].
Caso de los vértices del cuadrado del lado opuesto al lado de contacto: R y S.
Apoyándonos en la figura del principio, podemos ver que los triángulos OQP, QHR y P F S son
congruentes, y a la vez, semejantes a AQM :

R(r1 , r2 )r1 = 2m2 r2 = 2m1 + m2 ,


r2 − r1 r1
de donde m1 = y m2 = .
2 2

TFM: Problemas sobre Geometría M.C. Rodríguez


40 CAP. III. PROBLEMAS Y OTROS DESARROLLOS

Sabemos que m21 + m22 = 25, entonces tenemos para R:


 r − r 2  r 2
2 1 1
+ = 25,
2 2
o bien (r2 − r1 )2 + r1 2 = 100.
Entonces el lugar geométrico esta, pues, en la elipse de ecuación ( y − x)2 + x 2 = 100, (elipse
azul), y es un arco de elipse, (arco azul comprendido entre R y S del cuadrado azul oscuro), que
se puede parametrizar como
Æ p p p
( y − x)2 + x 2 = 100 ⇔ ( y − x)2 = 100 − x 2 ⇔ y − x = 100 − x 2 ⇔ y = x + 100 − x 2 ,
p
con x ∈ [0, 10] e y ∈ [10, 10 2].
Análogamente, para S sale el arco de elipse, (arco rosa comprendido entre S y P del cuadrado
azul oscuro), y 2 + (x − y)2 = 100 con
Æ p p p
y 2 +(x − y)2 = 100 ⇔ (x − y)2 = 100 − y 2 ⇔ x − y = 100 − y 2 ⇔ x = y + 100 − y 2 ,
p
con y ∈ [0, 10] y x ∈ [10, 10 2].

R
R

S
R S
S
C k3 S
Q C
C
C
Q

X
O= Q
P P P P

Septiembre, 2014 Curso 20132014.


SEC. 4. PROBLEMAS DE OLIMPIADAS. FASE NACIONAL 41

4. Problemas de Olimpiadas. Fase Nacional

Ejercicio. 4.1. (Torredolones, 2007, Ver 30 en las Referencias Web)


Dada una semicircunferencia de diámetro AB = 2R, se considera una cuerda C D de longitud fija c.
Sea E la intersección de AC con BD y F la intersección de AD con BC.
Probar que el segmento E F tiene longitud constante y dirección constante al variar la cuerda C D
sobre la semicircunferencia.

A B

SOLUCIÓN. Los triángulos E F C y E DF son rectángulos, entonces el cuadrilátero E DF C es inscripti-


ble (1.5.) y E F es el diámetro.

β α

A O B

TFM: Problemas sobre Geometría M.C. Rodríguez


42 CAP. III. PROBLEMAS Y OTROS DESARROLLOS

Llamamos r al radio de la semicircunferencia de EC D. Por el teorema del seno4 en EC D:


CD c
E F = 2r = = . (III.3)
sen E sen E
Pongamos α = ∠BOD y β = ∠COD. Entonces,

180 − β β
b=
E = 90 − .
2 2
Esta expresión prueba que el ángulo E es constante al serlo β y además el punto E se mueve en el
β
arco capaz de 90 − sobre AB.
2
Sustituyendo en (III.3) nos queda:
c c c
E F = 2r = ‹= = , (III.4)
β β β β

sen 90 − cos sen 90 − sen cos 90 cos
2 2 2 2
por otra parte
β c
=
sen . (III.5)
2 2R
Eliminamos β entre (III.4) y (III.5) y despejamos E F . Para ello, realizamos lo siguiente: despejamos
β
cos en (III.4), elevamos al cuadrado la expresión resultante y (III.5), y sumamos ambas expresio-
2
nes, es decir,
β c
cos = ,
2 EF
β c
sen = ,
2 2R
entonces,

β β c2 c2 c2 c2
cos2 + sen2 = + ⇒ 1 = +
2 2 E F 2 4R2 E F 2 4R2
c2 c2 c2 4R2 − c 2 2cR
⇒ = 1 − ⇒ = ⇒ E F = .
EF2 4R2 EF2 4R2
p
4R2 − c 2
Esta expresión muestra que E F es constante al serlo c y R.
Además F es el ortocentro del triángulo ABE como intersección de las alturas AD y BC, por ello E F
que está sobre la tercera altura es siempre perpendicular a AB. ƒ

4
Si en un triángulo ABC, las medidas de los lados opuestos a los ángulos A, B y C son respectivamente a, b, c,
entonces:
a b c
= = = 2R,
sen A sen B sen C
donde R es el radio de la circunferencia circunscrita.

Septiembre, 2014 Curso 20132014.


SEC. 4. PROBLEMAS DE OLIMPIADAS. FASE NACIONAL 43

Ejercicio. 4.2. (Murcia, 2001, Ver 30 en las Referencias Web)


ABC D es un cuadrilátero inscrito en una circunferencia de radio 1 de modo que AB es un diámetro
y el cuadrilátero admite circunferencia inscrita. Probar que:
p
C D ≤ 2 5 − 4.

SOLUCIÓN.

p C

b
a


β α
A O B

Sea O el centro de la semicircunferencia. Pongamos a = BC, b = AD, p = C D, 2α = ∠BOC, 2β =


∠AOD, 2γ = ∠COD.
La condición necesaria y suficiente para que ABC D admita una circunferencia inscrita (1.6.) es:

p+2= a+ b (III.6)

Puesto que la suma de los ángulos centrales de una semicircunferencia es 180o , en este caso tenemos
que 2α + 2β + 2γ = 180o , entonces
β = 90 − (α + γ).
Además
a = 2 sen α, p = 2 sen γ,

b = 2 sen β = 2[sen 90 cos(α + γ) − sen(α + γ) cos 90]


(III.7)
= 2 cos(α + γ) = 2 cos α cos γ − 2 sen αsen γ.
Vamos a expresar la condición (III.6) en función del ángulo α y el dato p que determina por completo
el cuadrilátero.
 p 2
cos γ + sen γ = 1 ⇒ cos γ = 1 − sen γ ⇒ cos γ = 1 −
2 2 2 2 2
.
2

TFM: Problemas sobre Geometría M.C. Rodríguez


44 CAP. III. PROBLEMAS Y OTROS DESARROLLOS

Entonces, v p
t p2 4 − p2
cos γ = 1 − = ,
4 2
sustituyendo en (III.7), y teniendo en cuenta la siguiente propiedad, obtenemos b,
 p 2 p
sen2 γ = 1 − cos2 γ = 1 − 1 + ⇒ sen γ = ,
2 2
entonces, p 
4 − p2 p p
b=2 cos α − 2 sen α = 4 − p2 cos α − p sen α,
2 2
y sustituyendo en (III.6), queda:
p
p + 2 = 2 sen α + 4 − p2 cos α − p sen α,

o lo que es lo mismo: p
p+2= 4 − p2 cos α + (2 − p) sen α. (III.8)
Por tanto, existirá circunferencia inscrita para los valores de p que hagan compatible la ecuación
(III.8) en la incógnita α.
−→
Interpretando

→ p  (III.8) como el producto escalar de los vectores u (cos α, sen α) de módulo
la ecuación
1y v 4 − p2 , 2 − p . La condición (III.8) queda:



v cos δ = p + 2, (III.9)

siendo δ el ángulo formado por los vectores −



u y−

v.
Para que (III.9) sea compatible debe cumplirse p + 2 ¶ |−

Æ
v | = 4 − p2 + (2 − p)2 , elevando al cua-
drado y operando queda:
p2 + 8p − 4 ¶ 0.
p
Las raíces de la ecuación son p = ±2 5 − 4 . Como p es positivo la condición final es:
p
0 ¶ p ¶ 2 5 − 4.

Septiembre, 2014 Curso 20132014.


SEC. 4. PROBLEMAS DE OLIMPIADAS. FASE NACIONAL 45

Ejercicio. 4.3. (Santander, 2012, Ver 30 en las Referencias Web)


Sea ABC un triángulo acutángulo (tiene todos sus águlos menores de 90o ), ω su circunferencia
inscrita de centro I, Ω su circunferencia circunscrita de centro O y M el punto medio de la altura
AH, donde H pertenece al lado BC. La circunferencia ω es tangente a este lado BC en el punto D.
La recta M D corta a ω en un segundo punto P y la perpendicular desde I a M D corta a BC en N .
Las rectas N R y N S son tangentes a la circunferencia Ω en R y S respectivamente. Probar que los
puntos R, P, D y S están en una misma circunferencia.

SOLUCIÓN.

A
R

P
ω

M
I

O
C N
B D H

Supongamos que b = c. Entonces, el pie de la altura H coincide con el punto de tangencia D, luego
DM es perpendicular a BC y N no está definido.
Asumimos entonces sin pérdida de generalidad que b < c.
Sea U el punto de la recta BC cuya potencia es la misma respecto de ω y Ω. Para obtener este punto,
basta con construir el eje radical de las dos circunferencias y cualquier punto de este eje tendrá igual
potencia respecto a una u otra circunferencia.
Obviamente, hay exactamente dos tangentes a cada una de ambas circunferencias que pasan por U,
siendo D el punto de tangencia de una de ellas con ω. Llamamos E al punto de tangencia con ω de
la segunda recta que pasa por U. La distancia de U a los cuatro puntos de tangencia es la misma,
luego existe una circunferencia de centro U que pasa por los cuatro puntos, es decir, si demostramos
que U = N , el problema quedaría resuelto.
El eje radical de la circunferencia descrita con centro U y ω, es claramente la recta DE y la perpen-
dicular a esta recta por I es la mediatriz de la cuerda DE, luego pasa por U.

TFM: Problemas sobre Geometría M.C. Rodríguez


46 CAP. III. PROBLEMAS Y OTROS DESARROLLOS

Basta entonces con demostrar que el punto W de la altura AH cuya potencia es la misma respecto
a la circunferencia de centro U por D y por E, y respecto a ω es el punto medio de AH, con lo que
sería P = E y N = U. Ahora bien, dicha potencia es

U D2 − UW 2 = I D2 − IW 2

Pero UW 2 = U H 2 + W H 2 , IW 2 = (W H − I D)2 + H D2 , con lo que la anterior condición es equivalente


a
HD · UD
U D2 − 2W H · I D = U H 2 − H D2 = U D(U D − 2H D), WH = ,
ID

y el problema se reduce a demostrar que esta última expresión es la mitad de la altura.


Llamando s al semiperímetro de ABC, tenemos que BD = s − b, C D = s − c, BH = c cos B, y al estar
U definido como el punto sobre BC tal que su potencia es la misma respecto de ω y Ω y llamando
P
al área de ABC y usando la fórmula de Herón5 para la misma, tenemos

U D2 = (U D − BD)(U D + C D),

P2
BD · C D (s − b)(s − c)
UD = = = .
C D − BD b−c s(b − c)(s − a)

Luego


h a (a + b + c) − 2ab − a − c + b
2 2 2
h a (s − b − c cos B)
WH = =
2 (b − c) (s − a) 2 (b − c) (b + c − a)
 (III.10)
h ac − ab − c + b
2 2
h
= = ,
2 (b − c) (b + c − a) 2

que es lo que queríamos demostrar.

5
La fórmula de Herón relaciona el área de un triángulo en términos de las longitudes de sus lados a, b y c:

X Æ
= s (s − a) (s − b) (s − c),

donde s es el semiperímetro del triángulo:

Septiembre, 2014 Curso 20132014.


SEC. 4. PROBLEMAS DE OLIMPIADAS. FASE NACIONAL 47

A
R

P
ω

I M

O
C N
B D H

TFM: Problemas sobre Geometría M.C. Rodríguez


48 CAP. III. PROBLEMAS Y OTROS DESARROLLOS

Ejercicio. 4.4. (Castellón, 1995, Ver 30 en las Referencias Web)


En la figura, AB es un segmento fijo y C un punto variable dentro de él. Se construyen triángulos
equiláteros AC B 0 y C BA0 , de lados AC y CB, respectivamente, en el mismo semiplano definido por
AB, y otro ABC 0 , de lado AB, en el semiplano opuesto. Demuestra:
(a) Las rectas AA0 , BB 0 y C C 0 son concurrentes.
(b) Si llamamos P al punto común a las tres rectas del apartado (a), hallar el lugar geométrico de
P cuando C varía en el segmento AB.
(c) Los centros A00 , B 00 y C 00 de los tres triángulos forman un triángulo equilátero.
(d) Los puntos A00 , B 00 , C 00 y P están sobre una circunferencia.

C0

C 00

C
A B
B 00

A00

P
A0

B0

SOLUCIÓN.

(a) Se traza la circunferencia circunscrita al triángulo ABC 0 y se llama P a la intersección de C C 0


con ella.
Evidentemente (arco capaz) ∠AP B = 120o y P C 0 es su bisectriz con lo que ∠AP C = ∠C P B = 60o
y P ha de estar en las circunferencias circunscritas a los triángulos AC B 0 y BCA0 .
Por tanto las tres circunferencias se cortan en P.

Septiembre, 2014 Curso 20132014.


SEC. 4. PROBLEMAS DE OLIMPIADAS. FASE NACIONAL 49

C’

C”

A C
B
B”

A”
P
A’

B’

Como ∠C P B 0 = 120o y ∠C P B = 60o sumando queda que ∠BP B 0 = 180o y P está alineado con
BB 0 .
Como ∠C PA0 = 120o y ∠C PA = 60o sumando queda que ∠APA0 = 180o y P está alineado con
AA0 .
Por tanto, AA0 , BB 0 y C C 0 son concurrentes y se cortan en el punto P.

(b) Como P está definido por la intersección de la recta C C 0 con la circunferencia circunscrita al
triángulo ABC 0 el lugar pedido es el arco AP B de esa circunferencia.

C’

C C
C
A B

P P P
B’ A’
B’ A’

B’ A’

TFM: Problemas sobre Geometría M.C. Rodríguez


50 CAP. III. PROBLEMAS Y OTROS DESARROLLOS

(c) Los lados del triángulo son perpendiculares a las cuerdas PA, P B y P C que forman ángulos de
60o o 120o . Por ello, entre sí forman ángulos iguales de 60o .

(d) Basta comprobar que los centros C 00 , B 00 , A00 y P verifican el Teorema de Ptolomeo:6

P C 00 · A00 B 00 = PA00 · B 00 C 00 + P B 00 · A00 C 00 ⇔ P C 00 = PA00 + P B 00 ⇔ AB = AC + C B.

La primera implicación es clara, ya que A00 B 00 C 00 forman un triángulo equilátero, y por tanto los
lados A00 B 00 , B 00 C 00 , A00 C 00 son iguales.
La segunda implicación es cierta si se da:
AB AC CB
= = . (III.11)
P C 00 PA00 P B 00
Vamos a demostrar las anteriores igualdades.

(1) A
Ö00 PA = Ø
C 00 P C = C
Ù 00 P C 0 = B
× 00 P B.

(2) Giramos en sentido horario de las agujas del reloj, con centro P, el ángulo A00 PA. Entonces,
P, A00 y A están alineados.
P, C 00 y C están alineados.
P, B 00 y B están alineados.
(3) Entonces, el segmento B 00 C 00 es paralelo a el segmente BC 0 , ya que tenemos dos triángulos
equiláteros con un lado común.
C 00 B 00 y P
(4) Entonces PÙ Ö C 0 B son iguales.
(5) Por tanto, AC
Ö 0P = A Ù 00 C 00 P, y AC
Ö 0 P = AC
Ö 0 P, luego AC
Ö 0C = A 00 C 00 P.
Ù
00 00 00 00
Entonces los triángulos A C P y A C C son semejantes, ya que tienen dos ángulos iguales,
uno es AC
Ö 0C = AÙ00 C 00 P y el otro es C Ö0 AC = 60o = A Ù00 P C 00 .

6
El Teorema de Ptolomeo afirma que en todo cuadrilátero inscribible en una circunferencia, la suma de los productos
de los pares de lados opuestos es igual al producto de sus diagonales, es decir,

AC · BD = AB · C D + BC · AD.
B

Septiembre, 2014 Curso 20132014.


SEC. 4. PROBLEMAS DE OLIMPIADAS. FASE NACIONAL 51

Tendríamos entonces la siguiente situación:

C0
α

C 00
α

60◦ 60◦
A C P A00

Entonces,
AC AC 0 CC0
= = ,
PA00 P C 00 A00 C 00
y como AB = AC 0 , tenemos que:
AC AB CC0
= = .
PA00 P C 00 A00 C 00
Nos quedaría por demostrar la segunda igualdad en (III.11), y así tendríamos probado lo que que-
ríamos.
Para ello, desarrollamos un razonamiento análogo al anterior y llegamos a la situación:

C0

β
C 00

60◦ 60◦
C B P B 00

Entonces,
BC 0 BC CC0
= = ,
P C 00 P B 00 C 00 B 00

TFM: Problemas sobre Geometría M.C. Rodríguez


52 CAP. III. PROBLEMAS Y OTROS DESARROLLOS

y como AB = BC 0 , tenemos que:


AC AB BC
= = .
PA00 P C 00 P B 00
Por tanto, uniendo los resultados de ambos razonamientos, demostramos lo que se quería, es decir,
hemos probado que
AB AC CB
00
= 00
= .
PC PA P B 00
ƒ

Ejercicio. 4.5. (Madrid, 1994, Ver 30 en las Referencias Web)


Sea OX Y Z un triedro trirectángulo de vértice O y aristas X , Y , Z. Sobre la arista Z se toma un punto
fijo C, tal que OC = c. Sobre X e Y se toman respectivamente dos puntos variables P y Q de modo
que la suma OP + OQ sea una constante dada k. Para cada par de puntos P y Q, los cuatro puntos
O, C, P, Q están en una esfera, cuyo centro W se proyecta sobre el plano OX Y . Razonar cuál es el
lugar geométrico de esa proyección. Razonar también cuál es el lugar geométrico de W .

SOLUCIÓN. Sean tres circunferencias resultantes de intersecar la esfera con los planos coordenados.
Las proyecciones del centro W de la esfera sobre estos planos coinciden con los centros de estas
circunferencias (denotados F , G y H) y al ser el triedro trirectángulo, F , G y H están en los puntos
medios de los segmentos PQ, QC y C P que son diámetros de sus circunferencias.

Parametrizando con la distancia OP = λ y teniendo en cuenta que OP + OQ = k y OC = c, tenemos


con respecto a OX Y Z la siguientes coordenadas:

P(λ, 0, 0); Q(0, k − λ, 0); C(0, 0, c);

Septiembre, 2014 Curso 20132014.


SEC. 4. PROBLEMAS DE OLIMPIADAS. FASE NACIONAL 53

λ k−λ k−λ c λ λ k−λ c


 ‹  ‹  ‹  ‹
c
F , , 0 ; G 0, , ;H , 0, ;W , ,
2 2 2 2 2 2 2 2 2
λ k−λ k
Por tanto, el lugar geométrico de F es la recta x + y = + = del plano X OY .
2 2 2
Estudiamos ahora el caso del punto W .
A partir de los datos que se tienen, podemos obtener la siguiente matriz:
 
1
x

 2
 
2 y − k −1
 
 
 c 
z
2
Para que tenga rango 1, se debe cumplir:

1


x
2 = 0 ⇒ −x − y + k = 0 ⇒ x + y = k ,



2 2
2 y − k −1

1


x
2
= 0 ⇒ −z + c = 0 ⇒ z = c .



c
2 4 2
z − 0

2
Por tanto, el lugar geométrico de W es la intersección de los planos:

k
x + y = 2

 c
z =

2
ƒ

TFM: Problemas sobre Geometría M.C. Rodríguez


54 CAP. III. PROBLEMAS Y OTROS DESARROLLOS

Ejercicio. 4.6. (Valladolid, 2010, Ver 30 en las Referencias Web)


Sea P un punto cualquiera de la bisectriz del ángulo A en el triángulo ABC, y sean A0 , B 0 , C 0 puntos
respectivos de las rectas BC, CA, AB, tales que PA0 es perpendicular a BC, P B 0 es perpendicular a CA
y P C 0 es perpendicular a AB. Demuestra que PA0 y B 0 C 0 se cortan sobre la mediana AM , siendo M el
punto medio de BC.

A0
0 M
B

A B
C0

SOLUCIÓN. Sea E el punto de intersección de PA0 y B 0 C 0 . Demostrar que PA0 y B 0 C 0 se cortan sobre
la mediana AM cuando P se mueve sobre la bisectriz AI (I en el incentro) equivale a demostrar que
el lugar geométrico de E cuando P se mueve sobre AI es la mediana AM .

F
A0
B0 M

P
E

B
A C0

Si P se mueve sobre la bisectriz AI (I es el incentro), la figura P B 0 C 0 E es homotética de sí mis-


ma con respecto al punto A. Luego E describe una recta que pasa por A. La bisectriz AI corta a la
circunferencia circunscrita a ABC en F , que se proyecta en el punto medio M de BC.

Septiembre, 2014 Curso 20132014.


SEC. 4. PROBLEMAS DE OLIMPIADAS. FASE NACIONAL 55

Si P = F , la recta B 0 C 0 es la recta de Simson7 de F . Luego el lugar geométrico de E es la mediana


AM .

B0

A0 0
A0
A
B0 M
P

P E
B0
E
P

A B
C0 C0 C0

7
Una recta de Simson en un triángulo es cualquier recta que une los pies de las perpendiculares a los lados del
triángulo, trazadas desde un punto de la circunferencia circunscrita.

TFM: Problemas sobre Geometría M.C. Rodríguez


56 CAP. III. PROBLEMAS Y OTROS DESARROLLOS

Ejercicio. 4.7. (Girona, 2009, Ver 30 en las Referencias Web)


En el interior de una circunferencia de centro O y radio r se toman dos puntos A y B, simétricos
respecto de O. Se considera un punto variable P sobre esta circunferencia y se traza la cuerda P P 0 ,
perpendicular a AP. Sea C el punto simétrico de B respecto de P P 0 . Halla el lugar geométrico del
punto Q, intersección de P P 0 con AC, al variar P sobre la circunferencia.

SOLUCIÓN. Primera solución:

P0

B O A

P 00

Tomamos r = 1 y unos ejes de coordenadas en los que la ecuación de la circunferencia es x 2 + y 2 = 1,


es decir, con centro (0, 0), y las coordenadas de A(a, 0), B(−a, 0), con 0 < a < 1.
Sea P 0 (x 0 , y0 ) punto de la circunferencia, entonces x 0 2 + y0 2 = 1. Por las condiciones del problema
(C simétrico a B), P 0 es el punto medio de BC.
Llamando (x 1 , y1 ) a las coordenadas de C, se tiene:
x 1 = 2x 0 + a,
y1 = 2 y0 .
Entonces la ecuación de la recta8 CA es
x −a y −0
= ⇒ x 0 y − y0 x + y0 a = 0.
2x 0 + a − a 2 y0 − 0
8
Sea r una recta que pasa por dos puntos, A(x 1 , y1 ) y B(x 2 , y2 ). Su ecuación se obtiene:
x − x1 y − y1
=
x2 − x1 y2 − y1

Septiembre, 2014 Curso 20132014.


SEC. 4. PROBLEMAS DE OLIMPIADAS. FASE NACIONAL 57

y0 x0 + a
La pendiente9 de P 0 B es y la de P 0 P es − , ya que las rectas P 0 P y P 0 B forman un ángulo
x0 + a y0
1
recto, entonces la pendiente de P 0 P es − siendo m la pendiente de P 0 B.
m
0
Ahora obtenemos la ecuación de P P.
Sea y = mx + b la ecuación general de una recta donde m es su pendiente. En nuestro caso, tomamos
x0 + a
el punto P 0 (x 0 , y0 ) y la pendiente es − .
y0
Calculamos b:
x0 + a x0 + a
y0 = − x 0 + b ⇒ b = y0 + x0.
y0 y0
Una vez obtenido b, sustituimos en la ecuación general:

x0 + a x0 + a
y =− x + y0 + x 0 ⇔ y0 y = −(x 0 + a)x + y0 2 + x 0 2 + x 0 a
y0 y0
x 0 2 + y0 2 =1
⇔ y0 y = −(x 0 + a)x + 1 + x 0 a ⇔ y0 y + (x 0 + a)x − x 0 a − 1 = 0.
Resolviendo el sistema de ecuaciones:

x 0 y − y0 x + y0 a =0
§
y0 y + (x 0 + a)x − x 0 a − 1 = 0

obtenemos las coordenadas del punto Q, intersección de AC y P 0 P:

x 0 + a y0 (1 − a2 )
 
Q , .
1 + x0 a 1 + x0 a

Denotando por x, y a las coordenadas de Q y despejando los valores de x 0 e y0 se obtiene


a−x −y
x0 = , y0 = .
ax − 1 ax − 1
Imponiendo ahora la condición x 0 2 + y0 2 = 1, se llega a

(a − x)2 y2
+ = 1,
(ax − 1)2 (ax − 1)2

y2
y mediante operaciones se transforma en la ecuación x + = 1, que es la ecuación de una
2
1 − a2
elipse.

9
Pendiente de una recta dados dos puntos, A(x 1 , y1 ) y B(x 2 , y2 ):
y2 − y1
m=
x2 − x1

TFM: Problemas sobre Geometría M.C. Rodríguez


58 CAP. III. PROBLEMAS Y OTROS DESARROLLOS

Segunda solución:
Demostramos primero que dados dos puntos A, B del plano, el conjunto de los puntos P (del mismo
plano) tales que PA2 + P B 2 es constante y mayor o igual que AB 2 , es una circunferencia de centro el
punto medio de AB y que tiene a los puntos A y B en su interior.
Supongamos A = (d, 0), B = (−d, 0) y sea P = (x, y) cualquier punto. Se tiene entonces

PA2 + P B 2 = (x − d)2 + (x + d)2 + ( y − 0)2 + ( y − 0)2 = 2x 2 + 2 y 2 + 2d 2 .

Así que si por hipótesis PA2 + P B 2 = k ≥ AB 2 = (d − (−d))2 = 4d 2 , se tiene que 2x 2 + 2 y 2 + 2d 2 =


k
k ≥ 4d 2 ⇒ x 2 + y 2 = − d 2 ≥ d 2 .
2
Sea R el punto donde BC corta a P P 0 (que es perpendicular a BC). Aplicando el Teorema de Pitógoras,
vemos que el punto R satisface

PR2 = BP 2 − BR2 = AR2 − AP 2 ,

luego R está en la circunferencia y es distinto de P, con lo que R = P 0 .


Ahora bien, se tiene

2 2
AP 2 + BP 0 − (AP − BP 0 )2 k − P P 0 − (AP − BP 0 )2 k − AB 2
AP · BP =
0
= = ,
2 2 2

2 2
donde k = AP 2 + BP 2 = AP 0 + BP 0 .
Además, la potencia de A respecto de la circunferencia es

k k − AB 2
r2 − d2 = − 2d 2 = ,
2 2

con lo que el segundo punto S en el que AP corta a la circunferencia es tal que AS = BP 0 = C P 0 .


Como C P 0 ⊥ P P 0 ⊥ AP se tiene que AS es paralelo a C P 0 y ASP 0 C es un paralelogramo.
Finalmente,

2 2 2 2
P P 0 + PS 2 = AP 0 + AS 2 + 2 · AS · AP = AP 0 + BP 0 + k − AB 2 = 2k − 4d 2 = 4r 2 ,

es decir, P 0 S = AC = 2r. Como AQ + BQ = AC = 2r, el lugar de Q es la elipse interiormente tangente


a la circunferencia dada, con A y B como focos.

Septiembre, 2014 Curso 20132014.


SEC. 4. PROBLEMAS DE OLIMPIADAS. FASE NACIONAL 59

P
P0
C
Q
Q
P

P0

B O A

P0

Q P 00
C

TFM: Problemas sobre Geometría M.C. Rodríguez


60 CAP. III. PROBLEMAS Y OTROS DESARROLLOS

Ejercicio. 4.8. (La Rioja, 2002, Ver 30 en las Referencias Web)


En un triángulo ABC, A0 es el pie de la altura relativa al vértice A y H el ortocentro.
AA0
(a) Dado un número real positivo k tal que = k, encontrar la relación entre los ángulos B y C
HA0
en función de k.
(b) Si B y C son fijos, hallar el lugar geométrico del vértice A para cada valor de k.

SOLUCIÓN.

C0

B0

B
A0 C

Tenemos:
BA0 = c cos B; AA0 = c sen B,
AA0 AA0
0 0
HA HA k c sen B 1
tan H BA0 = 0
= = = = = tan B.
BA c cos B c cos B k cos B k cos B k
1 1
También tenemos que tan B = 0
y por tanto, tan C = . Entonces,
tan H CA tan H BA0
1 1
tan H BA0 = = tan B ⇒ k = tan B · tan C. (III.12)
tan C k

Para el segundo apartado, tomamos a = BC, unos ejes con origen en el punto medio de BC y eje OX
sobre el ladoBC. 
a a 
Por tanto, B − , 0 y C ,0 .
2 2

Septiembre, 2014 Curso 20132014.


SEC. 4. PROBLEMAS DE OLIMPIADAS. FASE NACIONAL 61

Sea A(x, y), entonces la condición (III.12) se escribe:

a2
‹ 
y y
a · a =k⇔ y =k 2
2
−x .
−x +x 4
2 2

Entonces,
x2 y2
+ = 1,
a2 ka2
4 4
es la ecuación de una elipse.
 
x2 y2
Para la ecuación general de la elipse, + 2 = 1 , podemos distinguir dos casos:
a2 b
p
(1) a > b ⇒ a2 = b2 + c 2 ⇒ c = a2 − b2 .

La semidistancia focal es c y el semieje mayor es a.

En este caso se trata de una elipse horizontal.


p
(2) a < b ⇒ b2 = a2 + c 2 ⇒ c = b2 − a2 .

La semidistancia focal es c y el semieje mayor es b.

En este caso se trata de una elipse vertical.

En nuestro caso concreto tenemos:

Si k < 1, estamos en el primer caso, entonces tenemos una elipse horizontal con eje mayor
sobre OX .
v
t a2 ka2 ap
c= − = 1 − k,
4 4 2
a
siendo c la semidistancia focal y semieje mayor .
2

Si k > 1, estamos en el segundo caso, entonces tenemos una elipse vertical con eje mayor sobre
OY ,
v
t ka2 a2 ap
c= − = k − 1,
4 4 2
ap
siendo c la semidistancia focal y semieje mayor k.
2

TFM: Problemas sobre Geometría M.C. Rodríguez


62 CAP. III. PROBLEMAS Y OTROS DESARROLLOS

A F

B B C
A0 F C
F0 A0

F0

Ejercicio. 4.9. (Requena, 2014, Ver 30 en las Referencias Web)


Sean B y C dos puntos fijos de una circunferencia de centro O, que no sean diametralmente opuestos.
Sea A un punto variable sobre la circunferencia, distinto de B y C, y que no pertenece a la mediatriz
de BC. Sean H, el ortocentro del triángulo ABC; y M y N los puntos medios de los segmentos BC
y AH, respectivamente. La recta AM corta de nuevo a la circunferencia en D, y, finalmente, N M
y OD se cortan en un punto P. Determinar el lugar geométrico del punto P cuando A recorre la
circunferencia.

SOLUCIÓN. Consideramos el caso en el que 4ABC es acutángulo. En primer lugar, denotaremos por
A0 el punto diametralmente opuesto a A con lo que los triángulos ACA0 y ABA0 son rectángulos. Los
segmentos H B y CA0 son paralelos por ser perpendiculares a AC. Igualmente, H C y BA0 también son
paralelos por ser perpendiculares a AB.

Septiembre, 2014 Curso 20132014.


SEC
K . 4. P ROBLEMAS DE OLIMPIADAS. FASE NACIONAL 63

A A

N N

O O

H H

C B C B
M M
P

A’
D

Entonces, C H BA0 es un paralelogramo y, por tanto, M es el punto medio de HA0 . Los triángulos AA0 H
y OA0 M son semejantes con razón de semejanza conocida. Es decir, tenemos que
OM M A0 1 AH
= = ⇒ OM = = AN = N H.
AH HA0 2 2
Luego OM H N es otro paralelogramo.
Sea D la intersección de AM con la circunferencia y sea P el punto de corte de OD con N M . Puesto
que 4AOD es isósceles, entonces ∠OAD = ∠ODA. Como OM y AN son paralelos, pues ambos son
perpendiculares al lado BC, además de iguales, entonces AOM N es también un paralelogramo. Y,
de aquí, tenemos que ∠OAM = ∠AM N = ∠P M D por ser opuesto por el vértice.
Sintetizando, tenemos que
∠P M D = ∠OAM = ∠OAD = ∠ODA = ∠P DM ,
con lo que 4P DM es isósceles y, por tanto, P M = P D.
Finalmente, tenemos que
OP + P M = OP + P D = OD = r = Cte.
Es decir, con A variable, el punto P se mueve sobre una elipse incompleta con focos en O y M , y eje
mayor el radio de la circunferencia. En esta elipse hay que descartar los cuatro vértices. En efecto,
si el punto P estuviese sobre el eje mayor de la elipse, tambien estaría D y por tanto A, lo cual está
excluido del enunciado ya que en este caso AD y N M son coincidentes. Si el punto P fuese uno de
r r
los vértices del eje menor de la elipse, tendríamos OP = P M = . Como OD = r, entonces P D = .
2 2
Supongamos que P está del lado de B, entonces la paralela a BM por el punto medio de OB es el eje
menor de la elipse, con lo que el punto medio de OB es precisamente P y D coincide con B, lo que
implicará que A coincide con C, lo cual está excluido del enunciado ya que ABC sería degenerado. El
resto de puntos de la elipse se pueden obtener cuando A es distinto de B y C y no está en la mediatriz
de BC.

TFM: Problemas sobre Geometría M.C. Rodríguez


64 CAP. III. PROBLEMAS Y OTROS DESARROLLOS
K

A
A

N N

M
C B

P
P

D D

Septiembre, 2014 Curso 20132014.


SEC. 5. PROBLEMAS DE OLIMPIADAS. FASE INTERNACIONAL 65

5. Problemas de Olimpiadas. Fase Internacional

Ejercicio. 5.1. (Checoslovaquia,1962, [1, Enunciado, solución: páginas 26, 29, Ejercicio 4.5])
Sean A, B, C puntos distintos de una circunferencia K. Dibujar con regla y compás un cuarto punto
D en la circunferencia K tal que una circunferencia se puede inscribir en el cuadrilátero ABC D.

SOLUCIÓN. Sea AB ≥ BC y D el cuarto punto que se necesita. El punto D pertenece al arco AC, que
no contiene B. A partir de la condición AB + C D = AD + BC (1.6.) se deduce que
AB − BC = AD − C D ≥ 0. (III.13)
Entonces, el punto D pertenece a la circunferencia circunscrita al triángulo ABC y B y D se encuentran
en el mismo lado de la línea bisectriz del segmento AC.

C
A

Sea E un punto del segmento AD tal que DE = DC. Entonces E DC es un triángulo isósceles cuyo
π − D̂ P r op. (1.5.) ∠ABC
ángulo en E es: ∠DEC = = .
2 2
∠ABC
Resulta que ∠AEC = π − , ya que los ángulos AEC y DEC son adyacentes10 , y por tanto
2
suplementarios.
10
Dos ańgulos son adyacentes si tienen un lado en común y los otros dos están en la misma recta.

TFM: Problemas sobre Geometría M.C. Rodríguez


66 CAP. III. PROBLEMAS Y OTROS DESARROLLOS

Obtenemos que E es el punto de intersección de la circunferencia de radio r = AB − BC 11 y centro



A, y la circunferencia que pasa por los puntos A, C, que define el ángulo ∠AEC = π − . Se obtiene
2
de esta manera el punto E. Y a partir de esto, se obtiene obviamente el punto D buscado. ƒ

Ejercicio. 5.2. (Alemania del Este, 1965, [1, Enunciado, solución: páginas 45, 49, Ejercicio 7.6])
Sea n puntos en un plano, n ≥ 3, y d la distancia máxima entre dos puntos de este conjunto. De-
mostrar que a lo sumo n parejas de puntos están situados a una distancia d.

SOLUCIÓN. Sea M el conjunto de puntos dados. Llamamos diámetro de M al segmento X Y de


longitud d, donde X , Y ∈ M . Para cualquier punto X ∈ M , C (X ) es el disco circular de centro X
y radio d. Si AB es un diámetro de M , entonces cualquier punto X ∈ M es un punto interior del
dominio C (A) ∩ C (B) o es un punto de los arcos que definen esto.

A B

Si AB, AD son diámetros del conjunto M , cualquier punto X es un punto interior del dominio C (A) ∩
C (B) ∩ C (C) o un punto de los arcos que lo definen.

11

DE=DC (III.13)
AD = DE + EA ⇒ EA = AD − DE ⇒ EA = AD − DC ⇒ EA = r = AB − BC.

Septiembre, 2014 Curso 20132014.


SEC. 5. PROBLEMAS DE OLIMPIADAS. FASE INTERNACIONAL 67

A D

Supongamos que AB, AD, AE son diámetros del conjunto M y que B, D, E se encuentran en C (A), en
el orden las agujas del reloj. Entonces ar c(BE) ≤ 60o . Aplicando el argumento anterior se deduce
que AD es el único diámetro que termina en D, como se ilustra en el dibujo siguiente.

A D
i

Por tanto, se deben considerar los siguientes casos:


(1) Existe un punto A, A ∈ M , de manera que al menos dos diámetros AB, AD acaban en A. El
conjunto {A, B, D} contiene a lo sumo 3 diámetros y cualquier otro punto E ∈ M contribuye al
conjunto de diámetros con un elemento, como máximo.

TFM: Problemas sobre Geometría M.C. Rodríguez


68 CAP. III. PROBLEMAS Y OTROS DESARROLLOS

(2) Cada punto A, A ∈ M , es un punto final de como máximo un diámetro. Si quitamos el punto D
del diámetro AD te siguen quedando todos los puntos de dentro del dominio, pero estos no se
encuentran a distancia d de A.

El siguiente ejemplo muestra un conjunto M para el que el máximo n es conocido.


Tomamos un triángulo equilátero ABC de lados d y describimos tres circunferencias iguales de radio
d y centros en A, B, C. Tomamos en los arcos AB, BC, AC diferentes puntos en el orden de n − 3.

A
C1 Bq
C2
B2
Cr
B1

B C
A1 A Ap
2

Ejercicio. 5.3. (Rumanía, 1969, [1, Enunciado, solución: páginas 70, 74, Ejercicio 11.4])
Sea AB el diámetro de la semicircunferencia Γ y sea C un punto de Γ , C diferente de A y B. La
proyección perpendicular de C en el diámetro es D. Las circunferencias Γ1 , Γ2 , Γ3 se dibujan de la
siguiente manera: son tangentes a AB, Γ1 está inscrita en el triángulo ABC y Γ2 , Γ3 son ambas tangentes
al segmento C D y Γ . Demostrar que las circunferencias Γ1 , Γ2 , Γ3 , tienen una segunda tangente común.

SOLUCIÓN. Usaremos coordenadas para demostrar este problema.


Sea O el centro de Γ y supongamos que OB es el eje OX . Podemos considerar que:
El radio de Γ es 1.
Las coordenadas de C son C(a, b).
Esto es, OD = a, C D = b y a2 + b2 = 1 (I.2).

Septiembre, 2014 Curso 20132014.


SEC. 5. PROBLEMAS DE OLIMPIADAS. FASE INTERNACIONAL 69

Sean O1 , O2 , O3 los centros de Γ1 , Γ2 , Γ3 respectivamente. Las circunferencias Γ1 , Γ2 , Γ3 tienen una se-


gunda tangente común si y sólo si O1 , O2 , O3 son puntos colineales, ver página 16, y en este caso la
recta tangente requerida es el reflejo de AB en la recta O1 O3 .
Sean r1 , r2 , r3 los radios de Γ1 , Γ2 , Γ3 respectivamente, T2 , T3 los puntos de tangencia de Γ y Γ2 , Γ3 res-
pectivamente y S1 , S2 , S3 las proyecciones de O1 , O2 , O3 en AB, respectivamente.

El centro O2 tiene coordenadas x 2 = a + r2 , y2 = r2 donde r2 se puede obtener usando la condición


rΓ =1
OO2 + O2 T2 = OT2 = 1.

T. Pitágoras
Æ
OO2 2 = r2 2 + (a + r2 )2 ⇒ OO2 = r2 2 + (a + r2 )2 ,

O2 T2 = r2 .

Entonces,
q Æ
(a + r2 )2 + r22 + r2 = 1 ⇒ r2 = −(a + 1) + 2(1 + a).

Otra forma de obtener el valor de r2 sin utilizar la condición anterior, sería aplicando un corolario
del Teorema de Pitágoras12 .

q q Æ
1 − r2 = (a + r2 )2 + r22 ⇒ (a + r2 )2 + r22 + r2 = 1 ⇒ r2 = −(a + 1) + 2(1 + a).

El centro O3 tiene coordenadas x 3 = a − r3 , y3 = r3 donde r3 se puede obtener usando la condición


OO3 + O3 T3 = OT3 = 1.

T. Pitágoras
Æ
OO3 2 = r3 2 + (a − r3 )2 ⇒ OO3 = r3 2 + (a − r3 )2 ,

O3 T3 = r3 .

Entonces,
q Æ
(a − r3 )2 + r32 + r3 = 1 ⇒ r3 = −(1 − a) + 2(1 − a).

12
Del Teorema de Pitágoras se deduce una serie de corolarios, siendo de ayuda uno de ellos en este ejercicio, el cual
establece que:
Æ
h = c1 2 + c2 2 ,

siendo h la hipotenusa de un triángulo rectángulo y c1 , c2 sus respectivos catetos.


En este caso, tomamos OS2 O2 triángulo rectángulo, h = 1 − r2 , c1 = a + r2 y c2 = r2

TFM: Problemas sobre Geometría M.C. Rodríguez


70 CAP. III. PROBLEMAS Y OTROS DESARROLLOS

T3
C

O3
T2
O1
O2

A S3 O S1 D S2 B

El centro O1 tiene coordenadas x 1 = 1 − BS1 , y1 = r1 .


base · al tur a (1 + 1)b
Sea S el área y p el semiperímetro de ∆ABC. Luego S = ⇒S= ⇒S=b=
p 2 2
1 − a2 13 .
Los lados AC, BC se pueden obtener por el Teorema del cateto:14
Æ
AC 2 = AD · AB ⇒ AC 2 = 2(1 + a) ⇒ AC = 2(1 + a),
Æ
BC 2 = BD · AB ⇒ BC 2 = 2(1 − a) ⇒ BC = 2(1 − a).
13
Tomando el triángulo rectángulo ODC y aplicando el Teorema de Pitágoras, tenemos que
p
1 = a2 + b2 ⇒ b = 1 − a2 .

14
Teorema del cateto para triángulos se enuncia de la siguiente forma:
El cuadrado de un cateto es igual al producto de la hipotenusa por la proyección del cateto sobre la hipotenusa, es
decir, b2 = na y c 2 = ma.

b c
h

A n m B
a

Septiembre, 2014 Curso 20132014.


SEC. 5. PROBLEMAS DE OLIMPIADAS. FASE INTERNACIONAL 71

Luego Æ Æ v v
2+ 2(1 + a) + 2(1 − a) t1 + a t1 − a
p= =1+ + .
2 2 2
S
Usando la fórmula r1 = (ya que Γ1 está inscrito en ∆ABC), obtenemos:
p
p v v
1 − a2 t1 + a t1 − a
y 1 = r1 = v v = −1 + + .
t1 + a t1 − a 2 2
1+ +
2 2
O1 es el incentro de ABC. En el dibujo se puede observar que p = AC + BS1 , siendo S1 la proyección
del incentro sobre el lado AB, entonces,
v v v v v v
t1 + a t1 − a Æ t1 + a t1 − a t1 + a t1 − a
BS1 = p−AC = 1+ + − 2(1 + a) = 1− + ⇒ x1 = − .
2 2 2 2 2 2

La condición determinante de puntos colineales puede ser verificada.


x2 + x3 y2 + y3
Podemos probar además que: x 1 = y y1 = .
2 2
Por tanto, O1 es el punto medio del segmente O2 O3 .
En efecto:
x2 + x3 r2 − r3
=a+
2 2 Æ Æ
−(1 + a) + 2(1 + a) + (1 − a) − 2(1 − a)
=a+
v v 2
t1 + a t1 − a
=a−a+ −
2 2
= x1

y 2 + y 3 r2 + r3
=
2 2 Æ Æ
−(1 + a) + 2(1 + a) − (1 − a) + 2(1 − a)
=
v v 2
t1 + a t1 − a
= −1 + +
2 2
= r1

TFM: Problemas sobre Geometría M.C. Rodríguez


72 CAP. III. PROBLEMAS Y OTROS DESARROLLOS

Ejercicio. 5.4. (Rumanía, 1959, [1, Enunciado, solución: páginas 8, 11, Ejercicio 1.6])
Sean P y Q planos que se cortan a lo largo de una recta p. Los puntos A, C se encuentran en P, Q,
respectivamente, pero no en la recta p. Encontrar el punto B en P y D en Q tal que ABC D es un
trapecio isósceles (AB k C D) en el que una circunferencia puede ser inscrita.

SOLUCIÓN. Sean a, c rectas paralelas a p en los planos P, Q, respectivamente, tal que A ∈ a y C ∈ c.


Entonces B ∈ a y D ∈ c, ya que por hipótesis AB k C D. El trapecio ABC D se encuentra en el plano
definido por las rectas a y c.
Una circunferencia está inscrita en ABC D (Mira el Teorema (1.6.).) si y sólo si:

BC=AD despe jando AB C D


AB + C D = BC + AD = 2AD ⇔ + = AD = BC. (III.14)
2 2

a A E B

c
D H F C

Sea H la proyección de A en c y sea E F la bisectriz perpendicular a los segmentos AB y C D. La


condición anterior es equivalente a:

AB C D (III.14)
+ = AE + C F = C H = AD = BC.
2 2
Por lo tanto, los puntos B y D se pueden dibujar con compás, considerando la circunferencia de
centro C y radio C H.
Una solución del problema existe si y sólo si C B > E F . Equivalente, C H > AH, o ∠ADH ≤ 45◦ .
Cuando ocurre la igualdad ∠ADH = 45◦ , el trapecio pasa a ser un cuadrado.

Septiembre, 2014 Curso 20132014.


SEC. 5. PROBLEMAS DE OLIMPIADAS. FASE INTERNACIONAL 73

a A E B

c
D=H F C

Ejercicio. 5.5. (Rumanía, 1959, [1, Enunciado, solución: páginas 7, 9, Ejercicio 1.5])
Sea AB un segmento dado y M un punto del segmento. Sean AM C D y M BE F cuadrados tomados
en el segmento AB. Las circunferencias circunscritas de estos cuadrados tienen centros en P y Q,
respectivamente, y se cruzan entre sí en los puntos M y N .
(a) Demostrar que las rectas AF y BC se cortan en el punto N .
(b) Demostrar que, para cualquier punto M , la recta M N contiene un punto fijo.
(c) Determinar el lugar geométrico del punto medio del segmento PQ cuando el punto M es variable
en el segmento AB.

SOLUCIÓN. Comenzamos demostrando el primer apartado.


Sea N el punto de intersección de AF y BC. Vamos a demostrar que N se encuentra en ambas
circunferencias.
Para demostrar esto, es suficiente comprobar que AN y BN son perpendiculares.
Denotamos:
AB = a,
AM = x,
∠FAM = α,
∠M BN = β.

TFM: Problemas sobre Geometría M.C. Rodríguez


74 CAP. III. PROBLEMAS Y OTROS DESARROLLOS

Entonces

M B = M F = a − x,

a−x
tan α =
x
x
tan β = .
a−x

π π
Resulta que tan α · tan β = 1 y esto implica que α + β = . Entonces, ∠AN B = , ya que la suma
2 2
de los ángulos de un triángulo es 180o .

D C

F N E

A B
M

Otra forma de ver que N se encuentra en ambas circunferencias es estudiando el eje radical. Como
vemos en el dibujo siguiente, el eje radical de dos circunferencias secantes (como es el caso), pasa
por sus dos puntos comunes, los cuales pertenecen a ambas. En este caso pasa por N y M , y por
tanto ambos puntos pertenecen a las dos circunferencias.

Septiembre, 2014 Curso 20132014.


SEC. 5. PROBLEMAS DE OLIMPIADAS. FASE INTERNACIONAL 75

Eje radical

D C

F N E

A B
M

En el segundo apartado, denotamos S como el punto medio del segmento AB.


La mediatriz del segmento AB se corta con la recta M N en el punto T . El ángulo entre las rectas M N
y AB es:

π
∠BM N = 180o − ∠AM N = 180o − (180o − ∠M AN − ∠AN M ) = ∠M AN + ∠AN M = α + .
4
Para entender esta serie de igualdades, hay que tener en cuenta una serie de propiedades:

(I) Los ángulos BM N y AM N son adyacentes15 . Los ángulos adyacentes son suplementarios. Así
tenemos que ∠BM N + ∠AM N = 180o ⇒ ∠BM N = 180o − ∠AM N .

(II) La suma de los ángulos de un triángulo es 180o . De ahí, que ∠AM N = 180o − ∠M AN − ∠AN M .

(III) El ángulo M AN coincide con el ángulo FAM , que por hipótesis es α.

(IV) El ángulo AN M es la mitad del ángulo AN B, que hemos calculado en el primer apartado que
π
era .
2

Tenemos que tan ∠SM T = tan ∠BM N ya que son ángulos opuestos por el vértice.
15
Dos ańgulos son adyacentes si tienen un lado en común y los otros dos están en la misma recta.

TFM: Problemas sobre Geometría M.C. Rodríguez


76 CAP. III. PROBLEMAS Y OTROS DESARROLLOS

Entonces,
π a−x
tan α + tan +1
π 4 tan α + 1 x a
tan ∠SM T = tan(α + ) = π = = = .
4 1 − tan α · tan 1 − tan α 1 − a − x 2x − a
4 x
Por otro lado
ST ST ST 2ST
tan ∠SM T = = = a = .
SM AM − AS x− 2x − a
2
Igualando las dos expresiones anteriores:
n q
a 2ST a
= ⇒ a = 2ST ⇒ ST = .
2x − a 2x − a 2
a
Así que llegamos ST = , y por lo tanto T es un punto fijo.
2

D C

N E

S
A B
M

b1

Otra forma de realizar este apartado sería aplicando el Teorema de la tangente y de la secante, para
cada una de las circunferencias dadas. Tomamos una tangente en cada una de las circunferencias y
la recta secante común a ambas. Aplicando el teorema, tenemos:

(TA)2 = T M · T N ,

(T B)2 = T M · T N .

Septiembre, 2014 Curso 20132014.


SEC. 5. PROBLEMAS DE OLIMPIADAS. FASE INTERNACIONAL 77

Igualando tenemos que TA = T B, entonces el punto T tiene igual potencia respecto a ambas circun-
ferencias y por tanto T es fijo.
El tercer apartado no depende de ninguno de los anteriores.
Sea J el punto medio del segmento PQ. Sea P 0 , Q0 y J 0 las proyecciones perpendiculares en AB de
P, Q y J, respectivamente. Entonces es obvio que J J 0 es la mediana del trapecio PQQ0 P 0 .
a
Luego J J 0 = . Por tanto, el lugar geométrico del punto J cuando M varía en AB, es un segmento
4
a
paralelo a AB a distancia .
4

Q
a/4 P J Q J
P

J0

A B A P0 M B
P0 J0 = M Q0 Q0

P
a/4 J

A P0 J0 M Q0 B

TFM: Problemas sobre Geometría M.C. Rodríguez


78 CAP. III. PROBLEMAS Y OTROS DESARROLLOS

Ejercicio. 5.6. (Rumanía, 1960, [1, Enunciado, solución: páginas 12, 16, Ejercicio 2.5])
Sea ABC DA0 B 0 C 0 D0 un cubo, X un punto variable del segmento AC e Y un punto variable del seg-
mento B 0 D0 .
(a) Encontrar el lugar geométrico del punto medio del segmento X Y .
(b) Encontrar el lugar geométrico del punto Z, Z en el segmento X Y , tal que

Z Y = 2X Z.

SOLUCIÓN. Consideramos el primer caso. Sea Z el punto medio del segmento X Y . Es obvio que si
π, π0 son planos paralelos, X un punto variable en π e Y un punto variable en π0 , entonces el lugar
geométrico del punto medio del segmento X Y es un plano paralelo a la misma distancia de π y π0 .

Π0

X
Π

En nuestro caso, el lugar geométrico está contenido en un plano paralelo con ABC D y A0 B 0 C 0 D0 a la
a
misma distancia, , de estos planos, donde AA0 = a.
2
Fijamos un punto arbitrario X en AC y sea Y un punto variable en el segmento p B 0 D0 . Entonces, el
a 2
lugar geométrico de Z es la línea media del triángulo D0 X B 0 . Tiene longitud . Cuando X se
2

Septiembre, 2014 Curso 20132014.


SEC. 5. PROBLEMAS DE OLIMPIADAS. FASE INTERNACIONAL 79

a
mueve en el segmento AC, la línea media se mueve paralelamente a distancia respecto de las dos
2
bases.
Considerando las posiciones límite, X = A y X = C, concluimos que el lugar geométrico de Z es el
cuadrado cuyos vértices
p están en los centros de las caras laterales del cubo. El lado del cuadrado
a 2
tiene una longitud .
2

C0 B0

D0
A0

B
C

D A

También, es posible dar la solución mediante coordenadas.


Supongamos

A = (0, 0, 0), A0 = (0, 0, 1),


B = (1, 0, 0), B 0 = (1, 0, 1),
C = (1, 1, 0), C 0 = (1, 1, 1),
D = (0, 1, 0), D0 = (0, 1, 1).

Entonces X = (α, α, 0) e Y = (β, 1 − β, 1) donde 0 ≤ α, β ≤ 1.


ZX
Supongamos = c, siendo c una constante real positiva. Entonces Z tiene coordenadas16 :
ZY
α + cβ α + c(1 − β) c
 ‹
Z , , .
1+c 1+c 1+c
16
Las coordenadas de un punto P(x p , y p , z p ) que divide un segmento AB según una razón r son:
xA + r x B y + r yB z + rzB
xp = , yp = A , zp = A
1+r 1+r 1+r

TFM: Problemas sobre Geometría M.C. Rodríguez


80 CAP. III. PROBLEMAS Y OTROS DESARROLLOS

c
Entonces Z pertenece al plano z = . Para precisar el lugar geométrico de Z en este plano
1+c
tenemos que estudiar el problema para el conjunto de puntos

α + cβ α + c(1 − β)
§ ª
M = (x, y)|x = ,y= , 0 ≤ α, β ≤ 1 .
1+c 1+c

El conjunto M sería un rectángulo, como se representa en la figura siguiente:

D C

A B

Este sería un método general, donde c toma valores según la relación entre Z X y Z Y . La resolución
del segundo apartado, se puede obtener siguiendo un razonamiento análogo al método anterior, solo
1
tendríamos que tener en cuenta que c = . ƒ
2

Ejercicio. 5.7. (Alemania del Este, 1965, [1, Enunciado, solución: páginas 44, 47, Ejercicio 7.5])
Sea OAB un triángulo tal que ∠AOB = α, α < 90o . Para cualquier punto M del plano, M 6= 0, P y Q
son los pies de las perpendiculares de M en OA y OB, respectivamente. El punto H es el ortocentro
del triángulo OPQ. Encontrar el lugar geométrico del punto H en los siguientes casos:
(a) M es un punto variable sobre el segmento AB.
(b) M es un punto variable dentro del triángulo AOB.

SOLUCIÓN.
Sean K, L los pies de altura desde A, B, respectivamente, en el triángulo OAB.
Vamos a demostrar que el lugar geométrico del punto H es el segmento K L, cuando M es un punto
variable del segmento AB.

Septiembre, 2014 Curso 20132014.


SEC. 5. PROBLEMAS DE OLIMPIADAS. FASE INTERNACIONAL 81

A
P
L
Q0
K0

M
H

O P0 K Q B
L0

Sean Q0 , K 0 los pies de las perpendiculares de Q, K, respectivamente, en la recta OA y P 0 , L 0 los pies


de las perpendiculares de P, L, respectivamente, en la recta OB.
AM KQ K 0Q 0
Sea = k. Entonces = 0 = k. De ello se deduce que la recta paralela QQ0 a K K 0 en el
MB QB QL
triángulo LK 0 K divide el lado LK con razón k. De la misma manera P P 0 divide LK con razón:

K P0 AP AM
= = = k.
P 0 L0 PL MB
Luego QQ0 y P P 0 intersecan el segmento K L en el mismo punto. Cuando M es uno de los puntos A,
B se obtienen los puntos finales K, L, respectivamente.

A
P
P
PP
L M
H

H M

H
M

H M

O
K Q Q Q Q B

Para la segunda parte del problema, cuando M es un punto interior del triángulo OAB, demostrare-
mos que el lugar geométrico de H es el conjunto de puntos interiores del triángulo OK L.

TFM: Problemas sobre Geometría M.C. Rodríguez


82 CAP. III. PROBLEMAS Y OTROS DESARROLLOS

Sea A0 B 0 el segmento paralelo a AB a través del punto M . Entonces ∆OA0 B 0 es la imagen de ∆OAB bajo
una transformación de homotecia de razón λ17 , λ < 1, y centro O. Cuando M es un punto variable
sobre el segmento A0 B 0 , el lugar geométrico de H es la imagen de LK en virtud de la transformación
considerada. De esta manera obtenemos el interior del triángulo OK L.
A

P
L

P
H

P
M M
H

M
P
H
M H
O
Q B
Q Q Q K

Ejercicio. 5.8. (Rumania, 1959, [1, Enunciado, solución: páginas 7, 9, Ejercicio 1.4])
La hipotenusa AB de un triángulo rectángulo ABC tiene una longitud c y la mediana correspondiente
al vértice C es la media geométrica de los lados AC y BC. Construir el triángulo ABC usando regla
y compás.

SOLUCIÓN. El triángulo está inscrito en una semicircunferencia de diámetro AB, AB = c. Ésta co-
rresponde con el arco capaz de ángulo 90o .
Sea O el centro de la circunferencia y al ser ésta circunscrita a un triángulo, O coincide con el
circuncentro del triángulo inscrito. O es el punto medio del segmento AB, situado a una distancia
c c
, tanto del vértice A como de B. Luego, la mediana OC tiene longitud , ya que el circuncentro se
2 2
encuentra a igual distancia de todos los vértices.
p c c2
Por el enunciado sabemos que ab = , entonces ab = . Usamos la fórmula del área del trián-
2 4
1 1 c
gulo , tenemos que S = ab = ch, donde h es la altura desde el vértice C. Por lo tanto, h = .
18
2 2 4
17
Una homotecia es una transformación afín que, a partir de un punto fijo, multiplica todas las distancias por un
mismo factor. En general una homotecia de razón λ diferente de 1 deja un único punto fijo, llamado centro.
base · al tur a
18
S=
2

Septiembre, 2014 Curso 20132014.


SEC. 5. PROBLEMAS DE OLIMPIADAS. FASE INTERNACIONAL 83

c
Para construir el punto C es suficiente dibujar una línea paralela a AB a una distancia. Corta al
4
arco capaz en dos puntos: C1 y C2 . De esta manera, hemos calculado dos lugares geométricos, es
decir, dos conjuntos de puntos que verifican las condiciones que nos pedía el enunciado. En este
caso son dos triángulos cuyos puntos cumplen las características pedidas.

C1 C2

A B
c/2 O c/2

Ejercicio. 5.9. (Rumanía,1960, [1, Enunciado, solución: páginas 12, 15, Ejercicio 2.3])
Sea ABC un triángulo rectángulo, h la longitud de su altura desde el vértice A (el ángulo recto) y n
un entero positivo impar. La hipotenusa BC tiene longitud a y está divida en n segmentos iguales. El
segmento que contiene el punto medio de BC es visible desde el punto A bajo un ángulo α. Demostrar
que

4nh
tan α = .
(n2− 1)a

SOLUCIÓN. En este caso, el arco capaz de ángulo 90o coincide con la semicircunferencia cuyo diá-
metro es el segmento BC, ya que estamos trabajando con un triángulo rectángulo.
Sea H el pie de la altura desde el vértice A. Por conveniencia introducimos las notaciones BH = x y
∠HAK = β. Es posible determinar x a partir de la ecuación:

h2 = x(a − x). (III.15)

la cual se obtiene por el Teorema de la altura para triángulos rectángulos.19


19
Teorema de la altura. En un triángulo rectángulo, el cuadrado de la altura sobre la hipotenusa es igual al producto
de las proyecciones de los catetos sobre la hipotenusa, es decir, h2 = m · n.

TFM: Problemas sobre Geometría M.C. Rodríguez


84 CAP. III. PROBLEMAS Y OTROS DESARROLLOS

α
β

B O L C
H K

Luego, tan α se puede determinar a partir de los triángulos HAK y HAL. Para ello utilizamos la
fórmula de la tangente de la suma de dos ángulos.20

‹   ‹
LH KH

tan(α + β) − tan β h h h(LH − K H) LH−K H=LK h · LK
tan α = = ‹= 2 = . (III.16)
1 + tan(α + β) tan β h + LH · K H h2 + LH · K H

LH K H
1+ ·
h h

Los segmentos LH, K H y LK están dados por las fórmulas:

α = 90◦

A B
n m

tan α + tan β
20
tan(α + β) = ⇒ tan(α + β) − tan(α + β) tan α · tan β = tan α + tan β.
1 − tan α · tan β
Agrupando términos:
tan(α + β) − tan β
tan α(1 + tan(α + β) tan β) = tan(α + β) − tan β ⇒ tan α = .
1 + tan(α + β) tan β

Septiembre, 2014 Curso 20132014.


SEC. 5. PROBLEMAS DE OLIMPIADAS. FASE INTERNACIONAL 85

a
LK = ,
n
n+1
LH = a − x,
2n
n−1
KH = a − x.
2n
Aplicando estas fórmulas a la ecuación (III.16) tenemos que:

a

4anh
tan α = ‹n  ‹= 2 2
n+1 n−1 4n h + a n − a2 − 4ax n2 + 4n2 x 2
 2 2
h2 + a−x · a−x
2n 2n

(III.15) 4anh 4anh


= = 2
4n2 x(a − x) + − a − 4ax n + 4n x
a2 n2 2 2 2 2 4n x(a − x) + a n2 − a2 − 4ax n2 + 4n2 x 2
2

4nh 4nh
= = .
an2 − a a (n2 − 1)
ƒ

TFM: Problemas sobre Geometría M.C. Rodríguez


Bibliografía

[1] M. Becheanu, International Mathematical Olympiads. 1959–2000, The Academic Distribution


Center, 2001. 5.1., 5.2., 5.3., 5.4., 5.5., 5.6., 5.7., 5.8., 5.9.

[2] Cristóbal Sánchez-Rubio and Manuel Ripollés Amela, Manual de matemáticas para preparación
olímpica, Universitat Jaume I. Castellón, 2000. 3.3., 3.4., 3.5., 3.6.
Refencias Web:

Sobre relaciones métricas en la circunferencia:

1. http://www.sangakoo.com/es/temas/definicion-y-elementos-basicos-de-la-circunferencia

2. http://es.wikipedia.org/wiki/Circunferencia

3. http://www.profesorenlinea.cl/geometria/Ecuacion_Circunferencia.html

4. http://es.slideshare.net/natacha1313/circunferencia-25521537?next_slideshow=1

5. http://es.slideshare.net/sitayanis1/relaciones-metricas-en-la-circunferencia-21712522

6. http://es.wikipedia.org/wiki/Potencia_de_un_punto

7. http://piziadas.com/2012/03/geometria-metrica-arco-capaz-sobre-un-segmento.html

8. http://es.wikipedia.org/wiki/Arco_capaz

9. http://www.dmae.upct.es/~pepemar/angulo/home.htm

10. http://www.vitutor.com/geo/eso/ac_4.html

11. http://matematica.pe/category/cuadrilatero-inscriptible/

12. http://es.wikipedia.org/wiki/Teorema_de_Ptolomeo

13. http://apuntes123.blogspot.com.es/2007/12/puntos-colineales-y-no-colineales.html

14. http://es.wikipedia.org/wiki/Trapecio_(geometría)

15. http://www.ditutor.com/geometria/trapecio_isosceles.html

Sobre ángulos y rectas:

16. http://www.profesorenlinea.cl/geometria/angulos_y_rectas.html

Sobre rectas:

17. http://www.vitutor.com/geo/rec/d_4.html
90 BIBLIOGRAFÍA

18. http://www.vitutor.com/geo/rec/d_7.html

19. http://es.wikipedia.org/wiki/Recta_de_Simson

Sobre triángulos:

20. http://www.uv.es/lonjedo/esoProblemas/3eso14triangulo.pdf

21. http://www.vitutor.com/geo/eso/s_6.html

22. http://ficus.pntic.mec.es/dbab0005/triangulos/Geometria/tema4/Teoremas1.html

Sobre elipses:

23. http://es.wikipedia.org/wiki/Elipse

24. http://www.vitutor.com/geo/coni/g_1.html

25. http://clasesdeapoyonuevo.s3.amazonaws.com/capitulos/apuntes/2.6.2.1.pdf

Sobre lugares geométricos:

26. http://es.wikipedia.org/wiki/Lugar_geometrico

27. http://www.amolasmates.es/pdf/Temas/1BachCT/Lugar%20Geometrico.pdf

28. http://ocw.upm.es/geometria-y-topologia/geometria-de-ayer-y-hoy/contenidos/unidad3/
archivos/ecuacionesparametricasconicas.pdf

29. http://matematica.50webs.com/lugar-geometrico-2.html

Problemas fase local y nacional:

30. http://platea.pntic.mec.es/~csanchez/olimprab.htm

Septiembre, 2014 Curso 20132014.


Índice alfabético

arco capaz elipse, 17


arco capaz, 10
fórmula de Herón, 46
bisectriz, 17 foco, 18
centro, 17 focos, 17
centros del triángulo geometría
circuncentro, 11 lugar geométrico, 17
circunferencia, 1, 17
ángulo central, 13 hipérbola, 18
ángulo exterior, 13 homotecia, 82
ángulo inscrito, 12
ángulo interior, 12 mediatriz, 11, 17
ángulo semiinscrito, 12 parábola, 18
arco, 1 polígono
centro, 1 convexo, 16
centro radical, 9 puntos
cuerda, 1 colineales, 16
diámetro, 1 no colineales, 16
eje radical, 6
haz de circunferencias, 28 radio, 17
inscrita, 14 recta de Simson, 55
ortogonales, 10
polos del haz, 29 semieje mayor, 17
potencia, 4
teorema de la altura, 83
punto de tangencia, 1
teorema de Pitágoras, 69
radio, 1
teorema de Ptolomeo, 50
recta secante, 1
teorema del cateto, 26, 70
recta tangente, 1
trapecio, 15
semicircunferencia, 1
isóceles, 15
conjunto algebraico, 18
triángulo
cuádrica, 18
circunferencia circunscrita, 11
cuadrilátero
circunscrito, 14
inscrito, 14
directriz, 18
Universidad de Granada

Facultad de ciencias

Problemas de olimpiada sobre


geometrı́a: Movimientos y
transformaciones en el plano y en
el espacio

Autor: Tutor:
Carlos López Molina Pascual Jara Martı́nez
Índice
Introducción II

1. Movimientos en el plano 1
1.1. Puntos y elementos dobles . . . . . . . . . . . . . . . . . . . . . . 1
1.2. Traslaciones . . . . . . . . . . . . . . . . . . . . . . . . . . . . . . 1
1.3. Giros . . . . . . . . . . . . . . . . . . . . . . . . . . . . . . . . . . 2
1.4. Simetrı́a central . . . . . . . . . . . . . . . . . . . . . . . . . . . . 2
1.5. Simetrı́a axial . . . . . . . . . . . . . . . . . . . . . . . . . . . . . 2
1.6. Producto de movimientos . . . . . . . . . . . . . . . . . . . . . . 2
1.6.1. Producto de traslaciones . . . . . . . . . . . . . . . . . . . 2
1.6.2. Producto de giros del mismo centro . . . . . . . . . . . . 2
1.6.3. Producto de dos simetrı́as axiales . . . . . . . . . . . . . . 3
1.6.4. Producto de dos giros de diferentes centros . . . . . . . . 3
1.6.5. Producto de traslación por giro . . . . . . . . . . . . . . . 3
1.7. Movimientos directos e inversos . . . . . . . . . . . . . . . . . . . 4
1.8. Congruencia . . . . . . . . . . . . . . . . . . . . . . . . . . . . . . 4
1.9. Ejercicios propuestos . . . . . . . . . . . . . . . . . . . . . . . . . 4

2. Homotecia y semejanza 6
2.1. Homotecia . . . . . . . . . . . . . . . . . . . . . . . . . . . . . . . 6
2.1.1. Propiedades de las homotecias . . . . . . . . . . . . . . . 6
2.2. Semejanzas . . . . . . . . . . . . . . . . . . . . . . . . . . . . . . 7
2.2.1. Propiedades de las semejanzas . . . . . . . . . . . . . . . 7
2.2.2. Centro de semejanza directa . . . . . . . . . . . . . . . . . 7
2.3. Ejercicios propuestos . . . . . . . . . . . . . . . . . . . . . . . . . 8

3. La inversion en el plano 9
3.1. Propiedades de la inversión . . . . . . . . . . . . . . . . . . . . . 9
3.2. Ejercicios propuestos . . . . . . . . . . . . . . . . . . . . . . . . . 10

4. Problemas de la fase local 11

5. Problemas de la fase nacional 19

6. Problemas de la olimpiada internacional 28

Bibliografı́a 38

i
Introducción
En este trabajo se ha intentando crear un documento que sirva para ver la
relación entre la geometrı́a y el álgebra, partiendo de problemas sobre geometrı́a,
se ha utilizado el álgebra para poder demostrarlos.
El texto comienza con unas breves notas sobre la teorı́a, que ayudan a re-
forzar algunos conocimientos e introducen conocimientos necesarios para la re-
solución de problemas. El objetivo de estas notas está destinado a una función
práctica, es decir, no pretenden tener un gran rigor teórico, sino que su objeti-
vo es práctico, ofrecer una serie de resultados y ayudar al lector a comprender
los conceptos necesarios para poder aplicarlos a los problemas relacionados con
estos temas. Se han propuesto una serie de ejercicios en dichas notas para for-
talecer los conocimientos. Estas notas se han dividido en tres temas: Un primer
tema dedicado a movimientos en el plano, un segundo tema dedicado a homo-
tecias y semejanzas y un último y más breve tema dedicado a la inversión en el
plano.
Además de las notas, hay una serie de problemas resueltos de olimpiadas
matemáticas. Están divididos en tres secciones según su dificultad. La primera
sección está destinada a problemas de la fase local, estos problemas son los
de menor dificultad. La segunda sección está dedicada a problemas de la fase
nacional y la tercera sección a problemas de la fase internacional, donde los
ejercicios se han sacado de las short list de la IMO. La mayorı́a de problemas
se resuelven utilizado semejanzas, aunque también hay sobre congruencias y
movimientos en el plano y en menor medida sobre la inversión.
Para la realización de figuras que ayudan a comprender la teorı́a y sirven de
apoyo para la realización de los problemas se ha utilizado el software matemático
geogebra.

ii
1. Movimientos en el plano
En general se denominan movimientos en el plano a las transformaciones del
plano (como conjunto de puntos) en sı́ mismo tales que conservan la alineación y
las distancias. También se conocen como congruencias. Usaremos como notación
la general de transformaciones: mayúsculas para los puntos del plano y funcional
para la transformación, de modo que f(P) designa al punto imagen u homólogo
de P mediante la transformación f.
Hablaremos de transformación o movimiento producto de dos dados f y g en
el sentido habitual fe la composición de funciones f · g(P ) = f (g(P )) que, en
general no es conmutativa.
Para la transformación inversa usaremos f − 1, significando que para todo
punto P:

f (f − 1(P )) = f − 1(f (P )) = P .

La conservación de las distancias garantiza la inyectividad de los movimientos


y por tanto la existencia de movimiento inverso.

1.1. Puntos y elementos dobles


Decimos que un puntos es doble para la transformación f cuando f (P ) = P .
Una idea muy parecida pero conceptualmente diferente es la de elementos
dobles.
Diremos que un subconjunto M del plano es doble cuando:

∀P ∈ M ⇒ f (P ) ∈ M
Evidentemente un conjunto de puntos dobles es un elemento doble pero no a la
inversa. De hecho, M puede ser doble sin que lo sea ninguno de sus puntos.
Una transformación en la que todos los puntos son dobles la llamaremos
identidad, la designaremos por I y en ella I(P)=P para todo punto P.
La determinación de puntos y elementos dobles es con frecuencia muy in-
teresante para el estudio del movimiento.
Admitiremos como algo conocido que los movimientos con la composición
forman un grupo no conmutativo. Por tanto el inverso de todo movimiento es
movimiento.
Es inmediato comprobar que el conjunto de movimientos que deja invariante
una figura dada es un subgrupo del grupo de los movimientos.
Comenzaremos estudiando tres tipos de transformaciones: Traslaciones, giros
y simetrı́as.

1.2. Traslaciones
Puesto que dos puntos definen un único vector libre, es evidente que una
traslación queda definida conociendo un punto y su homologo.
El conjunto de traslaciones del plano forman un grupo conmutativo isomorfo
al de los vectores libres del plano, ya que la traslación producto de dos dadas
es otra traslación asociada al vector suma.
A pesar de su sencillez la traslación es una herramienta muy útil para la
resolución de muchos problemas geométricos.

1
1.3. Giros
Dados un punto O fijo del plano y un ángulos α, llamaremos giro de centro
o y amplitud α a la trasformación definida ası́:

OP = OQ
g(P ) = Q ⇔
∠P OQ = α
El ángulo dado α tiene signo y por ello el giro puede ser en dos sentidos
considerando como es habitual el positivo en sentido contrario al de las agujas
del reloj.
Un giro queda definido conociendo una pareja de puntos P y Q y sus homólo-
gos P’ y Q’, es decir, conociendo un segmento y su imagen mediante el giro.
La amplitud de la rotación coincide con el ángulo que forman ambos seg-
mentos y el centro se determina cortando las mediatrices de los segmentos PP’
y QQ’ formados por cada punto y su homólogo.

1.4. Simetrı́a central


Dado un punto O se llama simetrı́a central de centro O a la transformación
so definida ası́:
so (P ) = Q ⇔ O es el punto medio del segmento PQ.
Claramente es un giro de 180o . Diremos que es involutivo (su cuadrado es la
identidad) y que está determinado por un punto P y su homólogo P’ ya que
permite hallar el centro como el punto medio del segmento PP’.

1.5. Simetrı́a axial


Dada una recta r llamada eje, una simetrı́a axial es la transformación definida
ası́:
sr (P ) = Q ⇔ r es la mediatriz del segmento PQ.
Como todo movimiento, conserva la alineación y la distancia, pero cambia el
sentido. Por ello se dice que es un movimiento inverso.
También es involutivo y queda determinado conociendo un punto y su homólo-
go, quedando el eje definido como la mediatriz del segmento PP’.

1.6. Producto de movimientos


Es especialmente útil para la resolución de problemas estudiar la transfor-
mación resultante de la aplicación sucesiva de dos movimientos.

1.6.1. Producto de traslaciones


Dadas dos traslaciones de vectores u y v es inmediato comprobar que el
producto de ambas es otra traslación de vector u+v.

1.6.2. Producto de giros del mismo centro


Es inmediato que el producto de dos giros del mismo centro O y amplitudes
α y β es otro giro con el mismo centro y amplitud α + β.

2
1.6.3. Producto de dos simetrı́as axiales
Distinguiremos dos casos:
a) Los ejes son paralelos.
Sean dos simetrı́as axiales de ejes r y t paralelos, y llamemos u a un vector
perpendicular a ambos ejes, de módulo doble de la distancia entre ellos y sentido
del primer eje al segundo. Es inmediato demostrar que el producto de las dos
simetrı́as es una traslación de vector u. El producto no es conmutativo, sr st =
−st sr , es decir, los vectores de ambas traslaciones son opuestos.
Es análogo demostrar el proceso inverso. Toda traslación puede descompo-
nerse como producto de dos simetrı́as de ejes paralelos entre sı́.
b) Los ejes no son paralelos.
Dadas dos simetrı́as de ejes r y t que se cortan en el punto O y forman un
ángulo de amplitud α. Es fácil demostrar que el producto de las dos simetrı́as
es un giro de centro O y ángulo 2α. El sentido del giro depende del orden en el
que se apliquen las simetrı́as.
Igual que en el caso anterior podemos invertir el razonamiento: todo giro se
puede descomponer en producto de dos simetrı́as con ejes no paralelos.

1.6.4. Producto de dos giros de diferentes centros


Sea g el giro de centro O y amplitud α y f el giro de centro O’ y amplitud β.
Según se vio en el producto de simetrı́as de ejes no paralelos podemos des-
componer un giro en producto de dos simetrı́as cuyos ejes concurren en el centro
de giro y forman ángulo mitad de la amplitud del giro, pudiendo elegir la direc-
ción de uno de los ejes.
El primer giro g se descompone en producto de las simetrı́as de ejes e1 y e2
siendo e2 la recta OO’ que une los centros y e1 la que forma ángulo α/2 con e2
El segundo giro f se descompone en producto de las simetrı́as de ejes e2 y e3
formando entre ellas ángulo β/2.
Llamando a las simetrı́as por el nombre de sus ejes, obtenemos:

f · g = e3 · e2 · e2 · e1 = e3 · e1
De aquı́ deducimos que el producto de dos giros de diferente centro es un
giro de centro O”(punto de corte de e1 y e3 y amplitud α + β
Si α + β = 360o el producto da como resultado una traslación de vector
perpendicular a la dirección común de e1 y e3 (que son paralelos), y módulo el
doble de la distancia entre ambos ejes.

1.6.5. Producto de traslación por giro


Sea la traslación f de vector PP’ y el giro g de centro O y amplitud α. Des-
componemos la traslación en producto de dos simetrı́as axiales de ejes paralelos
entre sı́ y perpendiculares al vector PP’, e1 y e2 , donde e2 pasa por O. Análo-
gamente descomponemos g en dos simetrı́as axiales e2 y e3 concurrentes en O
y formando un ángulo α/2. Haciendo el producto nos queda:

f · g = e3 · e2 · e2 · e1 = e3 · e1
Con lo cual vemos que el producto nos da un giro de centro 0’ y amplitud
α.

3
1.7. Movimientos directos e inversos
Los movimientos se clasifican en directos e inversos según conserven o in-
viertan el sentido. Un movimiento f es directo si dados tres puntos A,B,C no
alineados y sus homólogos A’,B’,C’, el sentido del ángulo que forman los pri-
meros es igual al que forman los segundos. En caso contrario el movimiento se
denomina inverso.
Los giros y traslaciones son ejemplos de movimientos directos y las simetrı́as
de inversos.
El conjunto de los movimientos directos es un grupo mientras que el conjunto
de los movimientos inversos no lo es, ya que el producto de dos movimientos
inversos es directo.

1.8. Congruencia
Diremos que dos figuras se llaman congruentes si existe algún movimiento
que transforme una en otra.
Teorema 1 Dados dos triángulos ABC y A’B’C’ congruentes, existe un
único movimiento que transforma el primero en el segundo.
Teorema 2 Todo movimiento es producto de a lo más tres simetrı́as axiales.
Teorema 3 Todo movimiento con un punto invariante es producto de dos
simetrı́as como máximo.
Teorema 4 Todo movimiento directo es o una traslación o un giro.
Teorema 5 Todo movimiento inverso es una simetrı́a o una reflexión-deslizamiento.

1.9. Ejercicios propuestos


1. Dado un triangulo ABC, sea M el punto medio del lado AB. Sean r y s
las rectas que pasan por el punto M y son paralelas las rectas BC y AC
respectivamente. Sea N el punto de corte de r con AC y P el punto de corte
de s con BC. Demuestra que los triángulos AMN, MBP, PNM y NPC son
congruentes. ¿Qué relación puedes deducir entre los segmentos MN y BC;
MP y AC; NP y AB?
2. Sea ABC un triángulo y M y N puntos pertenecientes a los lados AC y AB
respectivamente. Halla un punto P sobre el lado BC para que el perı́metro
del triángulo MNP sea mı́nimo. Halla un punto Q sobre el lado BC para
que el perı́metro del triángulo MNP sea mı́nimo.
3. Sea O un puntos y r una recta, O no perteneciente a r. Sea R un punto
variable de la recta r. Se construye el triángulo rectángulo isósceles ROS
siendo ∠ROS = 90o . Calcula el lugar geométrico de S. Encuentra la po-
sición del punto R para que el perı́metro del triangulo ROS sea mı́nimo.
Calcular ese perı́metro en función de la distancia d.
4. Se considera un triangulo ABC y sea O su circuncentro. Demuestra usando
composición de simetrı́as axiales, que el ángulo ∠AOC es el doble del
ángulo formado por las mediatrices de los lados AB y BC.

5. Utiliza argumentos de congruencias de triángulos para probar la construc-


ción con regla y compás del método para dividir un segmento indicando
los movimientos en el plano que generan cada congruencia.

4
6. Utiliza argumentos de congruencias de triángulos para probar la construc-
ción con regla y compás del método para bisecar un ángulo indicando los
movimientos en el plano que generan cada congruencia.

7. Utiliza argumentos de congruencias de triángulos para probar la cons-


trucción con regla y compás del método para dividir un segmento en n
partes iguales indicando los movimientos en el plano que generan cada
congruencia.

5
2. Homotecia y semejanza
Los movimientos o congruencias se definieron como transformaciones pun-
tuales del plano que conservan alineación y distancia.
Si rebajamos la condición de la conservación de la distancia de la siguiente
manera:

f (A)f (B) = k · AB
siendo k una constante, obtenemos la definición de semejanza.

2.1. Homotecia
Dado un punto O cualquiera pero fijo del plano y una constante real k6=0,
llamamos homotecia de centro O y razón k a la transformación que hace corres-
ponder cualquier punto P del plano (distinto de O) el punto P’ alineado con O
y con P de modo que:
OP 0
=k
OP

2.1.1. Propiedades de las homotecias


Las principales propiedades de las homotecias son:
Si k es positivo, P’ está en la semirrecta OP y si k es negativo en la opuesta.
Dos figuras son homotéticas si sus puntos corresponden en una homotecia.
Si k=1, la homotecia es la identidad.

Si k=-1, la homotecia es una simetrı́a central de centro O.


Si k6= 0, no hay puntos dobles exceptuando el propio O, que para todas
las homotecias es punto doble.
La homotecia mantiene la alineación.

La transformación de una recta que no pasa por O es otra recta paralela


a la anterior.
La razón entre dos segmentos homólogos es la razón de homotecia.

Las rectas que pasan por O son los únicos elementos dobles.
Las homotecias conservan los ángulos.
La transformación inversa de una homotecia es otra homotecia de mismo
centro y razón 1/k.

El producto de dos homotecias h y h’ del mismo centro O y razones k y


k’ respectivamente, es una homotecia de mismo centro y razón k•k’. En
consecuencia, las homotecias de centro O forman grupo.

6
2.2. Semejanzas
Se denomina semejanza a la transformación obtenida por el producto de una
homotecia por un movimiento.

2.2.1. Propiedades de las semejanzas


De las propiedades de las homotecias y de los movimientos se deducen las
siguientes propiedades de las semejanzas:

Las semejanzas conservan la alineación.


Los segmentos homólogos son proporcionales.
Las semejanzas conservan los ángulos.
Si el movimiento se directo la semejanza se llama directa, es decir, conserva
el sentido. En caso contrario la semejanza se dice que es inversa.
Si entre dos figuras podemos establecer una correspondencia que cumpla
las tres primeras propiedades se dicen semejantes.
Las semejanzas con el producto de transformaciones forman grupo. Las
semejanzas directas forman subgrupo de estas.
Toda semejanza queda determinada conociendo un par de segmentos orien-
tados homólogos y la clase de semejanza

2.2.2. Centro de semejanza directa


En este apartado vamos a descomponer cada semejanza directa en una homo-
tecia por un movimiento. Para ello vamos a buscar el punto O, que llamaremos
centro de semejanza directa. Distinguiremos dos casos:

Los segmentos que determinan la semejanza son paralelos.


En este caso la intersección de las rectas AA’ y BB’ definen un punto O
que es el centro de homotecia que lleva AB sobre A’B’. En este caso el
movimiento es la identidad.
Los segmentos que determinan la semejanza no son paralelos.
En este caso, nuestro objetivo es determinar un punto O que sea a la vez
centro de un giro y de una homotecia, de manera que el producto nos de
la semejanza que buscamos.
Supongamos que existe O. Por ser una semejanza directa se tiene: ∠OBA =
∠OB 0 A0 ; entonces, si P es la intersección de las rectas definidas por AB y
A’B’ la igualdad anterior se puede poner como:

∠OBP = ∠OB 0 P

Igualdad que establece que O está en la circunferencia definida por los pun-
tos P,B y B’. Análogamente se demuestra que O está en la circunferencia
P, A y A’. Por tanto O queda determinado como el punto de intersección
de las dos circunferencias anteriores que no es P.

7
2.3. Ejercicios propuestos
1. Demostrar que en cualquier triangulo, los puntos medios de uno de los
lados forman un segmento paralelo al tercer lado e igual a la mitad de
este.
2. Demostrar que dos polı́gonos regulares de igual número de lados son se-
mejantes. Hallar la razón entre sus perı́metros, sus apotemas y sus áreas.

3. Demostrar que el centro de la circunferencia circunscrita de un triángulo


rectángulo es el punto medio de la hipotenusa.
4. En todo triángulo las medianas se cortan en proporción 1:2.
5. Sea ABC un triangulo rectángulo con ∠ACB = 90o ny CD⊥AB. Demos-
trar que ∠BAC = ∠BCD.

6. Sea ABC un triángulo rectángulo en A, H es el pie de altura correspon-


diente al vértice A. Utilizando semejanza de triángulos, demuestra que:

AH 2 = BH · HC

7. En el mismo triángulo del ejercicio anterior demuestra que:


a) AB 2 = BH · BC
b)AC 2 = CH · BC
8. Sea γ una circunferencia de centro O y radio R. Sea P un punto interior a
γ y AB y CD dos cuerdas cualesquiera que contienen al punto O. Usando
semejanza de triángulos demuestra que:

P A · P B = P C · P D = OP 2 − R2

8
3. La inversion en el plano
Dada una circunferencia de centro O y radio k, la inversión de centro O y
radio k es una transformación del plano que a cada punto A distinto de O, le
asocia otro punto A’ de la semirrecta OA cumpliendo la relación OA·OA0 = k 2 .
Es fácil ver que un punto exterior a la circunferencia se transforma en un
punto interior y un punto exterior a la circunferencia se transforma en un punto
interior. Los puntos de la circunferencia de inversión se invierten en sı́ mismos,
es decir, son puntos fijos de la transformación.
Es conveniente observar que hay exactamente un punto del plano, el centro de
inversión O, que se queda sin imagen por la transformación. Cuando se trabaja
con inversión se supone que a todos los puntos del plano se le añade un ”punto
ideal.o ”punto del infinitoçon lo que obtenemos el plano inversivo. Dicho punto
ideal será la imagen del centro de inversión.

3.1. Propiedades de la inversión


Las propiedades de la inversión nos permiten hacer demostraciones geométri-
cas que no son sencillas cuando se intentan con otros métodos.

1. Si A y B son puntos distintos y A’ y B’ sus homólogos en una inversión


de centro O y radio k entonces:

AB · k 2
A0 B 0 =
OA · OB

2. Si una recta pasa por el centro de inversión su homóloga es ella misma.


3. Si una recta l no pasa por el centro de inversión O, dicha recta se trans-
forma en una circunferencia con diámetro OM’, siendo M la proyección
ortogonal de O sobre l y M’ el inverso de M. Como se ve en la figura 1.

Figura 1: Recta - Circunferencia

4. Análogamente al caso anterior, si una circunferencia pasa por el centro


de inversión siendo OM’ un diámetro suyo entonces esa circunferencia se

9
transforma en la recta perpendicular a OM’ por el punto M, el inverso de
M’. Como se ve en la figura 1.

5. La inversión de una circunferencia de radio r y centro M que no pasa


por el centro de inversión O, es otra circunferencia que para calcularla
usamos los puntos de corte de la circunferencia con la recta que un O y
M, que llamaremos A y B, Calculando sus homólogos A’ y B’ calculamos
el diámetro de la circunferencia homóloga. La relación entre los radios es:
2
r0 = |OMr·k2 −r2 |

6. La inversión conserva los ángulos.


7. Una inversión deja fija una circunferencia ortogonal a la circunferencia de
inversión.

3.2. Ejercicios propuestos


1. Dados un punto y dos circunferencias, trazar una circunferencia que pase
por el punto y sea tangente a las dos circunferencias.

2. Sea ABC un triángulo y D, E, F los puntos de contacto de la circunfe-


rencia inscrita con los lados BC, CA y AB, respectivamente. Demostrar
que la inversión de la circunferencia inscrita transforma la circunferencia
circunscrita a ABC en la circunferencia de los nueve puntos de DEF.
3. (Teorema de Euler) Si R y r son los radios de las circunferencias circuns-
critas e inscritas a un triángulo y O e I son su circuncentro e incentro,
demostrar que OI 2 = R2 − 2Rr.

10
4. Problemas de la fase local
En esta sección hemos puesto los problemas propuestos en la fase local en
los que en su resolución se utiliza como herramienta movimientos y transforma-
ciones en el plano y en el espacio.

Problema 1(1999)
Prueba que la longitud de los catetos de un triángulo rectángulo isósceles es
siempre igual a la suma de los radios de sus circunferencias inscrita y circuns-
crita.

Solución

Figura 2: Problema 1

Sea ABC un triángulo rectángulo e isósceles con ángulo recto en el vértice


C. Sea O el centro de la circunferencia inscrita, la cual consideramos tangente
a la hipotenusa AB en su punto medio M y al lado AC en el punto P. El radio
de esta circunferencia es r = OP = OM
Por ser esta circunferencia tangente a los lados del triángulo, los radios OM
y OP son perpendiculares a los lados AB y AC en M y P respectivamente.
Por ser el triángulo isósceles, CO (y CM) es bisectriz del ángulo recto en C,
por lo que el triángulo OCP es, también, rectángulo e isósceles (ángulo OCP de
45o ); es decir, OP = P C
Por estar O en la bisectriz del ángulo CAB, los triángulos rectángulos AMO
y APO son iguales (simétricos respecto a la hipotenusa), resultando que AM =
AP
Por otra parte, como el triángulo es rectángulo, el punto M, punto medio de
la hipotenusa, es también el centro de la circunferencia circunscrita a ABC. Por
lo que R = AM , es el radio de esta circunferencia.
En conclusión, la longitud del cateto AC la podemos escribir como:

AC = AP + P C = AM + OP = R + r

11
Problema 2(2000)
Sea P un punto del lado BC de un triángulo ABC. La paralela por P a AB
corta al lado AC en el punto Q y la paralela por P a AC corta al lado AB en el
punto R. La razón entre las áreas de los triángulos RBP y QPC es k 2 .
Determı́nese la razón entre las áreas de los triángulos ARQ y ABC.

Solución

Figura 3: Problema 2

Los triángulos RBP y QPC son semejantes, de razón k. El cuadrilátero


ARPQ es un paralelogramo, y PQ = RA. Si BR = x, entonces:

PQ = RA = kx; BA = (1 + k)x.
Área RBP = S = BR2
PX= xh2
CY = k . PX = kh; CZ = CY + YZ = CY + PX = (1 + k)h
Área ABC = (1 + k)2 S
QT = YZ = PX = h
Área ARQ = AR kxh
2 · Y Z = 2 = kS
areaARQ kS k
areaABC = (1 + k)2 S = (1 + k)2

Problema 3(2003)
Dado un triángulo de vértices A, B y C, y con lados de longitud , y , llamemos
D al punto de intersección del lado AB con la bisectriz del ángulo C. Demuestra
C
2abCos
que: CD = 2
a+b

Solución
A partir del vértice B trazamos una paralela a la bisectriz CD y prolongamos
el lado AC hasta obtener el punto E.

12
Figura 4: Problema 3

Y, también, CF perpendicular a BE
Ası́, CB = CE = a
Por ángulos alternos-internos, en el triángulo BCF tenemos: Cos C2 = FB
a =
EB AC CD
2a . Los triángulos ACD y AEB son semejantes: AE = EB

C
AC · EB 2abCos
CD = = CD = 2
AE a+b

Problema 4(2005)
Se considera un triángulo ABC con ∠ACB = 30o y ∠BAC = 45o . Si M
es el punto medio del lado BC , se pide demostrar que ∠AM B = 45o y que
BC · AC = 2 · AM · AB .

Solución

Figura 5: Problema 4

Sea D el punto de AC tal que BD⊥AC. Puesto que ∠DBA = 90o −∠BAD =
90 − 45o = 45o , el triangulo ADB es isósceles con AD=DB.
o

Como el triangulo CDB es rectángulo en D, CM=MD y, por tanto, ∠CDM =


30o . El teorema del ángulo exterior aplicado ahora al triangulo ACM en M da
inmediatamente ∠AM B = 30o + 15o = 45o .
En consecuencia, los triángulos ABC y MBA son semejantes y, por tener la
misma altura, la razón de sus áreas es igual a la razón de sus bases:

13
AreaABC BC
= =2
AreaM BA BM
√ AC
Por consiguiente, la razón de semejanza vale 2. Tenemos, pues que AM =
√ BC

2 y AB = 2.
La relación que se pide resulta al multiplicar miembro a miembro las dos
igualdades anteriores.

Problema 5(2007)
Demostrar que, en un triángulo, la distancia de un vértice cualquiera al
ortocentro es el doble de la distancia del circuncentro al lado opuesto a ese
vértice.

Solución
Sean A’, B’, C’ los puntos medios de los lados BC, CA y AB respectivamente.
La circunferencia que pasa por A’, B’, C’ (circunferencia medial) es la imagen
de la circunscrita a A, B, C en la semejanza de centro el baricentro G y razón
− 12 .

Figura 6: Problema 5

Obviamente el circuncentro O de ABC es el ortocentro de A’B’C’ y se sigue


el resultado al corresponderse los segmentos AH y A’O en la semejanza anterior.

Problema 6(2008)
En el interior de un paralelogramo ABCD se dibujan dos circunferencias.
Una es tangente a los lados AB y AD, y la otra es tangente a los lados CD
y CB. Probar que si estas circunferencias son tangentes entre sı́, el punto de
tangencia está en la diagonal AC.

14
Figura 7: Problema 6

Solución
Veremos que los puntos A, K y C están alineados.
Sean O1 y O2 los centros de la primera y segunda circunferencia, respecti-
vamente. Notar que AO1 , biseca el ángulo DAB, y análogamente CO2 biseca
el ángulo DCB. Como los lados son paralelos dos a dos y los ángulos O1 AK y
CO2 K son iguales, entonces AO1 es paralelo a CO2 , y, como O1 K y O2 K están
alineados, los ángulos AO1 K y KO2 C son iguales.
Como O1 P ⊥ AB y O1 Q ⊥ CD, los triángulos APO1 y CQO2 son semejantes,
O1 A
por lo que O = OO2Qc , y como O1 P = O1 K y O2 Q = O2 K los triángulos AO1 K
1P 2
y KO2 C son semejantes, por lo que los puntos A, K y C están alineados.

Problema 7(2011)
En un triángulo llamaremos O al circuncentro, I al incentro y r al radio de la
circunferencia inscrita. Si la mediatriz del segmento OI corta a la circunferencia
circunscrita en L, y LI vuelve a cortarla en M, demuestra que IM = 2r

Solución

Figura 8: Problema 7

Por el Teorema de Euler, OI 2 = R2 − 2rR. Sean T y Q los puntos de corte


de la recta OI con la circunferencia circunscrita. Entonces tenemos

15
IL·IM = IT · IQ

Por simetrı́a, IL = OL = R. Por otra parte, IT = OI + OT = OI + R,


y también tenemos IQ = OQ − OI = R − OI. Por tanto, sustituyendo en la
ecuación anterior, se obtiene:

R · IM = (R + OI)(R − OI) = R2 − 2OI 2 = 2rR


de donde IM = 2r.

Problema 8(2011)
Demuestra que en un triángulo se verifica: si r es una recta que pasa por su
baricentro y no pasa por ningún vértice, la suma de las distancias a dicha recta
de los vértices que quedan en un mismo semiplano es igual a la distancia del
tercer vértice a dicha recta.

Solución

Figura 9: Problema 8

El triangulo GGM’ es semejante a GAA’ con razón de semejanza 2(pues


AG=2GM). Por tanto, AA’=2MM’.
Por otro lado, MM’ es la paralela media del trapecio BB’C’C, de donde
MM’=(BB’+CC’)7/2.
En consecuencia: AA’=2MM’=BB’+CC’.

Problema 9(2012)
Sea ABC un triángulo acutángulo con A = 45o , y sea P el pie de la altura
por B. Trazamos la circunferencia de centro P que pasa por C y que vuelve a
cortar a AC en el punto X y a la altura P B en el punto Y . Sean r y s las
rectas perpendiculares a la recta AY por P y X, respectivamente, y L, K las
intersecciones de r, s con AB. Demostrar que L es el punto medio de KB.

16
Solución

Figura 10: Problema 9

Por construcción es PX = PY = PC. Los triángulos PAY y PCB, rectángulos


en P, son iguales ya que AP = P B (el triángulo rectángulo AP B es isósceles) y
PY = PC. Por tanto los ángulos α y β son iguales. El triángulo rectángulo PYQ
es semejante a los anteriores, de manera que el ángulo γ = ∠LP B es igual a α.
Resulta que los segmentos P L y CB son paralelos, y por el teorema de Thales
queda KL=LB ya que PX=PC.

Problema 10(2012)
En un triángulo rectángulo de hipotenusa unidad y ángulos de 30o , 60o y
90o , se eligen 25 puntos cualesquiera. Demuestra que siempre habrá 9 de ellos
que podrán cubrirse con un semicı́rculo de radio 3/10.

Solución

Figura 11: Problema 10

17
Tenemos 3 triángulos y 25 puntos. En algún triángulo habrá al menos 9
puntos.√ La hipotenusa de cada uno de estos triángulos semejantes al inicial
mide 3/3 . Los triángulos son rectángulos y por lo tanto están cubiertos por
la mitad del cı́rculo circunscrito. Esto acaba el problema ya que el radio de este
cı́rculo circunscrito, r, cumple

1 3 3
r= <
2 3 10

18
5. Problemas de la fase nacional
En esta sección hemos puesto los problemas propuestos en la fase nacional
en los que en su resolución se utiliza como herramienta movimientos y transfor-
maciones en el plano y en el espacio.

Problema 1(1994)
El ángulo A del triángulo isósceles ABC mide 2/5 de recto, siendo iguales
sus ángulos B y C. La bisectriz de su ángulo C corta al lado opuesto en el punto
D. Calcular las medidas de los ángulos del triángulo BCD. Expresar la medida
a del lado BC en función de la medida b del lado AC, sin que en la expresión
aparezcan razones trigonométricas.

Solución

Figura 12: Problema 1

Con los datos del enunciado tenemos:


En el triángulo ABC ∠ BAC=36o ; ∠ ABC= ∠ ACB=72o en el triángulo
CBD ∠BCD =36o ;∠CDB=∠BD=72o en el triángulo ADC ∠DAC =∠ACD =
72o ;∠ADC = 108o por tanto los triángulos BCD y ADC son isósceles y además
el triángulo BCD es semejante al triángulo ABC.
Para los lados se tiene: DC = AD = a; BD = b - a. Expresando la propor-
cionalidad derivada de la semejanza anterior:
b−a a a a
= ⇔ a2 = b2 − ab ⇔ a2 + ab − b2 = 0 ⇔ ( )2 + − 1 = 0
a b b b

a 5−1
Y resolviendo queda b = 2 es decir a es la sección áurea de b.

19
Problema 2(1995)
Por el baricentro G de un triángulo ABC se traza una recta que corta al
lado AB en P y al lado AC en Q. Demuestra que:
P B QC 1
· ≤
P A QA 4

Solución

Figura 13: Problema 2

Dupliquemos el triángulo trazando AD paralela a BC y CD paralela a BA


como muestra la figura y tomemos la longitud del lado AB como unidad. Lla-
mando M a la intersección de CD con la recta PQ y x = PB; 1-x = AP, tenemos:
Por semejanza de AQP y QMC: QC MC
QA = AP = 1−x
MC

PB GB 1
Por semejanza de GPB y GMD: M D = GB = 2
Luego: MD = 2x y MC = 1 - 2x. Sustituyendo en el primer miembro de la
relación del enunciado queda:
P B QC
· ≤ 14 ⇔ x(1−2x) 1 2
(1−x)2 ≤ 4 ⇔ 9x − 6x + 1 ≥ 0 ⇔ (3x − 1) ≥ 0
2
P A QA
Relación válida para cualquier x. La igualdad se alcanza para PB=x= 31 ⇔
M C = 13 ⇔PQ paralela al lado BC.

Problema 3(1998)
Se considera el triángulo ABC y su circunferencia circunscrita. Si D y E son
puntos sobre el lado BC tales que AD y AE son, respectivamente, paralelas a
las tangentes en C y en B a la circunferencia circunscrita, demostrar que:

BE AB 2
=
CD AC 2

Solución
Los triángulos ABC y ADC son semejantes pues tienen los tres ángulos
iguales ya que:
∠ADC = ∠BCM = ∠BAC(la primera igualdad por ser AC y CM paralelas
y la segunda por ser ∠BCM ángulo semiinscrito) y el ángulo ∠ACD es común.

20
Figura 14: Problema 3

Estableciendo la proporcionalidad entre sus lados, resulta:


CD AC
= ⇔ CD · BC = AC 2 (1)
AC BC
De modo análogo los triángulos ABC y ABE son semejantes pues: ∠AEB =
∠EBM = ∠BAC y el ángulo ∠ABE es común. Estableciendo la proporcionalidad
entre sus lados, resulta:
BE AB
= ⇔ BE · BC = AB 2 (2)
AB BC
Dividiendo las igualdades (1) y (2) se obtiene el resultado.

Problema 4(2001)
Sea P un punto, en el interior del triángulo ABC, de modo que el triángulo
ABP es isósceles. Sobre cada uno de los otros dos lados de ABC se construyen
exteriormente triángulos BCQ y CAR, ambos semejantes al triángulo ABP.
Probar que los puntos P, Q, C y R o están alineados o son los vértices de un
paralelogramo.

21
Figura 15: Problema 4

Solución
Los triángulos ABC y PBQ son semejantes pues tienen un ángulo igual
∠ABC = ∠PBQ y los lados que lo forman proporcionales:
c BP
=
a BQ
De modo análogo, ABC es semejante a APR , por tanto PBQ y APR son
semejantes (y al ser PB = PA son iguales).
En particular: ∠ARP = ∠ACB y ∠BQP = ∠ACB Llamando α = ∠BAP =
∠ABP , resulta:

∠QP R = 360o −(180o −2α)−(A+B) = 180o +2α−(180o −∠ACB) = 2α+∠ACB

∠QCR = ∠ACB + 2α
∠P RC = 180 − 2α − ∠ARP = 180o − 2α − ∠ACB
o

∠P QC = 180o − 2α − ∠BQP = 180o − 2α − ∠ACB


Las cuatro igualdades establecen que los dos pares de ángulos opuestos del
cuadrilátero PQCR son iguales y es un paralelogramo.
La alineación es un caso particular y se producirá cuando∠ACB+2α = 180o ,
es decir cuando
180o − ∠ACB
α=
2

Problema 5(2004)
ABCD es un cuadrilátero cualquiera, P y Q los puntos medios de las diago-
nales BD y AC respectivamente. Las paralelas por P y Q a la otro diagonal se
cortan en O. Si unimos O con las cuatro puntos medios de los lados X, Y, Z y
T se forman cuatro cuadriláteros, OXBY, OYCZ, OZDT y OTAX. Probar que
los cuatro cuadriláteros tienen la misma área.

22
Figura 16: Problema 5

Solución
Al ser OP paralela a AC, los triángulos OXY, PXY tienen la misma base
e igual altura y por tanto la misma área. De ahı́ que los cuadriláteros OXBY,
PXBY también tienen la misma área, pero el área de PXBY (en amarillo en la
figura) es la cuarta parte del cuadrilátero inicial al ser semejantes con razón 2
del grande al pequeño.

Problema 6(2005)
Diremos que un triángulo es multiplicativo si el producto de las longitudes
de dos de sus lados es igual a la longitud del tercer lado. Sea ABC...XYZ un
polı́gono regular de n lados con todos sus lados de longitud 1. Las n – 3 diago-
nales que salen del vértice A dividen al triángulo ZAB en n – 2 triángulos más
pequeños. Probar que cada uno de esos triángulos es multiplicativo.

Solución
En la figura 6 hemos representado el caso de un octógono ABCDEFGH, pero
el razonamiento es válido para cualquier polı́gono.
Consideremos una inversión con centro A y radio r=1. La circunferencia
circunscrita al polı́gono pasa por el centro de inversión, por lo que su imagen es
una recta, la recta BH que pasa por los puntos de intersección de ambas.
Consideremos los triángulos AUV y ACD, y apliquemos la fórmula que re-
laciona las longitudes de segmentos transformados por una inversión.

r2 UV
1 = CD = U V · =
AU · AV AU · AV
Entonces U V = AU · AV y el triángulo AUV es multiplicativo.

Problema 7(2006)
ABC es un triángulo isósceles con AB = AC. Sea P un punto cualquiera de
la circunferencia tangente a los lados AB en B y a AC en C. Pongamos a, b y c

23
Figura 17: Problema 6

a las distancias desde P a los lados BC, AC y AB respectivamente. Probar que:


a2 = b · c

Solución
Pongamos m = PB; n = PC, Q, R y S las proyecciones de P sobre cada
lado y sea P’ el punto diametralmente opuesto a P . Por la semejanza de los
triángulos PB’ y PBS se tiene:
m 2r
= ⇔ m2 = 2cr(1)
c m
De modo análogo por la semejanza de PCP’ y PBC se cumple:
n 2r
= ⇔ n2 = 2br(2)
b n
Por el teorema de los senos en PBC:
n
sen(∠P BC) =
2r
y en el triángulo rectángulo PQB:
a
sen(∠P BQ) =
m
m 2 n2
de donde a= mn 2
2r ⇔ a = 4r 2 y por (1) y (2) queda finalmente:
a2 = b · c

Problema 8(2007)
Sea O el circuncentro de un triángulo ABC. La bisectriz que parte de A
corta al lado opuesto en P. Probar que se cumple:
2 2 2
AP + OA − OP = bc

24
Figura 18: Problema 7

Solución

Figura 19: Problema 8

Prolongamos AP hasta que corte en M al circuncı́rculo. Los triángulos ABM


y APC son semejantes al tener dos ángulos iguales.(∠ACB =∠ AMB por ins-
critos en el mismo arco y ∠BAN = ∠CAN por bisectriz). Entonces:

c AP
= ⇔ bc = AM · AP
AM b

como AM = AP + P M , queda:
2
bc = AP (AP + P M ) = AP + AP · P M

AP · P M es la potencia de P respecto de la circunferencia circunscrita y su valor

25
2 2
es OA − OP . Sustituyendo llegamos a:
2 2 2
AP + OA − OP = bc

Problema 9(2008)
Dada una circunferencia y en ella dos puntos fijos A, B, otro variable P y una
recta r; se trazan las rectas PA y PB que cortan a r en C y D respectivamente.
Determina dos puntos fijos de r, M y N, tales que el producto CM·DN sea
constante al variar P.

Solución

Figura 20: Problema 9

Trazamos las paralelas a r por A y B que cortan a la circunferencia en


A’ y B’ respectivamente de modo que AA’BB’ es un trapecio isósceles. Las
intersecciones de AB’ y BA’ con r determinan los puntos M y N buscados. En
efecto, los triángulos AMC y DNB (sombreados en la figura) son semejantes ya
que tienen dos ángulos iguales:

∠M AC = ∠B 0 BP = ∠N DB

donde la primera igualdad es cierta por ser ángulos inscritos en el mismo arco
y la segunda por ser BB’ paralela a r.

∠AM C = ∠AB 0 B = ∠DN B

con argumentos análogos a los anteriores.


Estableciendo la proporcionalidad de los lados resulta

AM ND
= ⇔ M C · N D = AM · BN
MC BN
Cantidad que no depende de P. Se observa que si la recta r pasa por el punto A,
M = A = C, no se forma el triángulo AMC. En este caso CM = 0 y el producto
CM · DN = 0, es constante. Análogamente este producto es cero si la recta r
pasa por B o por los puntos A y B en cuyo caso CM = DN = 0

26
Problema 10(2010)
Sea P un punto cualquiera de la bisectriz del ángulo A en el triángulo ABC,
y sean A’, B’, C’ puntos respectivos de las rectas BC, CA, AB, tales que PA’ es
perpendicular a BC, PB’ es perpendicular a CA y PC’ es perpendicular a AB.
Demuestra que PA’ y B’C’ se cortan sobre la mediana AM, siendo M el punto
medio de BC.

Solución
Solución

Figura 21: Problema 10

Sea E el punto de intersección de PA’ y B’C’. Si P se mueve sobre la bisectriz


AI (I es el incentro), la figura PB’C’E es homotética de sı́ misma con respecto
al punto A. Luego E describe una recta que pasa por A. La bisectriz AI corta
a la circunferencia circunscrita a ABC en F, que se proyecta en el punto medio
Am de BC; si P = F, la recta B’C’ es la recta de Simson de F, luego el lugar
geométrico de E es la mediana AAm .

27
6. Problemas de la olimpiada internacional
En esta sección hemos puesto los problemas propuestos en la olimpiada in-
ternacional en los que en su resolución se utiliza como herramienta movimientos
y transformaciones en el plano y en el espacio.

Problema 1(2006)
Sea ABCDE un pentágono convexo tal que ∠BAC = ∠CAD = ∠DAE y
∠ABC = ∠ACD = ∠ADE.
Las diagonales BD y CE se cortan en P. Demuestra que la recta AP divide
al lado CD en dos partes iguales.

Solución

Figura 22: Problema 1

Sea Q la intersección de las diagonales AC y BD, R la intersección de las


diagonales AD y CE y M el punto de corte de AP. Lo que queremos probar
entonces es que CM=MD.
La idea es mostrar que Q y R dividen a AC y AD con la misma proporción,
es decir:
AQ AR
=
QC RD
Las igualdades de ángulos dadas implican que los triángulos ACD, ABC y
ADE son semejantes. Entonces tenemos:
AB AC AD
= =
AC AD AE
Puesto que ∠BAD = ∠BAC + ∠CAD = ∠CAD + ∠DAE = ∠CAE, se
deduce a partir de AB AD
AC = AE que los triángulos ABD y ACE son también
semejantes. Sus bisectrices en A son AQ y AR respectivamente, entonces
AB AQ
=
AC AR
AQ AQ
Porque AB AB AC
AC = AR , obtenemos AC = AD que es equivalente a QC = AR
RD .
Aplicando el teorema de Ceva al triangulo ACD:
AQ CM DR
· · =1
QC M D RA
Lo cual nos lleva a CM=MD, que completa la demostración.

28
Problema 2(2006)
Sea ABCD un trapecio con lados paralelos AB>CD. Sean K y L puntos en los
segmentos AB y CD, respectivamente, tales que AK/KV=DL/LC. Suponemos
que existen los puntos P y Q en el segmento KL tales que

∠AP B = ∠BCD y ∠CQD = ∠ABC

Demostrar que los puntos P, Q, B y C son concı́clicos.

Solución

Figura 23: Problema 2

Como AB k CD, la relación AK/KV=DL/LC implica que las rectas AD,


BC y KL tienen un punto en común S.
Sean X e Y los puntos de intersección de la recta SK con los cı́rculos cir-
cunscritos a ABP y CDQ respectivamente. Puesto que APBX es un cuadrilátero
cı́clico y AB k CD, tenemos

∠AXB = 180o −∠AP B = 180o −∠BCD = ∠ABC

Esto demuestra que BC es tangente al cı́rculo circunscrito al triángulo ABP


en B. Análogamente se demuestra que BC es tangente al cı́rculo circunscrito al
triángulo CDQ. Entonces SP · SX = SB 2 y SQ · SY = SC 2 .
Sea h la homotecia con centro S y razón SC/SB. Como h(B)=C implica que
h transforma el cı́rculo circunscrito al triángulo ABP en el cı́rculo circunscrito
CDQ. También h relaciona AB con CD y fácilmente se demuestra que h(P)=Y,
h(X)=Q, donde SP/SY=SB/SC=SX/SQ.
Las igualdades Sp · SX = SB 2 y SQ/SX=SC/SB implican que SP · SQ =
SB · SC que es equivalente a que P, Q, B y C sean concı́clicos.

29
Problema 3(2007)
En un triángulo ABC, la bisectriz del vértice C es cortada por la circunfe-
rencia circunscrita y por las mediatrices de los lados BC y CA en los puntos R,
P y Q, respectivamente. S y T son los puntos medios de los lados BC y CA,
respectivamente. Demuestra que los triángulos RQT y RPS tienen la misma
área.

Solución

Figura 24: Problema 3

Si AC=BC el triángulo ABC es isósceles, Los triángulos RQT y RPS son


simétricos respecto la bisectriz de C y por lo cual su área es la misma. Por lo cual,
suponemos que AC<BC, sin perder generalidad. Llamamos O a la circunferencia
circunscrita y γ al ángulo de C. Los triángulos rectángulos CTQ y CSP tienen
el mismo ángulo en el vértice C , ya que son semejantes. Además ∠OP Q =
∠OQP = 90o − γ2 . Entonces el triángulo OPQ es isósceles, OP=OQ y además
∠P OQ = γ.
También es conocido que R es el punto medio del arco que une AB y que
∠ROA = ∠BOR = γ.
Consideremos el giro de centro O y ángulo γ. Esta transformación lleva A
en R, R en B y Q en P, entonces los triángulos RQA y BPR son congruentes y
tienen el mismo área.
Los triángulos RQT y RQA tienen a RQ como lado común, ası́ que la pro-
porción entre sus áreas es
area(RQT ) d(T, CR) CT 1
= = =
area(RQA) d(A, CR) CA 2
Análogamente se demuestra que

area(RP S) CS 1
= =
area(BP R) CB 2
Por lo cual
1 1
area(RQT ) = area(RQA) = area(BP R) = area(RP S)
2 2

30
Problema 4(2007)
Las diagonales del trapezoide ABCD cortan en el punto P. El punto Q es el
punto de corte de las paralelas BC y AD tal que ∠AQD = ∠CQB, y la recta
CD separada de los puntos P y Q. Prueba que ∠BQP = ∠DAQ.

Solución

Figura 25: Problema 4

Sea t= AD
BC . Consideramos la homotecia h con centro P y razón -t. Los
triángulos PDA y PBC son semejantes con razón t, entonces h(B)=D y h(C)=A.
Sea Q’=h(Q). Los puntos Q,P y Q’ están alineados. Los puntos Q y P se
encuentran en el mismo lado de AD, ası́ como en el mismo lado de BC. Por lo
tanto Q’ y P están también en el mismo lado de h(BC)=AD, y por lo cual Q y
Q’ están en el mismo lado de AD. Además, los puntos Q y C están en el mismo
lado de BD mientras que Q’ y A están en el lado contrario.
Por la homotecia, ∠AQ0 D = ∠CQD = ∠AQD, por lo cual el cuadrilátero
AQ’QD es cı́clico. Entonces

∠DAQ = ∠DQ0 Q = ∠DQ0 P = ∠BQP

Problema 5(2009)
Sea ABC un triangulo y O su circuncentro. Los puntos P y Q son puntos
interiores de los lados CA y AB respectivamente. La circunferencia k pasa por
los puntos medios de las segmentes BP, CQ y PQ. Prueba que si la recta PQ es
tangente a la circunferencia k entonces OP=OQ.

Solución
Sean K, L, M, B’ y C’ los puntos medios de BP, CQ, PQ, CA y AB respectiva-
mente. Como CAk LM, tenemos que ∠QP A = ∠LM P . Como k toca el segmen-
to PQ en M, obtenemos que ∠LM P = ∠LKM . Entonces ∠QP A = ∠LKM .
De forma similar obtenemos que ∠P QA = ∠KLM gracias a que ABk MK.
Entonces los triángulos APQ y MKL son similares entonces:
QB
AP MK 2 QB
= = PC
=
AQ ML 2
PC

31
Figura 26: Problema 5

De aquı́ obtenemos que AP · P C = AQ · QB que significa que la potencia de


los puntos P y Q respecto de la circunferencia circunscrita al triangulo ABC es
la misma, entonces OP=OQ.

Problema 6(2010)
Sea ABC un triángulo, I su incentro y Γ su circunferencia circunscrita. La
recta AI corta de nuevo a Γ en D. Sean E un punto en el arco formado por BDC
y F un punto en el lado BC tales que
1
∠BAF = ∠CAE < ∠BAC
2
Sea G el punto medio del segmento IF. Demuestre que las rectas DG y EI se
cortan sobre Γ.

Solución
Sea X el segundo punto de intersección de la recta EI con Γ, y L el pie de la
bisectriz del ángulo BAC. Sean G’ y T los puntos de intersección del segmento
DX con las rectas IF y AF respectivamente. Tenemos que probar que G=G’ o
IG’=G’F. Por el teorema de Menelao aplicado al triangulo AIF y la recta DX,
tenemos:
G0 F TF AD TF ID
1= IG0 = AT · ID , or AT = AD

Sea K el punto de intersección de la recta AF con Γ, K 6= A. Entonces


∠BAK = ∠CAE y tenemos que BK = CE y por lo cual, KE k BC. Por otro
lado ∠IAT = ∠DAK = ∠EAD = ∠EXD = ∠IXT ası́ que los puntos I, A,
X, T son cı́clicos. Entonces ∠IT A = ∠IXA = ∠EXA = ∠EKA, por lo cual,
IT k KE k BC. De aquı́ obtenemos que TAT F IL
= AI

32
Figura 27: Problema 6

IL CL
Como CI es la bisectriz del ángulo ACL, tenemos AI = AC . Además ∠DCL =
1
∠DCB = ∠DAB = ∠CAD = 2 ∠BAK, entonces los triángulos DCL y DAC
CL
son semejantes, por lo cual AC = DC
AD . Finalmente, es conocido que el punto
medio D del arco BD es equidistante de los puntos I, B, C, entonces DC ID
AD = AD .
TF ID
Uniendo todas estas igualdades llegamos a la conclusión AT = AD como
querı́amos.

Problema 7(2010)
El punto P se encuentra en el interior del triángulo ABC. Las rectas AP, BP,
CP cortan a la circunferencia circunscrita al triangulo ABC en los puntos K, L,
M, respectivamente. La tangente a la circunferencia circunscrita en C corta a la
recta AB en S. Demostrar que SC=SP si y sólo si MK = ML.

Solución

Figura 28: Problema 7

Asumimos que CA>CB, ası́ que el punto S corta en la recta AB.

33
Por la semejanza de los triángulos PKM y PCA y los triángulos PLM y
PM PA
PCB tenemos que KM = CA y PLM CB
M = P B . Multiplicando estas dos igualdades
tenemos que:
LM CB P A
= ·
KM CA P B
Donde la relación MK=ML es equivalente a CB PA
CA = P B
Llamamos E al pie de la bisectriz del ángulo B en el triangulo ABC. Re-
cordemos que el lugar geométrico de los puntos X tales que XB CA
XA = CB es la
circunferencia de Apolonio Ω con centro Q en la recta AB, y pasa por los pun-
tos C y E. Entonces tenemos que MK=ML si y solo si P esta en Ω, es decir,
QP=QC.
Ahora probaremos que S=Q resolviendo ası́ el problema. Tenemos que ∠CES =
∠CAE + ∠ACE = ∠BCS +∠ECB = ∠ECS ası́ que SC=SE. Entonces el pun-
to S se encuentra en el punto de corte de la recta AB con la mediatriz de CE y
por tanto coincide con Q.

Problema 8(2011)
Sea ABC en triangulo acutángulo con circunferencia circunscrita Ω. Sea B0
el punto medio del AC y C’ el punto medio de AB. Sea B el pie de altura de A
y G el baricentro del triangulo ABC. Sea ω una circunferencia que pasa por B0
y C’ que es tangente a Ω en el punto X 6= A. Demuestra que los puntos D, G y
X están alineados.

Solución

Figura 29: Problema 8

Si AB=AC, la demostración es trivial. Supondremos sin perdida de que


generalidad que AB<AC. Llamemos a las tangentes de Ω en los puntos A y X,
a y x respectivamente.

34
Ω1 la circunferencia circunscrita al triangulo AB0 C0 . Las circunferencias Ω
y Ω1 son homotéticas con centro A, ası́ que la tangente a A y a es su eje radical.
Las rectas a, x y B0 C0 son los tres ejes radicales de las tres circunferencias Ω,
Ω1 y ω. Entonces estas tres rectas son concurrentes en un punto W.
Los puntos A y D son simétricos respecto de la recta B0 C0 , por lo cual
WX=WA=WD. Esto significa que W es el centro de la circunferencia circuns-
crita γ del triangulo ADX. Además, ∠W AO = ∠W XO = 90o , donde O denota
el centro de Ω. Por lo cual ∠AW X + ∠AOX = 180o .
Sea T la segunda intersección de Ω y la recta DX. Notemos que O pertenece a
Ω1 . Usando las circunferencias Ω y γ, tenemos que ∠DAT = ∠ADX −∠AT D =
1 o 1 o 1 o
2 (360 −∠AQX) − 2 ∠AOX = 180 − 2 (∠AW X + ∠AOX) = 90 . ası́ que
AD ⊥ AT , y por lo cual ATk BC. Entonces ATCB es un trapecio isósceles
inscrito en Ω.
Llamemos A0 al punto medio de BC, y consideremos la imagen de ATCB
bajo la homotecia H con centro G y razón − 21 . Entonces h(A)=A0 , h(B)=B0
y h(C)=C0 . Por la simetrı́a sobre B0 C0 tenemos que ∠T CB = ∠CBA =
∠B0 C0 A = ∠DC0 B0 . Usando ATk DA0 concluimos que h(T)=D. Entonces
los puntos D, G y T están alineados y X pertenece a esta recta.

Problema 9(2012)
Sea ABCD un cuadrilátero cı́clico cuyas diagonales cortan en el punto E. La
extensión de los lados AD y BC por A y B corta en el punto F. Sea G el punto
tal que ECGD es un paralelogramo y sea H la imagen de E bajo la reflexión en
AD. Demuestra que D, H, F y G son concı́clicos.

Solución

Figura 30: Problema 9

Vamos a demostrar primero que los triángulos FDG y FBE son semejantes.
Puesto que ABCD es cı́clico, los triángulos EAB y EDC son semejantes, por

35
lo cual FAB y FCD también lo son. Por ser ECGD paralelogramo, GD=EC
y ∠CDG = ∠DCE, también ∠DCE = ∠DCA = ∠DBA por ser ángulos
inscritos.
Entonces

∠F DG = ∠F DC + ∠CDG = ∠F BA + ∠ABD = ∠F BE

GD CE CD FD
= = =
EB EB AB FB
Por lo cual FDG y FBE son semejantes y ∠F GD = ∠F EB.
Puesto que H es la reflexión de E con respecto de FD, concluimos que

∠F HD = ∠F ED = 180o −∠F EB = 180o −∠F GD.

Esto demuestra que D, H, F y G son concı́clicos.

Problema 10(2012)
Dado un triangulo ABC, el punto J es el centro del excı́rculo opuesto al
vértice A. Este excı́rculo es tangente al lado BC en M, y a las rectas AB y AC
en K y L, respectivamente. Las rectas LM y BJ se cortan en F, y las rectas KM
y CJ se cortan en G. Sea S el punto de intersección de las rectas AF y BC, y
sea T el punto de intersección de las rectas AG y BG. Demostrar que M es el
punto medio de ST.

Solución

Figura 31: Problema 10

Sea α = ∠CAB, β = ∠ABC y γ = ∠BCA. La recta AJ es la bisectriz de


∠CAB ası́ que ∠JAK = ∠JAL = α2 . Por ∠AKJ = ∠ALJ = 90o los puntos K
y L están en el circulo ω con diámetro AJ.

36
Como BM y BK son las tangentes al excı́rculo, el triángulo KNM es isósceles.
Puesto que BJ es la bisectriz de ∠KBM , tenemos que ∠M BJ = 90o − β2 y
∠BM K = β2 . Igualmente se deduce que ∠CM L = γ2 . Entonces ∠BM F =
∠CM L, por lo cual

∠LF J = ∠M BJ − ∠BM F = (90o − β2 ) − γ


2 = α
2 = ∠LAJ.

Entonces F esta en el circulo ω. (Por el cálculo del ángulo, F y A están en el


mismo lado de BC.) Análogamente, G también está en ω. Puesto que AJ es el
diámetro de ω, obtenemos que ∠AF J = ∠AGJ = 90o .
Las rectas AB y BC son simétricas con respecto a la bisectriz externa BF.
Como AF ⊥BF y KM ⊥BF , los segmentos SM y AK son simétricos con respecto
de BF, entonces SM=AK. Por simetrı́a TM=AL. Puesto que AK y Al son
iguales como tangentes del excı́rculo, SM=TM, y por lo cual la demostración
esta completa.

37
Referencias
[1] Sánchez-Rubio Garcia, Cristóbal, Ripollés Amela, Manuel: Manual de ma-
temáticas para la preparación olı́mpica. Universitat Jaume I, 2000.
[2] Sessions de preparació per a l’olimpı́ada matemática. Societat catalana de
matemátiques, 2000.
[3] Garcı́a Capitán, Francisco J.Inversión en olimpiadas. Revista escolar de la
olimpı́ada iberoamericana de matemática, 2005.

38
.

PROBLEMAS DE OLIMPIADAS MATEMÁTICAS SOBRE


GEOMETRÍA
El triángulo

ELISABETH GONZÁLEZ FUENTES

Máster de Matemáticas

Universidad de Granada. 2014


Problemas sobre triángulos

Trabajo Fin de Máster presentado en el


Máster Interuniversitario de Matemáticas

Realizado por:
ELISABETH GONZÁLEZ FUENTES

Dirigido por el
Prof. Dr. D. Pascual Jara Martínez

Máster de Matemáticas
Universidad de Granada. 2014
Introducción

Las olimpiadas matemáticas son un concurso que se celebra anualmente desde el año 1965, consistente en
resolver diversos problemas de alta dificultad pero en los que principalmente se utilizan técnicas de nivel de
bachillerato.
En cada sesión se proponen tres o cuatro problemas. Se conceden medallas de oro, plata y bronce. Cada una
de las medallas tiene también un premio en metálico y, además, estos nuevos estudiantes participan en las
olimpiadas españolas, de las que se seleccionan los que van a la olimpíada internacional.
Existen clases de preparación, que no son solo útiles como preparación para las olimpiadas, sino que son una
introducción a técnicas sencillas que los estudiantes podrán utilizar en sus clases, tanto en el Bachillerato como
en la Universidad. Estas lecciones son a su vez una oportunidad para que los estudiantes disfruten empezando
a manipular conceptos matemáticos nuevos.
En este Trabajo de Fin de Máster (TFM) se realizará una síntesis de aquellos aspectos que he considerado
especialmente significativos e importantes, que se pueden desarrollar a lo largo de los diferentes bloques de
Matemáticas cursados hasta segundo de bachillerato, centrándonos especialmente en aquellos relacionados
con el triángulo en el plano.
Tras un desarrollo teórico, necesario para resolver los problemas que se van a trabajar, que ocupa los dos
primeros capítulos de este trabajo, se relaciona una colección de problemas que han aparecido en las diversas
competiciones de las olimpiadas en Matemáticas, locales, nacionales e internacionales, y que tienen como
eje central el triángulo y sus propiedades. La dificultad de estos problemas es distinta en cada una de estas
competiciones, por lo que los clasificamos atendiendo a la misma o equivalentemente según las competiciones
en las que se han propuesto.
Hemos procurado que los ejercicios están complemente desglosados, explicando y analizando en detalle cada
uno de los pasos de su resolución, probando cada uno de los resultados parciales, lo que ha permitido engrosar
la parte teórica con estos resultados con objeto de hacer más accesible la lectura del texto.
Índice general

Introducción 5

I Puntos notables de un triángulo. Conceptos y resultados básicos 1


1 Triángulo . . . . . . . . . . . . . . . . . . . . . . . . . . . . . . . . . . . . . . . . . . . . . . . . . . . 1
2 Circunferencias en el triángulo . . . . . . . . . . . . . . . . . . . . . . . . . . . . . . . . . . . . . . 3
3 Fórmulas del triángulo . . . . . . . . . . . . . . . . . . . . . . . . . . . . . . . . . . . . . . . . . . . 6

II Relaciones métricas en el triángulo 9


4 Teoremas destacables para el triángulo . . . . . . . . . . . . . . . . . . . . . . . . . . . . . . . . . 9
5 Igualdad y semejanza de triángulos . . . . . . . . . . . . . . . . . . . . . . . . . . . . . . . . . . . 18
6 Potencia de un punto respecto de una circunferencia . . . . . . . . . . . . . . . . . . . . . . . . 20

III Olimpiadas Locales 23


7 Problemas . . . . . . . . . . . . . . . . . . . . . . . . . . . . . . . . . . . . . . . . . . . . . . . . . . . 23

IV Olimpiadas Nacionales 39
8 Problemas . . . . . . . . . . . . . . . . . . . . . . . . . . . . . . . . . . . . . . . . . . . . . . . . . . . 39

V Olimpiadas Internacionales 63
9 Problemas . . . . . . . . . . . . . . . . . . . . . . . . . . . . . . . . . . . . . . . . . . . . . . . . . . . 63

Bibliografía 81

Bibliografía. Referencias Web 83

Índice alfabético 85
.
Capítulo I

Puntos notables de un triángulo. Conceptos


y resultados básicos

1. Triángulo

Un triángulo, en geometría, es la unión de tres


segmentos que determinan tres puntos, no coli- A

neales, del plano. Cada punto dado pertenece a


dos segmentos exactamente. Los puntos comunes
a cada par de segmentos se denominan vértices b
del triángulo y los segmentos de recta deter- c
minados son los lados del triángulo. Dos lados
contiguos forman uno de los ángulos interiores
C
del triángulo.

a
Un triángulo es una figura estrictamente convexa
B
y tiene 3 ángulos interiores, 3 ángulos exteriores,
3 lados y 3 vértices entre otros elementos.

1.1. Mediatriz.Circuncentro

La mediatriz de un segmento es la recta perpen-


c
dicular al mismo en su punto medio. b
El circuncentro O de un triángulo es el punto de
intersección de las tres mediatrices de un triángu- O
C
lo.
B a
2 CAP. I. PUNTOS NOTABLES DE UN TRIÁNGULO. CONCEPTOS Y RESULTADOS BÁSICOS

1.2. Altura.Ortocentro
A

Una altura de un triángulo es el segmento per-


ha
pendicular comprendido entre un vértice y el lado
c b
opuesto. H
El ortocentro H de un triángulo es el punto de hb
hc
intersección de las tres alturas de un triángulo.
C
B a

1.3. Bisectriz.Incentro
A

La bisectriz es la semirrecta que divide a un ángu-


lo en dos partes iguales. c b
I
El incentro I de un triángulo es el punto de inter-
sección de las tres bisectrices de un triángulo.
C
B a

n q

Eb

A
La bisectriz de los ángulos exterioresa de un trián-
Ec
gulo se le llama bisectriz exterior.
Un exincentro E de un triángulo es el punto de
intersección de las bisectrices de cualesquiera dos
k
de los tres ángulos exteriores de un triángulo.
C
También se les llama excentros. B

Todo triángulo posee tres exincentros que son los


centros de las circunferencias exinscritas del trián-
gulo.
a
El ángulo exterior de un polígono está formado por un
lado cualquiera y la prolongación del que está a continuación.

Ea

PROPIEDAD

Como los dos ángulos externos son opuestos por el vértice, sus bisectrices son prolongación una de otra
y perpendiculares a la bisectriz interior del mismo vértice.

22 de septiembre de 2014 Curso 20132014.


SEC. 2. CIRCUNFERENCIAS EN EL TRIÁNGULO 3

1.4. Mediana. Baricentro

Una mediana es cada una de las rectas que une el


punto medio de un lado con el vértice opuesto. c
ma b
El baricentro G de un triángulo es el punto de in- G
tersección de las tres medianas de un triángulo. mb mc
C
B a

PROPIEDADES

Cada mediana divide al triángulo en dos regiones de igual área.

El baricentro divide a cada mediana en dos segmentos. El segmento que une el baricentro con el vértice
mide el doble que el segmento que une baricentro con el punto medio del lado opuesto.

2. Circunferencias en el triángulo

2.1. Circunferencia inscrita en el triángulo


A

Una circunferencia inscrita en un triángulo es


aquella que, siendo interior, es tangente a todos c E b
sus lados.
D I r
Al radio de una circunferencia inscrita en un polí-
gono se le denomina inradio. C
El incentro(1.3) es el centro de la circunferencia F
inscrita a dicho triángulo. a
B

PROPIEDAD

Sea:

p el semiperímetro(I.12) del triángulo ABC.

D, E y F los puntos de tangencia de la circunferencia inscrita en el triángulo con el lado c, b y a respec-


tivamente.

Entonces:
AD = p − a, (I.1)

TFM: OLIMPIADAS MATEMÁTICAS SOBRE EL TRIÁNGULO Elisabeth González


4 CAP. I. PUNTOS NOTABLES DE UN TRIÁNGULO. CONCEPTOS Y RESULTADOS BÁSICOS

BD = p − b. (I.2)
Análogamente:
AE = p − a, C E = p − c, (I.3)

C F = p − c, BF = p − b. (I.4)

DEMOSTRACIÓN. Sabemos que:

AD = AE,

EC = C F ,

F B = BD.

Entonces el semiperímetro del triángulo quedaría:

a+b+c
p= (ver semiperímetro(I.12) )
2
2AD + 2EC + 2F B
= (aplicando las igualdades anteriores)
2

= AD + EC + F B.

Despejando tenemos:
AD = p − (EC + F B) = p − a (I.5)

DB = F B = p − (AD + EC) = p − b. (I.6)


Análogamente se probaría:

AE = p − a, C E = p − c, C F = p − c, BF = p − b.
ƒ

2.2. Circunferencia circunscrita en el triángulo

La circunferencia circunscrita de un triángulo es


c b
la circunferencia que pasa por todos sus vértices y,
por tanto, lo contiene completamente en su inte- O
rior.
C
El centro de la circunferencia circunscrita es el cir- a
B
cuncentro(1.1) y su radio se llama circunradio.

22 de septiembre de 2014 Curso 20132014.


SEC. 2. CIRCUNFERENCIAS EN EL TRIÁNGULO 5

2.3. Circunferencias exinscritas en el triángulo


Las circunferencias exinscritas de un triángulo son las circunferencias tangentes a un lado y a las prolonga-
ciones de los otros dos.

A Eb
Ec

E3 E2

b1
E1
B C

Ea

PROPIEDAD
Sea:
p el semiperímetro(I.12) del triángulo ABC.

E1, E2 y E3 los puntos de tangencia de la circunferencia exinscrita en el triángulo con el lado c, b y a,


respectivamente.
Entonces:
BE1 = p − a, (I.7)

BE1 = p − a. (I.8)
Análogamente:
AE2 = p − c, C E2 = p − a, (I.9)

CA3 = p − b, BA3 = p − c. (I.10)


DEMOSTRACIÓN. Análoga a la demostración anterior (2.1). ƒ

TFM: OLIMPIADAS MATEMÁTICAS SOBRE EL TRIÁNGULO Elisabeth González


6 CAP. I. PUNTOS NOTABLES DE UN TRIÁNGULO. CONCEPTOS Y RESULTADOS BÁSICOS

2.4. Arco capaz

El arco capaz es el lugar geométrico de los puntos des-


de los que un segmento AB se ve con el mismo ángulo;
es decir, el lugar geométrico de los vértices de los ángu-
λ
los que tienen la misma amplitud y abarcan un mismo
segmento.
El arco capaz de un segmento AB, de ángulo λ, es un par
de arcos de circunferencia simétricos a cada lado del seg-
mento AB que contiene los vértices de ángulo λ y unidos
A
por los puntos A y B. B
El ángulo que subtiende el segmento AB visto desde el
centro del círculo es 2λ.

2.5. Ángulo inscrito en una circunferencia

Un ángulo inscrito en una circunferencia es un ángulo subtendido en un punto de la circunferencia por otros
dos puntos de esta. Un ángulo inscrito está definido por dos cuerdas de una circunferencia que tienen un
extremo común.

PROPIEDAD

C D
Si dos o más ángulos inscritos
comparten el mismo arco, éstos
α = 45◦ β = 45◦
miden lo mismo.
∠AC B = ∠ADB
α = β. A B

3. Fórmulas del triángulo

Perímetro del triángulo

El perímetro, P, es igual a la suma de las longitudes de sus tres lados y se denota con una P mayúscula.

P = a + b + c. (I.11)

Semiperímetro del triángulo

El semiperímetro, p, de un triángulo es igual a la suma de sus lados partido por 2 (Es el perímetro dividido
entre dos).
a+b+c
p= . (I.12)
2

22 de septiembre de 2014 Curso 20132014.


SEC. 3. FÓRMULAS DEL TRIÁNGULO 7

Área del triángulo

El área, S, de un triángulo es igual a base por altura partido por 2.

bh
S= . (I.13)
2
donde:

b es la base del triángulo.

h es la altura del triángulo.

Área en función del radio de su circunferencia inscrita

A
S = r p. (I.14)
donde:
c b
r es el radio de la circunferencia inscrita(2.1)
en el triángulo. r

p es el semiperímetro(I.12) del triángulo. B a C

Área en función del radio de su circunferencia circunscrita

a bc
S= . (I.15)
4R c b D

donde: R
O
R es el radio de la circunferencia C
circunscrita(2.2) en el triángulo. B a

p es el semiperímetro(I.12) del triángulo.

Área conociendo dos lados de un triángulo y el ángulo que forman


A

c b
1
S= ba sen C.
b (I.16)
2

B a C

TFM: OLIMPIADAS MATEMÁTICAS SOBRE EL TRIÁNGULO Elisabeth González


8 CAP. I. PUNTOS NOTABLES DE UN TRIÁNGULO. CONCEPTOS Y RESULTADOS BÁSICOS

22 de septiembre de 2014 Curso 20132014.


Capítulo II

Relaciones métricas en el triángulo

4. Teoremas destacables para el triángulo


4.1. Teorema de Pitágoras

A
Teorema 4.1 (Teorema de Pitágoras)
En todo triángulo rectángulo el cuadrado de la hi-
potenusa es igual a la suma de los cuadrados de c
b
los catetos.
Es decir, para todo triángulo se tiene:
B
c 2 = b2 + a2 . (II.1) C a

DEMOSTRACIÓN. Este Teorema tiene varias demostraciones interesantes en el siguiente enlace se pueden ver
detalladamente algunas de ellas.

http://es.wikipedia.org/wiki/Teorema_de_Pit%C3%A1goras ƒ

4.2. Teorema de Thales


Existen dos teoremas relacionados con la geometría clásica que reciben el nombre de Teorema de Thales,
ambos atribuidos al matemático griego Thales de Mileto.

A
Teorema 4.2 (Teorema 1)
Dado un triángulo ABC. Si se traza un segmento parale-
lo, B 0 C 0 , a uno de los lados del triángulo, se obtiene otro
triángulo AB 0 C 0 , cuyos lados son proporcionales a los del
B0 C0
triángulo ABC.

AB AC BC
= = . (II.2) C
AB 0 AC 0 B0 C 0 B
10 CAP. II. RELACIONES MÉTRICAS EN EL TRIÁNGULO

r s

Teorema 4.3 (Variante del Teorema de Thales)


Si dos rectas cualesquiera (r y s) se cortan por va-
A A0
rias rectas paralelas (AA0 , BB 0 , C C 0 ) los segmentos
a
determinados en una de las rectas (AB, BC) son
proporcionales a los segmentos correspondientes
en la otra (A0 B 0 , B 0 C 0 ).Es decir, b B B0

AB BC AC
0 0
= 0 0 = 0 0. (II.3)
AB BC AC c C C0

Teorema 4.4 (Teorema 2)


Sea B un punto de la circunferencia de diámetro AC, dis- O
A C
tinto de A y de C. Entonces el ángulo ABC, es recto.

DEMOSTRACIÓN. En la circunferencia de centro O, los


segmentos OA, OB y OC son iguales por ser todos radios
de la misma circunferencia, entonces los triángulos AOB B
y BOC son isósceles por tener dos lados iguales.
Notemos a los ángulos iguales del triángulo AOB con α α β
y a los del triángulo BOC con β, entonces fijándonos en
el triángulo ABC tenemos:
α O β
A C
2α + 2β = π. (II.4)

Dividiendo ambos miembros por dos tenemos:


π
Õ =α+β =
ABC . (II.5)
2
ƒ

Corolario 4.1 (Corolario 1)


En todo triángulo rectángulo la longitud de la mediana(1.4) correspondiente a la hipotenusa es siempre la
mitad de la hipotenusa.

22 de septiembre de 2014 Curso 20132014.


SEC. 4. TEOREMAS DESTACABLES PARA EL TRIÁNGULO 11

Corolario 4.2 (Corolario 2)


La circunferencia circunscrita(2.2) a todo triángulo rectángulo siempre tiene radio igual a la mitad de la hipo-
tenusa y su circuncentro(1.1) se ubicará en el punto medio de la misma.

4.3. Teoremas trigonométricos


Teorema 4.5 (Teorema del seno)
Si en un triángulo ABC, las medidas de los lados opuestos a los ángulos A
b, B
b y Cb son respectivamente a, b, c,
entonces:
a b c
= = . (II.6)
sen A sen B
b b sen Cb
DEMOSTRACIÓN.
A
Dado el triángulo ABC, denotamos por O su
circuncentro(1.1) y dibujamos su circunferencia
α
circunscrita(2.2) .
b
Prolongando el segmento BO hasta cortar la P
c
circunferencia, se obtiene un diámetro BP.
α O

C
Ahora, el triángulo P C B es rectángulo, puesto que
BP es un diámetro, y además los ángulos A by P
b son a
iguales, porque ambos son ángulos inscritos(2.5) B
que abren el segmento BC (véase definición de ar-
co capaz (2.4)).
Por definición de la función trigonométrica seno, se tiene:

BC a
b = sen P
sen A b= = . (II.7)
BP 2R
donde R es el radio de la circunferencia circunscrita.
Despejando 2R tenemos:
a
= 2R. (II.8)
sen A
b
Repitiendo el procedimiento con un diámetro que pase por A y otro que pase por C, se llega a que las tres
fracciones tienen el mismo valor 2R y por tanto:

a b c
= = . (II.9)
sen A
b sen B
b sen Cb
La conclusión que se obtiene suele llamarse teorema de los senos generalizado y establece:

Si en un triángulo ABC, las medidas de los lados opuestos a los ángulos A b y Cb son respectivamente
b, B
(2.2)
a, b, c y R es el radio de la circunferencia circunscrita , entonces:

a b c
= = = 2R. (II.10)
sen A
b sen B
b sen Cb
ƒ

TFM: OLIMPIADAS MATEMÁTICAS SOBRE EL TRIÁNGULO Elisabeth González


12 CAP. II. RELACIONES MÉTRICAS EN EL TRIÁNGULO

Teorema 4.6 (Teorema del coseno)


Si en un triángulo ABC, las medidas de los lados opuestos a los ángulos A
b, B
b y Cb son respectivamente a, b, c,
entonces:

a2 = b2 + c 2 − 2bc cos α. (II.11)

DEMOSTRACIÓN.
Vamos a hacer la demostración a través del Teorema de Pitágoras.
Notemos que el Teorema de Coseno es equivalente al Teorema de Pitágoras(4.1) cuando el ángulo Cb es recto.

Por tanto sólo es necesario considerar los casos cuando c es adyacente a dos ángulos agudos y cuando c
es adyacente a un ángulo agudo y un obtuso.

1. c es adyacente a dos ángulos agudos

Consideremos la figura adjunta.


Por el teorema de Pitágoras, la longitud c es calcu-
lada así: B
c 2 = h2 + u2 . (II.12)
Pero, la longitud h también se calcula así: c a
h

h2 = a2 − (b − u)2 . (II.13)
u
A C
Sumando ambas ecuaciones y luego simplificando b
obtenemos:

c 2 = a2 − b2 + 2bu. (II.14)

Por la definición de coseno, se tiene:

b−u
cos Cb = , (II.15)
a

y por lo tanto:

u = b − a cos C.
b (II.16)

Sustituimos el valor de u en la ecuación para c 2 , concluyendo que:

c 2 = a2 + b2 − 2a b cos C,
b (II.17)

y tenemos terminado el primer caso.

22 de septiembre de 2014 Curso 20132014.


SEC. 4. TEOREMAS DESTACABLES PARA EL TRIÁNGULO 13

2. c es adyacente a un ángulo obtuso

Consideremos la figura adjunta.


El teorema de Pitágoras establece nuevamente:
B
c 2 = h2 + u2 , (II.18)

pero en este caso a


h c

h2 = a2 − (b + u)2 . (II.19)
u
C
Combinando ambas ecuaciones obtenemos: A b

c 2 = u2 + a2 − b2 − 2bu − u2 , (II.20)

y de este modo:

c 2 = a2 − b2 − 2bu. (II.21)

De la definición de coseno, se tiene:


b+u
cos Cb = , (II.22)
a
y por tanto:
u = a cos Cb − b. (II.23)

Sustituimos en la expresión para c 2 y simplificamos:


c 2 = a2 − b2 − 2b(a cos Cb − b), (II.24)
concluyendo nuevamente:
c 2 = a2 + b2 − 2a b cos C.
b (II.25)
Esto concluye la demostración.
Es importante notar, que si se considera a u como un segmento dirigido, entonces sólo hay un caso y las
dos demostraciones se convierten en la misma.
ƒ

4.4. Teorema de la bisectriz


A

α α
Teorema 4.7 (Teorema de la bisectriz)
Dado el triángulo ABC, sea AD la bisectriz del án- b

gulo interno Ab, entonces se cumple la siguiente c

proporción:
BA BD
= . (II.26) C
AC DC
D
a
B

TFM: OLIMPIADAS MATEMÁTICAS SOBRE EL TRIÁNGULO Elisabeth González


14 CAP. II. RELACIONES MÉTRICAS EN EL TRIÁNGULO

DEMOSTRACIÓN.
C

Fijándonos en la figura de la derecha tenemos:


x x
a
|AC| ≡ b,
b
|BC| ≡ a,
|AD| ≡ m,
|BD| ≡ n, π− y B
y
∠AC D ≡ ∠DC B ≡ x,
n
∠ADC ≡ y. D
m
A

Aplicando el teorema del seno(4.5) al triángulo ADC tenemos:

m b
= . (II.27)
sen x sen y

Los ángulos y y π − y son suplementarios1 , lo que implica:

sen (π − y) = sen ( y), (II.28)

entonces aplicando ahora el teorema del seno al triángulo DBC tenemos:


n a
= . (II.29)
sen x sen y

Dividiendo la ecuación (II.28) por la ecuación (II.29) y simplificando obtenemos:

m b
= . (II.30)
n a
ƒ

4.5. Teorema de Stewart


A

Teorema 4.8 (Teorema de Stewart)


Dado el triángulo ABC, sean a, b, c las longitudes b
de los lados BC, AC y AB, respectivamente. c
d
Sea D un punto dentro del segmento BC. Si BD =
m, C D = n y AD = d, se cumple que:
C
2 2 2
d a = b m + c n − mna. (II.31) D
n
a
m
B

1
Los ángulos suplementarios son aquellos cuyas medidas suman 1800 .

22 de septiembre de 2014 Curso 20132014.


SEC. 4. TEOREMAS DESTACABLES PARA EL TRIÁNGULO 15

DEMOSTRACIÓN.
B DA = α, entonces ADC
Sea Õ Õ = 180o − α.

Utilicemos el Teorema del coseno(4.6) en los triángulos A


ABD y ADC.

d 2 + m2 − c 2
ABD: c 2 = d 2 + m2 − 2d m cos α → cos α = . b
2d m
c
d
ADC: b2 = d 2 + n2 − 2d n cos (180o − α) →

d 2 + n2 − b2 180o -α C
− cos α = a cos (180o − α) = α
D
2d n n
a
−d − n + b
2 2 2 m
→ cos α = . B
2d n
a
Ya que cos (180o − α) = − cos α

Igualando las dos expresiones de cos α tenemos:

d 2 + m2 − c 2 −d 2 − n2 + b2
=
2d m 2d n

(d 2 + m2 − c 2 )n = (−d 2 − n2 + b2 )m

d 2 n + m2 n − c 2 n = −d 2 m − n2 m + b2 m

d 2 n + d 2 m = b2 m + c 2 n − (n2 m + m2 n)

d 2 (n + m) = b2 m + c 2 n − nm(n + m)

d 2 a = b2 m + c 2 n − nma, (ya que m + n = BD + C D = BC = a).

4.6. Teorema de Apolonio

Teorema 4.9 (Teorema de Apolonio (Teorema de la mediana))


Para todo triángulo la suma de los cuadrados de dos lados cualesquiera, es igual a la mitad del cuadrado del
tercer lado más el doble del cuadrado de su mediana correspondiente.

DEMOSTRACIÓN.

TFM: OLIMPIADAS MATEMÁTICAS SOBRE EL TRIÁNGULO Elisabeth González


16 CAP. II. RELACIONES MÉTRICAS EN EL TRIÁNGULO

Vamos a hacer la demostración a través del Teore- A

ma del Coseno(4.6) .
m
Sea ABC un triángulo cualquiera de lados a, b y c,
c α b
para cuyo lado c se ha trazado la mediana corres-
n
pondiente Mc , entonces: α0

1 B Mc
C
m=n= c. (II.32)
2 a

La mediana Mc forma con el lado c los ángulos α y α0 como podemos ver en el dibujo. Entonces según el
Teorema del Coseno tenemos:
b2 = m2 + Mc2 − 2mMc cosα, (II.33)

a2 = n2 + Mc2 − 2nMc cosα0 . (II.34)


Teniendo en cuenta (II.32) y aplicando el coseno de los ángulos del segundo cuadrante en función de los del
primero2 en las dos últimas igualdades tenemos:

c2 c
b2 = + Mc2 − 2 Mc cosα, (II.35)
4 2

c2 c
a2 = + Mc2 + 2 Mc cosα. (II.36)
4 2
Sumando ambas ecuaciones tenemos:
c2
a2 + b2 =+ 2Mc2 . (II.37)
2
Esta expresión es la conclusión del Teorema de Apolonio realizada para la mediana Mc , como se trata de
una demostración general, con razonamientos similares se puede obtener expresiones equivalente para las
restantes medianas Ma y M b las cuales serían:

a2
b2 + c 2 = + 2Ma2 , (II.38)
2

b2
a2 + c 2 = + 2M b2 . (II.39)
2
ƒ

4.7. Teorema de Euler

El Teorema de Euler se usa para el cálculo de la distancia entre el incentro y el circuncentro.


Teorema 4.10
Sea d = IO. Entonces:

d 2 = R2 − 2Rr.
2
cos (180o − α) = − cos α.

22 de septiembre de 2014 Curso 20132014.


SEC. 4. TEOREMAS DESTACABLES PARA EL TRIÁNGULO 17

donde:

R es el radio de la circunferencia circunscrita(2.2) .

r es el: radio de la circunferencia inscrita(2.1) .

DEMOSTRACIÓN.
Queremos hallar d, para ello, sean:

N el segundo corte de la bisectriz que parte de A con la circunferencia circunscrita.

M el punto diametralmente opuesto.

Si llamamos:

2α = A
b,

2β = B
b.

Tenemos:

α=N
Õ AC = B
Õ AN ,

β = Id
BA = Id
BA.

Como ÕN BI = Õ
B I N = α + β, el triángulo I BN es isósceles y por tanto N B = N I. Por otro lado tenemos que la
potencia del incentro(1.3) I respecto de la circunferencia circunscrita(2.2) vale:
(6)

d 2 − R2 = −I N IA = −N B IA. (II.40)

Como:

N B = M N sen α,
IP
IA = .
sen α
queda:

IP
R2 − d 2 = M N sen α = M N I P = 2Rr.
sen α
Entonces nos queda:
d 2 = R2 − 2Rr. (II.41)
2
Una consecuencia importante se deriva del resultado anterior y de ser d ≥ 0. En efecto:

d 2 = R2 − 2Rr ≥ 0. (II.42)

Por lo que nos queda lo que conocemos como la desigualdad de Euler:

R ≥ 2r. (II.43)

TFM: OLIMPIADAS MATEMÁTICAS SOBRE EL TRIÁNGULO Elisabeth González


18 CAP. II. RELACIONES MÉTRICAS EN EL TRIÁNGULO

4.8. Desigualdad triángular

La suma de las medidas de dos lados de un triángulo es mayor o igual que la medida del tercero. La igualdad
se cumple si y solo si A, B y C están alineados en ese orden.

DEMOSTRACIÓN. Véase la demostración en el siguiente video:

https://www.youtube.com/watch?v=mKXU01rHWOs ƒ

4.9. Fórmula del Herón

Æ
S= p(p − a)(p − b)(p − c). (II.44)

donde:

p es el semiperímetro(I.12) del triángulo.

5. Igualdad y semejanza de triángulos

Criterios de igualdad de triángulos

Dos triángulos son iguales cuando tienen iguales un lado y sus dos ángulos adyacentes.
C0
0
b
A

A0
c α b
α’
a0
β 0
c
β’
B a C

B0

α = α0 , β = β 0 y c = c 0 .

Dos triángulos son iguales cuando tienen dos lados iguales y el ángulo comprendido.
C0
A b0

A0
c b

a0
β
0
c
β’
B a C

B0
0 0 0
β = β ,c = c y a = a .

22 de septiembre de 2014 Curso 20132014.


SEC. 5. IGUALDAD Y SEMEJANZA DE TRIÁNGULOS 19

Dos triángulos son iguales cuando tienen los tres lados iguales.

C0
0
A b

b A0
c

a0
c0
B a C

B0

b = b0 ,c = c 0 y a = a0 .

5.1. Semejanza de triángulos

Dos triángulos son semejantes si tienen la misma forma, aunque no necesariamente el mismo tamaño.
Cuando dos triángulos son semejantes, los ángulos correspondientes son congruentes y los lados correspon-
dientes son proporcionales en medida.

Razón de semejanza

Llamamos razón de semejanza entre dos triángulos ABC y A0 B 0 C 0 a la constante de proporcionalidad k entre
sus lados:
AB BC CA
= 0 0 = 0 0 = k. (II.45)
A0 B 0 BC CA
Si tomamos las proporciones entre los lados al revés, la razón de proporcionalidad será 1/k.

Criterios de semejanza de triángulos

Dos triángulos son semejantes si tienen dos ángulos iguales.


A

A0
c α b
c0 b0
0
α0
β
β
B0 a0 C0
B a C
0 0
α=α yβ =β .

Dos triángulos son semejantes si tienen los lados proporcionales.

TFM: OLIMPIADAS MATEMÁTICAS SOBRE EL TRIÁNGULO Elisabeth González


20 CAP. II. RELACIONES MÉTRICAS EN EL TRIÁNGULO

A
A0
c b b0
c0

B0 a0 C0
B a C

a b c
= 0 = 0.
a0 b c

Dos triángulos son semejantes si tienen dos lados proporcionales y el ángulo comprendido entre ellos
igual.

A
A0
b
c α c0 b0
α0

B0 a0 C0
B a C

b c
0
= 0 y α = α0 .
b c

Posición de Thales

Dos triángulos se dicen en posición de Thales


(Í ABC = T Í ABC) si:

Dos lados de uno contienen respectivamente


a dos lados del otro.

El tercer lado de uno es paralelo al tercer


lado del otro.

Dos triángulos en posición de Thales son semejantes

6. Potencia de un punto respecto de una circunferencia

Dado un punto cualquiera P y una circunferencia c, se traza una recta que pasa por P y corta a la circunferencia
en dos puntos M y N .
Se verifica que el producto de las distancias P M y P N toma siempre el mismo valor, sea cual sea la posición
de la recta.
Por tanto, tiene sentido definir la potencia de un punto respecto de una circunferencia como el resultado
de este producto:
k = |P M ||P N |. (II.46)

22 de septiembre de 2014 Curso 20132014.


SEC. 6. POTENCIA DE UN PUNTO RESPECTO DE UNA CIRCUNFERENCIA 21

Si la distancia entre el punto y el centro de la circunferencia es d, y el radio r, la potencia es:

k = |d 2 − r 2 | = (d + r)(d − r). (II.47)

Esta expresión nos permite observar fácilmente que:

si k < 0, entonces P es interior a la circunferencia.

si k = 0, entonces P es está en la circunferencia.


b
si k > 0, entonces P es exterior a la circunferencia.

P
Un caso de especial consideración es el formado
por una recta tangente y una secante, como en la T
figura. En esta situación el ángulo B
Õ T P es semi-
a a
inscrito y mide la mitad del arco BT , al igual que
B
el ángulo inscrito(2.5) T
Ô AP.
a
Se llama ángulo semiinscrito en una circunferencia a
cualquier ángulo que tenga su vértice en la circunferencia,
una de las semirectas que determina sus lados sea tangente
a la circunferencia y la otra sea secante A

La igualdad de ángulos nuevamente implica una semejanza de triángulos, entre los triángulos PAT y P T B.
Dicha semejanza implica:
PA PT
= . (II.48)
PT PB
Y por tanto:
P T 2 = PA P B. (II.49)

6.1. Puntos cocíclicos


D

Los puntos cocíclicos (o concíclicos) son aquellos


que pertenecen a una misma circunferencia.

TFM: OLIMPIADAS MATEMÁTICAS SOBRE EL TRIÁNGULO Elisabeth González


22 CAP. II. RELACIONES MÉTRICAS EN EL TRIÁNGULO

PROPIEDADES DE LOS PUNTOS COCÍCLICOS

1. Dos puntos siempre son cocíclicos (caso trivial). Tres puntos siempre serán cocíclicos excepto en el caso
de que estén alineados. En el caso de cuatro puntos D, C, B, A, serán cocíclicos sólo si los ángulos ADC
y C BA son suplementarios3 .
D C

Teorema 6.1 (Ángulo constante)


Sean A, B, C y D cuatro puntos cocíclicos, coloca-
2.
dos en este orden en el círculo. Entonces tenemos A
la igualdad de ángulos: AC
Õ B=A ÕDB.

DEMOSTRACIÓN. Ambos ángulos miden el doble del ángulo al centro AOB. Dicho de otro modo, si se
considera la cuerda AB y un punto móvil que recorre el círculo quedándose del mismo lado con relación
a AB, entonces el ángulo AM B es constante.
Se dice que se ve AB desde M bajo un ángulo constante.
Tomando otra cuerda, se obtiene otra igualdad: por ejemplo, con BC : BAC = BDC. ƒ

3
Los ángulos suplementarios son aquellos cuyas medidas suman 180o .

22 de septiembre de 2014 Curso 20132014.


Capítulo III

Olimpiadas Locales

7. Problemas

Ejercicio. 7.1. (2013, Ver 40 en las Referencias web)


Sean A, B y C los vértices de un triángulo y P, Q y R los respectivos pies de las bisectrices(1.3) trazadas desde
esos mismos vértices. Sabiendo que PQR es un triángulo rectángulo en P se pide probar:

Que ABC ha de ser obtusángulo1 .

Que en el cuadrilátero ARPQ, pese a no ser cíclico, la suma de sus ángulos opuestos es constante.

SOLUCIÓN.
Primero apliquemos el Teorema de la bisectriz(4.7) en el A
triángulo ABC: R
Q
BA PB b
= , c
AC PC
C
c PB B a
= . P
b PC

Expresando P C como a − P B y P B como a − P C tenemos:

c PB
= ,
b a − PB
c a − PC
= .
b PC
24 CAP. III. OLIMPIADAS LOCALES

Despejando P B y P C respectivamente tenemos:

ca
PB = ,
b+c

ab
PC = .
b+c

Análogamente tenemos:

ba
QC = ,
a+c

bc
QA = ,
a+c

cb
RA = ,
a+b

ca
RB = .
a+b

Ahora vamos a aplicar el Teorema de Stewart(4.8) :

AP 2 a = b2 P B + c 2 C P − P B P C a. (III.1)

Sustituyendo en (III.1) los valores calculados anteriormente con el Teorema de la bisectriz tenemos:

ca ab ca a b
b2 + c2 − a
b+c b+c b+c b+c
AP 2 =
a

b3 c + b2 c 2 + c 2 b2 + c 3 b − ca2 b
=
(b + c)2

bc(b2 + 2bc + c 2 − a2 )
=
(b + c)2

bc(b2 + 2bc + c 2 − (b2 + c 2 − 2 b c cos A


b))
= , aplicando el Teorema del Coseno(4.6) (a2 = b2 + c 2 − 2bc cos A)
(b + c)2

2b2 c 2 (1 + cos A
b)
= .
(b + c)2

22 de septiembre de 2014 Curso 20132014.


SEC. 7. PROBLEMAS 25

Aplicamos raíces en ambos lados y tenemos:

p p
2bc 1 + cos A
b
AP =
b+c
v
p p t 1 + cos A
b
2bc 2
2
=
b+c

A
b
2bc cos
= 2, aplicando la fórmula del coseno del ángulo mitad2 .
b+c

Ahora vamos a calcular los lados del triángulo PQR.

AQR:

QR2 = QA2 + PA2 − 2QA RA cos A


b, aplicando el Teorema del Coseno

b2 c 2 c 2 b2 bc c b
= + −2 cos A
b
(a + c)2 (a + b)2 a+c a+b

sustituyendo los valores del principio calculados con el Teorema de la bisectriz.

ARP:

PR2 = AP 2 + RA2 − 2PA RA cos (A


b/2), aplicando el Teorema del Coseno

4b2 c 2 c 2 b2 2b2 c 2
= cos (
2 b
A/2) + − 2 cos2 (A
b/2)
(b + c)2 (a + b)2 (b + c)(a + b)

sustituyendo los valores del principio calculados con el Teorema de la bisectriz

4b2 c 2 (a − c) c 2 b2
= cos (
2 b
A/2) + .
(b + c)2 (a + b) (a + b)2

1 + cos A
s
2
La fórmula para el coseno del ángulo mitad es: cos A/2 =
2

TFM: OLIMPIADAS MATEMÁTICAS SOBRE EL TRIÁNGULO Elisabeth González


26 CAP. III. OLIMPIADAS LOCALES

AQP:
QP 2 = AP 2 + QA2 − 2PA QA cos A/2, aplicando el Teorema del Coseno

4b2 c 2 c 2 b2 2b2 c 2
= cos (
2 b
A/2) + − 2 cos2 (A
b/2)
(b + c)2 (a + c)2 (b + c)(a + c)

sustituyendo los valores del principio calculados con el Teorema de la bisectriz

4b2 c 2 (a − b) c 2 b2
= cos (
2 b
A/2) + .
(b + c)2 (a + c) (a + c)2

Como sabemos que el triángulo PQR es rectángulo en P tenemos:


QR2 = PQ2 + PR2 . (III.2)

sustituyendo el valor de cada lado tenemos:


4b2 c 2 (a − b) 4b2 c 2 (a − c) bc c b
cos (
2 b
A/2) + cos2 (A
b/2) = −2 cos A
b
(b + c) (a + c)
2 (b + c) (a + b)
2 a+c a+b

2b2 c 2 (1 + cos A
b)(a − b)(a + b) 2b2 c 2 (1 + cos A)(a − c)(a + c) b2 c 2 (b + c)2
+ = −2 cos A
b
(b + c)2 (a + c)(a + b) (b + c)2 (a + b)(a + c) (a + c)(a + b)(b + c)2

(1 + cos A
b)(a2 − b2 + a2 − c 2 ) = −(b + c)2 cos A
b

2a2 − b2 − c 2 + (2a2 − b2 − c 2 + b2 + 2bc + c 2 ) cos A


b= 0

2a2 − b2 − c 2 + (2a2 + 2bc) cos A


b= 0

Despejando el coseno de esta expresión tenemos:


−2a2 + b2 + c 2
b=
cos A
2a2 + 2bc

−a2 − a2 + b2 + c 2
b=
cos A
2a2 + 2bc

−a2 − b2 − c 2 + 2 b c cos A
b + b2 + c 2
b=
cos A , aplicando el Teorema del Coseno
2a2 + 2bc

−a2 + 2 b c cos A
b
b=
cos A
2a + 2bc
2

b (2a2 + 2bc) = −a2 + 2 b c cos A


cos A b

b = −a2
2a2 cos A

1
b= − .
cos A
2

22 de septiembre de 2014 Curso 20132014.


SEC. 7. PROBLEMAS 27

b = 120o y con esto demostramos que el triángulo ABC es obtusángulo.


Entonces A

b = 90o por ser el triángulo PQR rectángulo en P. Con lo que:


Por otro lado sabemos que P

b+A
P b = 210o ,

b+Q
R b = 360o − ( P
b+A
b) = 360o − 210o = 150o ,

y con esto queda demostrado que en el cuadrilátero ARPQ la suma de sus ángulos opuestos es constante. ƒ

Ejercicio. 7.2. (2011, Ver 40 en las Referencias web)


Sean:

b = 45o .
ABC un triángulo acútangulo3 con A

P el pie de la altura(1.2) por B.

Trazamos la circunferencia de centro P que pasa por C y que vuelve a cortar a AC en el punto X y a la altura
P B en el punto Y .
Sean r y s las rectas perpendiculares a la recta AY por P y X respectivamente y L y K las intersecciones de r
y s con AB.
Demuestra que L es el punto medio de K B.

SOLUCIÓN.
Por construcción tenemos:

P X = PY = P C, (III.3)

por ser todos radio de la circunferencia.

Los triángulos PAY y P C B (son rectángulos en P por ser


A
P B la altura del triángulo ABC) son iguales aplicando el
segundo criterio de igualdad de triángulos(5) :
K
s X
c
1. A
Õ PY = BP
Õ C. b
r L
Q P
2. AP = P B por ser el triángulo AP B isósceles (tiene dos Y
ángulos iguales: B
Ô PA = 90o , P
Ô AB = 45o y por tanto B C
a
BA = 180 − 45 − 90 = 45 )
PÔ o o o o

3. PY = P C por (III.3).

Entonces todos sus ángulos y lados son iguales y en particular:

P
Ô AY = P
Õ BC. (III.4)

TFM: OLIMPIADAS MATEMÁTICAS SOBRE EL TRIÁNGULO Elisabeth González


28 CAP. III. OLIMPIADAS LOCALES

Por otro lado el triángulo PY Q (rectángulo en Q por ser r perpendicular a AY ) es semejante a APY por el
primer criterio de semejanza(5.1) .

QP = YÔ
1. YÕ PA por ser ambos ángulos rectos.

Y A = PY
2. PÔ Õ Q por ser el mismo ángulo.

Entonces todos sus ángulos son iguales y en particular:

PQ = P
YÕ Ô AY . (III.5)

Entonces:
L P B = YÕ
Õ PQ = P
Ô AY . (III.6)

Por otro lado el segmento P L es paralelo a BC:

Si nos fijamos en los triángulos QPA y BC P respectivamente tenemos:

Ô = 180o − PAQ
QPA Õ = 180o − P
Ô − AQP Ô AY − 90o = 180o − P
Õ BC − 90o = 180o − P
Õ BC − C
Õ P B = BC
Õ P,

teniendo en cuenta (III.3) y que los ángulos AQP


Õ y BC
Õ P son rectos.

Entonces los ángulos ÔL PA = QPA


Ô y BC Õ P son iguales, por lo que si desplazamos el segmento BC ha-
cia donde está el segmento P L el ángulo que forma con el segmento AC no varía.

Y a la misma vez los segmentos P L (recta r) y BC son paralelos a K X (recta s) por construcción, así que
aplicando el Teorema de Thales(4.3) , tenemos:
KL LB KB
= = , (III.7)
XP PC XC

nos quedamos con la primera igualdad y tenemos:


KL LB
=
XP PC
PC LB
=
XP KL
PX LB
= porque P C = P X (véase (III.3))
XP KL
LB
=1
KL

LB = K L.

22 de septiembre de 2014 Curso 20132014.


SEC. 7. PROBLEMAS 29

Ejercicio. 7.3. (2011, Ver 40 en las Referencias web)


En un triángulo llamaremos:

O al circuncentro(1.1) .

I al incentro(1.3) .

r al radio de la circunferencia inscrita(2.1) .

Si la mediatriz(1.1) del segmento OI corta a la circunferencia circunscrita(2.2) en L y L I vuelve a cortar en M ,


demuestra que:
I M = 2r. (III.8)
q s

SOLUCIÓN.
L
A

Sea R el radio de la circunferencia circunscrita, por lo que apli-


cando el Teorema de Euler(4.7) tenemos: c b R
T
r I
Q
OI 2 = R2 − 2Rr. (III.9) O

Sean T y Q los puntos de corte de la recta OI con la circunfe- B C


a
rencia circunscrita. t
M

Considerando las cuerdas L M y TQ y aplicando la potencia(6) del punto I respecto a la circunferencia circuns-
crita, con respecto a ambas cuerdas, tenemos:

I L I M = I T IQ. (III.10)

Sabemos que:

I L = O L, por ser L un punto de la mediatriz de IO.

O L = R, por ser O el centro de la circunferencia circunscrita y L estar en está.

Entonces tenemos:
I L = O L = R. (III.11)

TFM: OLIMPIADAS MATEMÁTICAS SOBRE EL TRIÁNGULO Elisabeth González


30 CAP. III. OLIMPIADAS LOCALES

Por otro lado tenemos:

IT = OI + OT = OI + R,

IQ = OQ - OI = R - OI,

por ser OT y OQ radios de la circunferencia circunscrita.

Sustituyendo en (III.10) estas dos igualdades tenemos:

I L I M = I T IQ

R I M = (OI + R)(R − OI) por (III.11)

R I M = R2 − OI 2

R I M = R2 − (R2 − 2Rr) por (III.9)

R I M = 2Rr.

Despejando I M nos queda:


2Rr
IM = = 2r. (III.12)
R
ƒ

Ejercicio. 7.4. (2010, Ver 40 en las Referencias web)


Determina los lados del triángulo rectángulo del que se conocen el perímetro(3) P = 96 y la altura(1.2) sobre
96
la hipotenusa h = .
5

SOLUCIÓN.
Consideramos el triángulo ABC rectángulo en A.

Tenemos que el área del triángulo(I.13) se puede calcular A

como:
ah c b
S= tomando como altura h y base a.
2 h

cb
S= tomando como altura b y base c. C
2 B a

Igualando ambas expresiones tenemos:

ah = c b. (III.13)

22 de septiembre de 2014 Curso 20132014.


SEC. 7. PROBLEMAS 31

Por otro lado como conocemos el perímetro tenemos que:

P = a + b + c = 96 → b + c = P − a. (III.14)

Elevando al cuadrado ambos miembros tenemos:

(b + c)2 = (P − a)2 . (III.15)

Desarrollamos:
b2 + 2bc + c 2 = P 2 − 2Pa + a2

b2 + 2bc + a2 − b2 = P 2 − 2Pa + a2 , aplicando Pitágoras(4.1) en el triángulo rectángulo ABC

2bc = P 2 − 2Pa

2ah = P 2 − 2Pa utilizando (III.13)

2a(h + P) = P 2 .

Despejando a tenemos:
P2
a= . (III.16)
2(h + P)

Como por el enunciado del ejercicio conocemos h y P tenemos:

962 9216
a= ‹= = 40. (III.17)
96 576

2 + 96 2
5 5

Ya tenemos determinado el lado a, para determinar los lados b y c basta con resolver la ecuación:

z 2 − (b + c)z + bc = 0. (III.18)

Las soluciones de esta ecuación serán los lados b y c del triángulo4 .

La ecuación es equivalente a:
z 2 − (P − a)z + ah = 0, (III.19)
usando (III.14) y (III.13).

Como conocemos a y P tenemos:

P2 P2 96
z 2 − (96 − )z + = 0 −→ z 2 − 56z + 768 = 0. (III.20)
2(h + P) 2(h + P) 5

4
La ecuación z 2 − (b + c)z + bc = 0 es cierta porque sabemos que los valores b y c son soluciones de la ecuación (z − b)(z − c) = 0
equivalente a la anterior.

TFM: OLIMPIADAS MATEMÁTICAS SOBRE EL TRIÁNGULO Elisabeth González


32 CAP. III. OLIMPIADAS LOCALES

Resolviendo la ecuación tenemos las dos soluciones:

p p
56 + 562 − 4 768 56 + 64 56 + 8
z= = = = 32, (III.21)
2 2 2
p p
56 − 562 − 4 768 56 − 64 56 − 8
z= = = = 24. (III.22)
2 2 2
Entonces los lados del triángulo son 40, 32 y 24. ƒ

Ejercicio. 7.5. (2008, Ver 40 en las Referencias web)


En el triángulo ABC, el área(3) S y el ángulo Cb son conocidos. Halla el valor de los lados a y b para que el lado
c sea lo más corto posible.

SOLUCIÓN.
Por el Teorema del Coseno(4.6) tenemos:
c 2 = a2 + b2 − 2a b cos Cb

= (a − b)2 + 2a b − 2a b cos Cb

= (a − b)2 + 2a b(1 − cos C).


b

Del área (I.16) del triángulo conociendo dos lados de un triángulo y el ángulo que forman tenemos:
1 2S
S= a b sen Cb → a b = . (III.23)
2 sen Cb
Entonces:
2S
c 2 = (a − b)2 + 2 (1 − cos C),
b teniendo en cuenta el valor de ab (véase (III.23)).
sen Cb
Como (a − b)2 > 0 la expresión anterior será mínima cuando a = b, entonces volviendo a (III.23) tenemos:
v
2S t 2S
a2 = →b=a= . (III.24)
sen Cb sen Cb
ƒ

Ejercicio. 7.6. (2008, Ver 40 en las Referencias web)


Sean D, E y F los puntos de tangencia de la circunferencia inscrita(2.1) al triángulo ABC con los lados BC, AC
y AB respectivamente. Demuestra que:
4S DE F ≤ SABC , (III.25)
donde SX Y Z denota el área(3) del triángulo X Y Z.

22 de septiembre de 2014 Curso 20132014.


SEC. 7. PROBLEMAS 33

SOLUCIÓN.
A
(1.3)
Sea I el incentro del triángulo ABC. Tenemos que:
E
I D ⊥ BC, F
c r b

I E ⊥ AC, I

I F ⊥ AB,
C
B D a
por ser D, E y F los puntos de tangencia de la circunferencia
inscrita.

Por otro lado usando el área(I.16) del triángulo conociendo dos lados y el ángulo que lo forman tenemos:

1 1
SABC = b = b c sen A
AB AC sen A b, (III.26)
2 2

1 1
SE I F = E I F = r 2 sen Ô
E I F I sen Ô EI F, (III.27)
2 2

1 1
SF I D = I D I F sen D
Ô I F = r 2 sen Ô
EI F, (III.28)
2 2

1 1
SE I D = I E I D sen E
Ô I D = r 2 sen E
Ô I D. (III.29)
2 2

Los ángulos A FA = Id
E I F son suplementarios5 (al fijarnos en el cuadrilatero AF I E tenemos que Id
by Ô EA = 90o
b+ Ô
por definición de los puntos F y E, entonces A E I F = 360o − 90o − 90o = 180o ).

Entonces tenemos que:

b = sen Ô
sen A EI F, verificando una de las propiedades de los ángulos suplementarios6 . (III.30)

Entonces:
SE I F r2
= . (III.31)
SABC bc

5
Los ángulos suplementarios son aquellos cuyas medidas suman 1800 .
6
El seno de dos ángulos suplementarios verifica: sen (180o − α) = sen α.

TFM: OLIMPIADAS MATEMÁTICAS SOBRE EL TRIÁNGULO Elisabeth González


34 CAP. III. OLIMPIADAS LOCALES

Análogamente tenemos:
SE I D r2
= , (III.32)
SABC ab

SF I D r2
= . (III.33)
SABC ca

Sumando estas tres fracciones tenemos:

S DE F S S S r 2 (a + c + b)
= EI F + EI D + F I D = . (III.34)
SABC SABC SABC SABC a bc

Por otro lado:

Usando el área(I.14) en función del radio de su circunferencia inscrita y el semiperímetro(I.12) :

a+b+c
 ‹
S = pr = r . (III.35)
2

Usando el área(I.15) en función del radio de su circunferencia circunscrita:


a bc
S= → 4RS = a bc. (III.36)
4R

Sustituyendo estas dos últimas igualdades en (III.34) tenemos:

S DE F 2rS r
= = . (III.37)
SABC 4RS 2R

Ahora aplicamos la desigualdad de Euler(II.43) (R ≥ 2r):

S DE F r r 1
= ≤ = . (III.38)
SABC 2R 2 2r 4

Por lo que nos queda:


4S DE F ≤ SABC . (III.39)
A

Y la igualdad se verifica cuando ABC es equilátero. Área de ABC = 6.2


c
Como podemos ver en el dibujo de la derecha, el E
F
área del triángulo chico es 1.55, que multiplica- I b ƒ
da por 4 da 6.2 (correspondiente con el área del
Área de FDE = 1.55
triángulo grande).
C
B a D

22 de septiembre de 2014 Curso 20132014.


SEC. 7. PROBLEMAS 35

Ejercicio. 7.7. (2007, Ver 40 en las Referencias web)


Demostrar que un triángulo, la distancia de un vértice cualquiera al ortocentro(1.2) es el doble de la distancia
del circuncentro(1.1) al lado opuesto a ese vértice.

SOLUCIÓN.
s

A
Sean:
R
H el ortocentro del triángulo. b
c H
O el circuncentro del triángulo. O

H 0 el simétrico de H respecto del lado BC. B


P Q C
0
O
O0 el simétrico de O respecto del lado BC.
H0

Como los triángulos BPA y BCR son rectángulos por estar AP y CR en dos de las alturas(1.2) del triángulo
ABC, ambos triángulos tienen un ángulo recto. Si a esto le sumamos que ambos triángulos también compar-
ten el ángulo ÕC BA tenemos que ambos triángulos son semejantes aplicando el primer criterio de semejanza
(5.1)
de triángulos .

Esto implica que también su tercer ángulo es igual, es decir:

B
Ô AP = RC
Õ B. (III.40)

Equivalentemente mirando el dibujo podemos ver que la igualdad anterior es equivalente a:

Ö0 = H
BAH Ö C P. (III.41)

TFM: OLIMPIADAS MATEMÁTICAS SOBRE EL TRIÁNGULO Elisabeth González


36 CAP. III. OLIMPIADAS LOCALES

Y por otro lado sabemos de estos dos ángulos que:

Ö0 = BC
BAH × H 0 por abarcar ambos ángulos el mismo arco7 .

H
Ö CP = P
× C H 0 por ser H 0 simétrico de H respecto de AB.

Entonces:
BC
× H0 = P
× C H 0. (III.42)

Y esto último prueba que H 0 esta en la circunferencia circunscrita(2.2) el triángulo.

Ahora tenemos dos igualdades:

1. Como H 0 es simétrico de H, tenemos que:

OH 0 H = H 0 HO0 . (III.43)

2. Como H H 0 y OO0 son dos paralelas (por estar dentro de la altura(1.2) y de la mediatriz(1.1) respectiva-
mente) cortadas por la secante HO0 tenemos que:

H
Ù 0 HO 0 = OO
× 0 H, (III.44)

por ser ángulos alternos internos 8 .

Entonces por (1) y (2) tenemos que:


OH
Ø 0H = H
Ù 0 HO 0 = OO
× 0 H. (III.45)

Por otro lado OA y OH 0 son radios de la circunferencia circunscrita por ser A vértice del triángulo y por estar H 0
en dicha circunferencia (demostrado anteriormente). Entonces el triángulo H 0 OA es isósceles , lo que implica
que sus ángulos OH
× 0 A = OH
Ø H 0 AO = HAO
0H y × Õ son iguales.

Y teniendo en cuenta (III.45) la anterior igualdad se queda:

OO
× 0 H = HAO.
Õ (III.46)

Ya hemos probado que los ángulos opuestos A c0 en el cuadrilátero AHO0 O son iguales, nos quedaría ver
by O

que también los ángulos O


byH
Ò son iguales. Tenemos:

1.
O
× 0 HA = 180o − H
Ù 0 HO 0 , por ser AP la altura del triángulo.
7
Los ángulos inscritos2,5 que abarcan el mismo arco son iguales.
8
Se les llama ángulos alternos internos a los que, en una transversal que corta a dos paralelas (o a dos rectas), son internos a las
rectas pero alternos en la transversal.

22 de septiembre de 2014 Curso 20132014.


SEC. 7. PROBLEMAS 37

2.

O
Ö 0 OA = O
Ù 0 OH 0 + ×
H 0 OA

= OH
Ø 0H + ×
H 0 OA, por (III.45)

= OH
Ø 0 H + (180o − 2OH
× 0 A), por ser el triángulo H 0 OA isósceles

= OH
Ø 0 H + (180o − 2OH
Ø 0 H), véase el dibujo

= 180o − OH
Ø 0H

= 180o − H
Ù 0 HO 0 , por (III.45).

Entonces el cuadrilátero AHO0 O es un paralelogramo9 .

Y por lo tanto por ser paralelogramo:


AH = OO0 . (III.47)

Y como OO0 = 2OQ por ser O0 simétrico de O respecto de Q tenemos:

AH = 2OQ. (III.48)

Alternativa de uno de los Ejercicios internacionales

Ejercicio. 7.8.
Usando regla y compás construye el triángulo ABC conocidas: ha , h b y mc , donde:

ha es la altura1,2 del triángulo que parte del lado BC (véase (1.2)).

h b es la altura del triángulo que parte del lado AC (véase (1.2)).

mc es la mediana del triángulo que parte del vértice C (véase (1.4)).

SOLUCIÓN. La construcción se podría realizar como sigue:

1. Dibujamos la mediana mc y trazamos una circunferencia que pase por sus extremos.

2. Con centro en el pie de la mediana Mc , trazamos dos circunferencias, una con radio la semidistancia de
H a (que llamaremos c1) y otra con la semidistancia de H b ( que llamaremos c2).
9
Si ambos pares de ángulos opuestos de un cuadrilátero son congruentes, entonces el cuadrilátero es un paralelogramo.

TFM: OLIMPIADAS MATEMÁTICAS SOBRE EL TRIÁNGULO Elisabeth González


38 CAP. III. OLIMPIADAS LOCALES

3. Estos circunferencias cortarán a la circunferencia en dos puntos cada una. Nos quedaremos con un punto
de cada. Tomaremos el punto superior de la c1 (que lo llamaremos P) y el punto inferior de la c2 (que
lo llamaremos O).

4. Trazaremos una paralela al segmento CO a la distancia H b . Donde esta paralela corte al lado C P, estará
el vértice B buscado.

5. Trazaremos una paralela al segmento C P a la distancia H a . Donde esta paralela corte al lado CO ten-
dremos el vértice A buscado.

6. Como el vértice C lo tenemos, queda determinado el triángulo ABC.

B
B
P

C
Mc

A
D O

i
ƒ

22 de septiembre de 2014 Curso 20132014.


Capítulo IV

Olimpiadas Nacionales

8. Problemas

Ejercicio. 8.1. (Pamplona, 2011, Ver 40 en las Referencias web)


b = 2Cb y A
Sea ABC un triángulo con B b > 90o Sean:

D: punto de la recta AB tal que: C D⊥AC.

M : punto medio de BC.

Demuestra que A
Ö MB = ×
DM C.

SOLUCIÓN.
La recta que pasa por A y es paralela a BC corta a DM y a DC en los puntos N y F respectivamente.

Tenemos:
AN DN
= , (IV.1)
BM DM

aplicando el Teorema de Thales(4.2) al triángulo DBM , y tenemos:

DN NF
= , (IV.2)
DM MC

aplicando el Teorema de Thales(4.2) al triángulo M DC.


40 CAP. fIV. OLIMPIADAS NACIONALES

α N F

α
2α α C
B M

Combinando las dos últimas igualdades tenemos:

AN DN NF
= = . (IV.3)
BM DM MC

BM = M C por ser M el punto medio, por lo que AN = N F . Esto implica que N C es la mediana del triángulo
AF C (triángulo rectángulo por C D⊥AC). Entonces aplicando el Corolario 1 del Teorema de Thales(4.1) tene-
mos que N C = AN .

22 de septiembre de 2014 Curso 20132014.


SEC. 8. PROBLEMAS 41

Esto último implica que el triángulo AN C es isósceles, entonces:

N
Õ AC = N
Õ CA, por ser ANC isósceles

= AC
Õ B, por ser NCA y ACB ángulos alternos internos1 .

Entonces:
N
Ö C B = 2AC
Õ B = ABC,
Õ (IV.4)
b = 2C.
donde en la última igualdad se utilizo la hipótesis: B b

Por otro lado como AN ||BC (por construcción) tenemos que el cuadrilátero ABC N es isósceles.

Entonces los otros dos lados del trapecio isósceles son iguales (AB = N C) por definición de trapecio isósceles 2 .

Entonces por:

AB = N C, por ser los lados iguales del trapecio isósceles anterior.

N
Ö C B = ABC,
Õ por (IV.4).

BM = M C, por ser M el punto medio de dicho segmento.

podemos usar el tercer criterio de igualdad de triángulos(5) y tenemos que:

ABM = N M C, (IV.5)

por lo que:
A
Ö MB = N
× MC = ×
DM C. (IV.6)

La última igualdad podemos verla claramente en el dibujo.

Entonces ya tenemos lo que queríamos demostrar:

A
Ö MB = ×
DM C. (IV.7)

Ejercicio. 8.2. (Torrelodones, 2007, Ver 40 en las Referencias web)


Sea O el circuncentro1,1 de un triángulo ABC. La bisectriz1,3 que parte de A corta al lado opuesto en P.
Probar que se cumple:
AP 2 + OA2 − OP 2 = bc. (IV.8)

1
Se les llama ángulos internos a los que, en una transversal que corta a dos paralelas (o a dos rectas), son internos a las rectas
pero alternos en la transversal.
2
Un trapecio isósceles es el que tiene los lados no paralelos de igual medida

TFM: OLIMPIADAS MATEMÁTICAS SOBRE EL TRIÁNGULO Elisabeth González


42 CAP. IV. OLIMPIADAS NACIONALES
j

SOLUCIÓN.
Dibujamos la circunferencia circunscrita(2.2) al triángulo
ABC y prolongamos AP hasta que corte con dicha
A
circunferencia. A este punto de corte le llamamos M .

Por otro lado tenemos que:


c b
AC
Õ B=A Ö M B por estar inscritos2,5 en el mismo arco O
(véase (2.5)).
C
B
Ö AM = CAM
Ö por definición de bisectriz.
B P a

Entonces los triángulos ABM y AP C son semejantes por


el primer criterio de semejanza(5.1) , y esto implica que:
M
c AM
= . (IV.9)
AP b

Despejando c b tenemos que:

c b = AM AP

= (AP + P M ) AP, descomponiendo AM = AP + P M

= AP 2 + P M AP

= AP 2 + (OA2 − OP 2 ), por ser P M AP potencia de P respecto de la circunferencia, (véase (6))

= AP 2 + OA2 − OP 2 .

22 de septiembre de 2014 Curso 20132014.


SEC. 8. PROBLEMAS 43

Ejercicio. 8.3. (Sevilla, 2006, Ver 40 en las Referencias web)


Sea ABC un triángulo isósceles con AB = AC y P un punto cualquiera de la circunferencia tangente a los lados
AB en B y AC en C.
Pongamos a, b, c las distancias desde P a los lados BC, AC y AB respectivamente. Probar que:

a2 = bc. (IV.10)

SOLUCIÓN.
A

S
c
Pongamos: R
P b

m = P B y n = P C.
m a n
B C
Sean Q, R, S las proyecciones de P sobre los lados Q
BC, AC y AB respectivamente.

Sea P 0 el punto diametralmente opuesto a P.

P0

Vamos a demostrar que P BP 0 y P BS son semejantes:

Ambos triángulos son rectángulos por los que ambos tienen un ángulo recto, en concreto:

Ô=P
PSB BP 0 .
Ö (IV.11)

Si nos fijamos en el triángulo BP B:

1. Llamamos u = P Ö 0 P B = OP
Õ B donde O es el centro de la circunferencia y a la misma vez vamos a
tener en cuenta que OBP = OP
Õ Õ B = u por ser el triángulo OBP isósceles, (OP = OB).
2. Entonces el ángulo que nos queda por conocer de este triángulo es P
Ö P 0 B = 180o −90o −u = 90o −u.
3. Si nos fijamos ahora en el triángulo SBP tenemos que ÕP BS = OBS
Õ − OBP Õ = 90o − u, por tanto este
ángulo es igual a P
Ö P 0 B.

Entonces los triángulos P BP 0 y P BS son semejantes aplicando el primer criterio de semejanza de triángulos(5.1) :

m 2r
= , (IV.12)
c m

TFM: OLIMPIADAS MATEMÁTICAS SOBRE EL TRIÁNGULO Elisabeth González


44 CAP. IV. OLIMPIADAS NACIONALES

de donde se tiene que:


m2 = 2c r. (IV.13)

Análogamente se puede demostrar que los triángulos P C P 0 y PRC son semejantes y obtenemos:

n 2r
= , (IV.14)
b n

de donde se tiene que:


n2 = 2br. (IV.15)

Por otro lado aplicando el Teorema del Seno(II.10) al triángulo P BC tenemos:

n
2r = , (IV.16)
sen P
Õ BC

de donde se tiene:
n
sen P
Õ BC = . (IV.17)
2r

Y del triángulo rectángulo PQB obtenemos:

a
sen P BC = sen P
Õ BQ = , (IV.18)
m

de donde se tiene despejando a:


a = sen P
Õ BC m

nm
= , por (IV.17).
2r

Si elevamos al cuadrado:
n2 m2
a2 =
4r 2

2c r2br
= , por (IV.13) y (IV.15)
4r 2

= bc.
ƒ

22 de septiembre de 2014 Curso 20132014.


SEC. 8. PROBLEMAS 45

Ejercicio. 8.4. (Santiago de Compostela, 2005, Ver 40 en las Referencias web)


En un triángulo de lados a, b, c donde el lado a es la media aritmética de b y c. Probar:

1. 0o ≤ A
b ≤ 60o .

2. La altura relativa al lado a es tres veces el inradio(2.1) r.

3. La distancia del circuncentro(2.2) al lado a es: R − r (donde R es el radio de la circunferencia


circunscrita(2.2) ).

SOLUCIÓN.

1. Por la desigualdad triángular(4.8) tenemos:


b+c
b≤a+c = + c, de donde se tiene que:
2
b
≤ 3. (IV.19)
c
b+c
c ≤a+b= + b, de donde se tiene que:
2
b 1
≥ . (IV.20)
c 3

Entonces tenemos:
1 b
≤ ≤ 3. (IV.21)
3 c

Por otro lado por el Teorema del coseno(4.6) tenemos:

c + b2
a2 = b2 + c 2 − 2bc cosA
b= + c 2 − 2bc cosA
b,
4

de donde despejando cosA


b se tiene:

−3b2 − 3c 2 + 2bc
b=
cosA . (IV.22)
−8bc

b
Dividimos por c 2 numerador y denominador, llamamos por comodidad x = y obtenemos:
c
−3x 2 + 2x − 3 3x 2 − 2x + 3 3 1 3 1 b
f (x) = cosA
b= = = x− + con ≤ ≤ 3. (IV.23)
−8x 8x 8 4 8x 3 c

TFM: OLIMPIADAS MATEMÁTICAS SOBRE EL TRIÁNGULO Elisabeth González


46 CAP. IV. OLIMPIADAS NACIONALES

Tenemos que:

1 1 1 9
 ‹
f = − + = 1,
3 8 4 8
9 1 1
f (3) = − + = 1,
8 4 8
3 3
f 0 (x) = − ,
8 8x 2
3 1 3 1
 ‹
de donde igualando a cero obtenemos que hay un mínimo en x = 1 f (1) = − + = .
8 4 8 2

Entonces nos queda:


1
≤ cos A
b ≤ 1. (IV.24)
2

Análogamente:
0o ≤ A
b ≤ 60o . (IV.25)

2. Designando:
A, B y C a los vértices opuestos a los lados
a, b y c, respectivamente.

I al incentro(1.3) .
A
(1.2) O
ha a la altura correspondiente al lado a. T
b
r R
S al area(3) del triángulo. ha c
da
I
p al semiperímetro(I.12) . B a S
C

r al inradio(2.1) .

Tenemos:

S = pr, ver (I.14)

a+b+c
= r, ver semiperímetro (I.12)
2
a  b+c
= + a r, usando a =
2 2
3a
= r.
2

22 de septiembre de 2014 Curso 20132014.


SEC. 8. PROBLEMAS 47

1
S= aha , ver (I.13).
2

Igualando ambas fórmulas tenemos:

ha = 3r. (IV.26)

3. Sea d0 la distancia entre el circuncentro(1.1) y el lado a.


En el triángulo rectángulo SOC sabemos por el Teorema de Pitágoras(4.1) que:

 a 2
R2 = da2 + , (IV.27)
2

de donde se tiene:

a2
da2 = R2 − . (IV.28)
4

Por otro lado en el triángulo rectángulo AI T tenemos:

d= r
tan A/2
AT
r
= , ver propiedad de la circunferencia inscrita (2.1)
p−a
r
= , ver semiperímetro (I.12)
a + b + c − 2a
2
r r 2r
= a = a = .
− +a a
2 2

Por lo que nos queda:

d= 2r
tan A/2 . (IV.29)
a

TFM: OLIMPIADAS MATEMÁTICAS SOBRE EL TRIÁNGULO Elisabeth González


48 CAP. IV. OLIMPIADAS NACIONALES

Ahora tenemos:
a
2R = , usando el Teorema del Seno generalizado en el triángulo ABC (II.10)
sen A
a
= , véase el seno en función de la tangente del ángulo mitad 3
2 tan A/2
d
€ Š2
1 + tan A/2
d

€ Š2
1 + tan A/2
d
=a
2 tan A/2
d

‹2
2r

1+
a
=a , por (IV.29).
2r
2
a

a2 + 4r 2
a2 a2 + 4r 2 a2
=a = = + r.
4r 4r 4r
a

Por lo que tenemos:


a2
= r(2R − r). (IV.30)
4

Volviendo a (IV.28) y sustituyendo (IV.30) tenemos:

da2 = R2 − 2Rr + r 2 = (R − r)2 . (IV.31)

de donde se tiene:
da = R − r. (IV.32)

2 tan A/2
d
3
Seno en función de la tangente del ángulo mitad: sen A = € Š2 .
1 + tan A/2
d

22 de septiembre de 2014 Curso 20132014.


SEC. 8. PROBLEMAS 49

Vamos a ver otra solución de este apartado sin usar trigonometría.

Sea S la intersección de la bisectriz de A con la mediatriz de a A


(que esta en el punto medio del arco BC). O
b
P3
Y sean P, P2 y P3 los puntos de tangencia de la circun- c
P2 I
ferencia inscrita(2.1) en el triángulo y los lados a, c y b, A0
C
respectivamente. B P Q a

Usando una propiedad de la circunferencia inscrita(2.1) llamamos:

x = P B = BP2
y = C P = C P3
z = AP2 = AP3

y tenemos:
b+c
a=x+y= , por ser a la media aritmética de b y c.
2
b = y +z → y = b−z
c=z+x →z=c−x

Entonces tenemos:
b+c
x= − y, despejando x en (3)
2

b+c
= − b + z, sustituyendo y por su valor (3)
2

b+c
= − b + c − x, sustituyendo z por su valor (3)
2

b + c − 2b + 2c
= − x.
2

Despejando x se tiene:
3c − b
x= . (IV.33)
4

Aplicando el Teorema de la bisectriz(4.7) al triángulo ABC tenemos:


c BQ
= . (IV.34)
b CQ

TFM: OLIMPIADAS MATEMÁTICAS SOBRE EL TRIÁNGULO Elisabeth González


50 CAP. IV. OLIMPIADAS NACIONALES

Sabemos que:

b+c
BQ + CQ = a = . (IV.35)
2
Dividimos por BQ y aplicamos (IV.34):

b b+c
1+ = . (IV.36)
c 2BQ

Despejamos BQ y nos queda:


c
BQ = . (IV.37)
2
Como BA0 es la mitad de a:
a b+c
BA0 = = . (IV.38)
2 4

Vamos a calcular:

QA0 − PQ = (BA0 − BQ) − (BQ − BP), (véase el dibujo)

b+c c
= − 2 + x, usándose (IV.37) y (IV.38)
4 2

b+c c(b + c) 3c − b
= −2 + , usándose (IV.38)
4 2(b + c) 4

b + c − 4c + 3c − b
=
4

= 0.

Por lo que QA0 = PQ.

Y como:

PQ = QA
IÔ Ö 0 S = 90o .

Ô =A
IQP Ö0 QS por ser ángulos alternos internos 4 .

Entonces con por las tres últimas igualdades tenemos que los triángulos P IQ y AS 0Q son iguales por el
primer criterio de igualdad de triángulos(5) , por tanto A0 S = I P = r, de donde queda finalmente:

OA0 = OS − A0 S = R − r. (IV.39)
ƒ

4
Se les llama ángulos internos a los que, en una transversal que corta a dos paralelas (o a dos rectas), son internos a las rectas
pero alternos en la transversal.

22 de septiembre de 2014 Curso 20132014.


SEC. 8. PROBLEMAS 51

Ejercicio. 8.5. (Canarias, 2003, Ver 40 en las Referencias web)


Las alturas(1.2) del triángulo ABC se cortan en el punto H. Se sabe que AB = C H. Determinar el valor del
ángulo ÕBCA.

SOLUCIÓN. Distinguimos tres casos:

Cb < 90o : Llamemos A0 y C 0 a los puntos en los que las alturas de A y C cortan al lado opuesto respectivamente.
Tenemos:
C HA0 = AH
El ángulo × Õ C porque son ángulos opuestos por
el vérticea . A
90o -α
En el triángulo CA0 H el ángulo ×
CA0 H es recto, entonces
el ángulo ×H CA0 = 180o − 90o − α = 90o − α. C0 α
b
c H

En el triángulo AH C 0 el ángulo ×
H C 0 A es recto, entonces
α
el ángulo ×HAC 0 = 180 − 90 − α = 90o − α.
o o

90o -α
a
Dadas dos rectas r y s, del plano, que se cortan en el punto P, dos ángu- B a C
A0
los se dicen opuestos por el vértice cuando los lados de uno son semirrectas
opuestas a los lados del otro. Dos ángulos opuestos por el vértice son con-
gruentes.

A
90o -α
Por otro lado tenemos:
C0 α
c H b
El ángulo × HAC 0 es igual al ángulo A
Õ0 AB del triángulo
0
rectángulo A AB, entonces: α
α 90o -α
A0 BA = 180o − 90o − ×
HAC 0 = 180o − 90o − (90o − α) = α. C
Õ
B A0 a

Entonces los triángulos C HA0 y A0 AB son iguales aplicando el primer criterio de igualdad de triángulos(5)
(C H = AB por ser una condición del enunciado y los ángulos adyacentes de los lados C H y AB son igua-
les).

Por tanto todos los lados son iguales, en particular AA0 = CA0 .
Si calculamos la tangente del ángulo Cb en el triángulo AA0 C tenemos:

AA0
tan Cb = = 1. (IV.40)
CA0

Análogamente:
Cb = 45o . (IV.41)

TFM: OLIMPIADAS MATEMÁTICAS SOBRE EL TRIÁNGULO Elisabeth González


52 e CAP. IV. OLIMPIADAS
d NACIONALES
f

Cb > 90o : Vamos a razonar análogamente al apartado anterior. Llamemos A0 , B 0 y C 0 a los puntos en los que
las alturas de A, B y C cortan al lado opuesto respectivamente, tenemos:
A
El ángulo A 0C H = ×
C 0 C B porque son ángulos opuestos C0
×
a c
por el vértice . 90o -α b
α A0
0 B a
CA0 H es recto, entonces
En el triángulo CA H el ángulo × C α
el ángulo A
×0 H C = 180 − 90 − α = 90o − α.
o o

B0
En el triángulo C C 0 H el ángulo C
× C 0 H es recto, entonces
el ángulo ×C 0 BC = 180o − 90o − α = 90o − α. 90o -α
a
Dadas dos rectas r y s, del plano, que se cortan ene el punto P, dos ángu- d f
H
los se dicen opuestos por el vérticeángulos!opuestos por el vértice cuando
los lados de uno son semirrectas opuestas a los lados del otro.Dos ángulos
opuestos por el vértice son congruentes.

A
0
c C
90o -α b
α A0
Por otro lado tenemos: a
B C
α
El ángulo × C 0 BC es igual al ángulo ABA
Õ0 del triángulo
0
rectángulo AA B, entonces: B0

Õ0 = 180o − 90o − ×
BAA C 0 BC = 180o − 90o − (90o − α) = α. 90o -α

Entonces los triángulos AA0 B y A0 C H son iguales aplicando el primer criterio de igualdad de triángulos(5)
(C H = AB por ser una condición del enunciado y los ángulos adyacentes de los lados C H y AB son igua-
les).
Por tanto todos lados son iguales, en particular AA0 = CA0 .

Õ0 en el triángulo ACA
Si calculamos la tangente del ángulo ACA Õ0 tenemos:

Õ0 = AA0
tan ACA =1 (IV.42)
CA0

Por tanto por una de las propiedades de dos ángulos suplementarios tenemos:

b = tan (180o − ACA


tan (C) Õ0 ) = − tan (ACA
Õ0 ) = −1 (IV.43)
4
La tangente de dos ángulos complementarios verifica: tan (180o − α) = − tan α.

22 de septiembre de 2014 Curso 20132014.


SEC. 8. PROBLEMAS 53

Análogamente:
Cb = 135o f de (IV.44)

Cb = 90o : En este caso C coincide con H.

Por lo que C H = 0. c
b

Como AB 6= 0, entonces este valor de Cb no puede darse,


g
por lo que este caso no es válido.
B a C=H

Ejercicio. 8.6. (Ciudad Real, 2004, Ver 40 en las Referencias web)


Demostrar que la condición necesaria y suficiente para que, en el triángulo ABC, la mediana(1.4) desde B sea
dividida en tres partes iguales por la circunferencia inscrita(2.1) en el triángulo, es:

a b c
= = . (IV.45)
5 10 13

SOLUCIÓN.

1. Primero demostraremos la condición necesaria.


Partimos de un triángulo ABC tal que la mediana C K C
(donde K es el punto medio de AC) corte a la circunfe- T
b
a
rencia inscrita en dos puntos M y N tales que: I K

N
BM = M N = N K = x. (IV.46) M H A
B c
Sea T el punto de tangencia de la circunferencia inscrita
y el lado BC.

En el triángulo se verifican las siguientes relaciones:

a + c − b = 2BT. (IV.47)

(Fórmula que deducimos directamente de la propiedad de la circunferencia inscrita(2.1) ).

TFM: OLIMPIADAS MATEMÁTICAS SOBRE EL TRIÁNGULO Elisabeth González


54 CAP. IV. OLIMPIADAS NACIONALES

2a2 + 2c 2 − b2 = 4BK 2 . (IV.48)


Fórmula de Apolonio(4.9) correspondiente a la mediana del lado b y multiplicada por dos. Y como
BK = 3x (véase (IV.46)), tenemos:

2a2 + 2c 2 − b2 = 4(3x)2 = 36x 2 . (IV.49)

La potencia del vértice B respecto de la circunferencia inscrita se puede escribir de dos maneras teniendo
en cuenta el apartado (6) con su caso especial.

BT 2 = BM BN . (IV.50)

Con lo que tenemos:

BT 2 = BM BN
‹2
a+c−b

= BM BN , sustituyendo el valor de BT (IV.47)
2
‹2
a+c−b

= x2x, teniendo en cuenta (IV.46).
2

Con lo que nos queda:


(a + c − b)2 = 8x 2 . (IV.51)

En el triángulo ABC, los puntos B y K están igualmente alejados del centro de la circunferencia inscrita.
Demostrémoslo:

a) Trazamos la perpendicular a BK que pasa por I.


b) Llamamos H al punto donde esta perpendicular corta a BK.
c) Entonces los triángulos rectángulos en H (I H K, I H B) son semejantes aplicando el tercer criterio
de semejanza de triángulos(5.1) por tener el lado I H común y el lado BH = K H (por ser I H perpen-
dicular a un arco de la circunferencia5 ) y además el ángulo comprendido entre los lados iguales
ser igual.

Entonces todos los lados son iguales, en particular BI = K I.

a) El lado I C es común en ambos triángulos.


b) BI = K I (lo acabamos de demostrar).
c) C
Ô IB = Õ
C I K porque K
Õ IH = Õ
BI H
d) Entonces los triángulos C K I y C I B son semejantes aplicando el tercer criterio de semejanza de
triángulos(5.1) .
5
Cualquier recta perpendicular que pase por el punto medio de cualquier cuerda de una circunferencia pasa por el centro de dicha
circunferencia.

22 de septiembre de 2014 Curso 20132014.


SEC. 8. PROBLEMAS 55

Entonces todos los lados son iguales, en particular BC = KC. Por lo que tenemos:

b = 2a. (IV.52)

Sustituyendo esta última igualdad en (IV.49) y (IV.51) tenemos:

2a2 + 2c 2 − 4a2 = 36x 2 .

Simplificando y dividiendo entre dos tenemos:

c 2 − a2
c 2 − a2 = 18x 2 → x 2 = . (IV.53)
18

(a + c − 2a)2 = 8x 2 .

Simplificando tenemos:
(c − a)2
(c − a)2 = 8x 2 → x 2 = . (IV.54)
8

Igualando las dos igualdades de x 2 tenemos:

c 2 − a2 (c − a)2
= ,
18 8

y nos queda:
c+a 9
= . (IV.55)
c−a 4

Porque sabemos que c − a 6= 0 y haciendo cálculos nos queda:

c 13
= . (IV.56)
a 5

Uniendo esta última igualdad con (IV.52) tenemos:

a b c
= = . (IV.57)
5 10 13

2. Ahora demostraremos la condición suficiente.


Podemos suponer sin perdida de generalidad:

a=5
b = 10
c = 13

TFM: OLIMPIADAS MATEMÁTICAS SOBRE EL TRIÁNGULO Elisabeth González


56 CAP. IV. OLIMPIADAS NACIONALES

C
T2
T

a
I b
K

H
N
M
T3
B
c A

Antes de empezar con este apartado vamos a resolver el siguiente sistema (que tiene que ver con una
propiedad de la circunferencia inscrita(2.1) ):

x + y = 5
y + z = 10 donde: x = BT = BT 3, y = C T = C T 2 y z = AT 2 = AT 3,
z + x = 13 
cuya solución es: x = 4, y = 1 y z = 9.
Estos valores los usaremos más tarde para terminar el apartado.

Ahora sustituyendo en las fórmulas (IV.48) y (IV.49) usadas en la condición necesaria tenemos:

a + c − b = 2BT → 8 = 2BT → BT = 4 → BT 2 = 16. (IV.58)

p
2a2 + 2c 2 − b2 = 4BK 2 → 50 − 100 + 338 = 4BK 2 → 4BK 2 = 288 → BK = 6 2. (IV.59)

Resumiendo tenemos: p
BK = 6 2, (IV.60)
y
16 = BT 2 = BM BN . (IV.61)

Si calculamos el semiperímetro(I.12) tenemos que p = 14 y usando esto calculamos la superficie(3) del


triángulo mediante la fórmula de Herón(II.44) :
Æ p
S = 14(14 − 5)(14 − 10)(14 − 13) = 6 14. (IV.62)

Despejando r (radio de la circunferencia inscrita) de la fórmula el área(I.14) de un triángulo en función


del radio de su circunferencia inscrita tenemos:
p
S 6 14 6
r= = =p . (IV.63)
p 14 14

22 de septiembre de 2014 Curso 20132014.


SEC. 8. PROBLEMAS 57

El triángulo BC K es isósceles por ser BC = C K (véase (1)) y C I común, entonces la bisectriz del ángulo
C es también altura6 . Llamemos H al pie de dicha altura.
Consideremos el triángulo rectángulo BI T en el ángulo B Ô T I por ser T el punto de tangencia de la
circunferencia inscrita, entonces aplicando el Teorema de Pitágoras(4.1) :

BI 2 = BT 2 + C I 2

= r 2 + 42

36
= + 16, teniendo en cuenta (2)
14

130
= .
7

I H B por ser C H la altura del triángulo C BK.


Por otra parte el triángulo es rectángulo BI H en el ángulo Õ
Entonces aplicando el Teorema de Pitágoras.

BI 2 = H B 2 + H I 2 . (IV.64)

Por lo que:
H I 2 = BI 2 − H B 2

130
= − H B2, por (2)
7

130
= − (BK/2)2 , por ser H el punto medio de B y K
7

130 p
= − (3 2)2 , por (IV.59)
7

4
= .
7
Y finalmente en el triángulo I H M aplicando Pitágoras y (2):

36 4
H M 2 = I M 2 − I H2 = − = 2. (IV.65)
14 7

6
En un triángulo isósceles la bisectriz del ángulo opuesto a la base, es perpendicular a la base. La bisectriz coincide con la altura
correspondiente al lado AB.

TFM: OLIMPIADAS MATEMÁTICAS SOBRE EL TRIÁNGULO Elisabeth González


58 CAP. IV. OLIMPIADAS NACIONALES

p
Como H es el punto medio de M y N tenemos que M N = 2 2, y por otro lado tenemos:

BM = BH − M H

= (BK/2) − M H
p
= 3 2 − M H, usando (IV.59)
p p
= 3 2 − 2, usando (2)
p
= 2 2.

KN = KN − MH

= (BK/2) − N H
p
= 3 2 − N H, usando (IV.59)
p p
= 3 2 − 2, usando (2)
p
= 2 2.

Por tanto queda demostrado que BM = M N = N K.


ƒ

Ejercicio. 8.7. (Girona, 2009, Ver 40 en las Referencias web)


Sean:

ABC un triángulo acutángulo 7 .

I el centro de la circunferencia inscrita(2.1) .

r el radio de la circunferencia inscrita.

R el radio de la circunferencia circunscrita(2.2) .

Se traza la altura(1.2) AD = ha con D perteneciente al lado BC.


Demuestra:
DI 2 = (2R − ha )(ha − 2r). (IV.66)

22 de septiembre de 2014 Curso 20132014.


SEC. 8. PROBLEMAS 59

SOLUCIÓN. Sean:

E y M las proyecciones ortogonales de I sobre BC y AD respectivamente.

F el punto de tangencia de la circunferencia inscrita con el lado AC.

Entonces si nos centramos en el triángulo rectángulo AF I tenemos:


 
A
b r
sen = , (IV.67)
2 AI

por ser AI una bisectriz del triángulo ABC (al ser I el incentro(1.3) ).

Por otro lado vamos a usar dos fórmulas diferentes para el área del triángulo ABC.

1. Vamos a despejar r de la fórmula del área(I.14) en función del inradio:


S
r= . (IV.68)
p

2. Vamos a usar la fórmula del área(I.16) conociendo dos lados y el ángulo que lo forman:
b c sen A
S=
2

2 tan bA2
bc  2
A
1 + tan 2 b
= , véase el seno en función de la tangente del ángulo mitad 8
2

sen A2
bc
cos A2 b c sen A2 cos2 A
2
= A
= = b c sen A2 cos A2 .
sen2 2 cos A2
1+
cos2 A2

Sustituyendo primero este valor de r y luego el de S en (IV.67), y despejando AI en la misma tenemos:

b c sen A2 cos A2 b c cos 2A


AI = = . (IV.69)
p sen A2 p

Elevamos esta expresión al cuadrado:


A
b2 c 2 cos2 2
AI 2 = . (IV.70)
p2

2 tan A/2
d
8
Seno en función de la tangente del ángulo mitad: sen A = € Š2 .
1 + tan A/2
d

TFM: OLIMPIADAS MATEMÁTICAS SOBRE EL TRIÁNGULO Elisabeth González


60 CAP. IV. OLIMPIADAS NACIONALES

Ahora teniendo en cuenta que:


p(p − a)
cos2 A/2 = . (IV.71)
bc
tenemos:
p(p − a)
b2 c 2
bc bc(p − a)
AI 2 = 2
= . (IV.72)
p p

Ahora también vamos a tener en cuenta otra serie de cosas:

Despejando el semiperímetro de la fórmula del área(I.14) del triángulo en función del inradio tenemos:
S
p= . (IV.73)
r

Despejando la base a de la fórmula clásica del área(I.13) del triángulo tenemos:


2S
a= . (IV.74)
ha

Haciendo cálculos en la fórmula del área(I.15) del triángulo en función del circunradio tenemos:

a bc
S=
4R

a bc
aha = .
4R

Despejando bc tenemos:
bc = 2Rha . (IV.75)

Entonces sustituyendo estas tres últimas expresiones la expresión (IV.72) tenemos:

bc(p − a)
AI 2 =
p

S 2S
 ‹
2Rha −
r ha
=
S
r

Sha − r2S
 ‹
2Rha
rha
=
S
r

= 2R(ha − 2r).

22 de septiembre de 2014 Curso 20132014.


SEC. 8. PROBLEMAS 61

Como el cuadrilátero I E DM es un rectángulo (M D = IR = r), aplicando el Teorema de Pitágoras (4.1) a ADI,


tenemos:
DI 2 = h2a + AI 2 − 2ha AM

= h2a + AI 2 − 2ha (ha − M D), observando el dibujo

= h2a + 2R(ha − 2r) − 2ha (ha − r), teniendo en cuenta (8)

= (2R − ha )(ha − 2r).


ƒ

Ejercicio. 8.8. (Murcia, 2001, Ver 40 en las Referencias web)


Sea P un punto interior del triángulo ABC, de modo que ABP verifica AP = BP.
Sobre cada uno de los otros dos lados se construyen, exteriormente, triángulos BQC y CRA, semejantes al
triángulo ABP, cumpliendo BQ = QC y CR = RA respectivamente.
Probar que los puntos P, Q, C y R, o están alineados, o son los vértices de un paralelogramo.

SOLUCIÓN.
Veamos que los triángulos ABC y P BQ son semejantes aplicando el tercer
criterio de semejanza(5.1) :

A
Ô BP = C
Õ BQ por ser los triángulos AP B y BQC semejantes. Sumamos
la misma cantidad a ambos miembros:
A
A
ÔBP + P
Õ BC = C
Õ BQ + P
Õ BC.

Entonces tenemos: c

R
Õ=P
ABC BQ.
Õ P
b
B
Y además los lados que forman dichos ángulos son proporcionales:

c BP a
= , por ser los triángulos APB y BQC semejantes.
a BQ Q C

Entonces ABC y P BQ semejantes, y por ello tenemos que todos sus ángulos
son iguales, en particular:

Õ = AC
BQP Õ B. (IV.76)

Análogamente tenemos que ABC es semejante a APR siguiendo el mismo razonamiento. Entonces todos sus
ángulos son iguales y en particular:
A
ÔRP = AC
Õ B. (IV.77)

TFM: OLIMPIADAS MATEMÁTICAS SOBRE EL TRIÁNGULO Elisabeth González


62 CAP. IV. OLIMPIADAS NACIONALES

Entonces si unimos estas dos semejanzas de triángulos tenemos que los triángulos P BQ y APR son semejantes
y como P B = PA, entonces todos sus lados tienen que ser iguales y por ello los triángulos son iguales por el
tercer criterio de igualdad de triángulos(5) .

Por otro lado vamos a llamar α = B ÔAP = A Ô BP por ser ABP isósceles al tener dos ángulos iguales. Enton-
ces tenemos que los cuatro ángulos del paralelogramo miden:

Õ = Cb + 2α.
QCR

Õ = 360o − A
QPR Ô PB − A
Ô PR − BPQ
Õ

= 360o − (180o − 2α) − A


Ô PR − BPQ,
Õ

teniendo en cuenta que A


Ô PB + B
Ô AP + PÔ
BA = 180o .

= 2α + (180o − B b),
b−A

por la igualdad de los ángulos de los triángulos semejantes ABC, BPQ y APR

= 2α + Cb

teniendo en cuenta que b+ B


A b + Cb = 180o

PRC = 180o − 2α − A
Õ ÔRP

= 180o − 2α − C.
b

Õ = 180o − 2α − BQP
PQC Õ

= 180o − 2α − C.
b

Tenemos que los ángulos opuestos del cuadrilátero PQCR son iguales, entonces PQCR es un paralelogramo9 .

La alineación es un caso particular y se producirá cuando:

180o − Cb
Cb + 2α = 180o → α = . (IV.78)
2
ƒ

9
Si ambos pares de ángulos opuestos de un cuadrilátero son congruentes, entonces el cuadrilátero es un paralelogramo.

22 de septiembre de 2014 Curso 20132014.


Capítulo V

Olimpiadas Internacionales

9. Problemas

9.1. Segunda Olimpíada Internacional de Matemáticas,1960 (Rumania)

La segunda Olimpíada Internacional de Matemáticas se celebró del 18 al 25 Julio de 1960 en las ciudades de
Sinaia y Bucarest.
Los países participantes fueron: Bulgaria, Checoslovaquia, Alemania Oriental, Hungría y Rumania.

Ejercicio. 9.1. (Hungría,[2, Enunciado, solución: pág 12,15, Ejercicio 2.4])


Usando regla y compás construye el triángulo ABC conocidas: ha , h b y ma , donde:

ha : altura1,2 del triángulo que parte del lado BC.

h b : altura del triángulo que parte del lado AC.

ma : mediana1,4 del triángulo que parte del vértice A.

SOLUCIÓN.

Denotamos: K

AH = ha . c ha b L

BK = h b . hb
ma
hd
AD = ma . H C
B D a
64 CAP. V. OLIMPIADAS INTERNACIONALES

En el triángulo ADC sea L el pie de la altura que parte del lado AC, hd .

Entonces:

1. AL D es un triángulo rectángulo por la definición de altura.

2. Consideremos los triángulos ADC y ABC. Entonces:

El área del triángulo ADC es la mitad que la del triángulo ABC, (véase la propiedad de la mediana
(1.4)).

Tomemos como base de ambos triángulos el lado AC.

Entonces la altura del triángulo ADC debe ser la mitad que la del triángulo ABC.

1
DL = hb. (V.1)
2

Por lo que una construcción del triángulo ABC puede ser la siguiente:

Trazamos una recta: l.

Tomamos un punto de la recta l: D ∈ l.

Trazamos una recta paralela a l y a la distancia ha de l: l 0 .

Dibujamos la circunferencia de centro D y radio ma siempre que ha < ma y obtenemos dos puntos de
corte de la circunferencia con la recta l 0 : A y A0 .

Nos quedamos con el punto A.

AD
Dibujamos otra circunferencia con diámetro AD de centro el punto medio de A y D y radio .
2

hb hb
Inscribimos en esta última circunferencia el triángulo AD L con D L = siempre que < ma .
2 2

Alargamos el lado AL del triángulo AD L hasta que corte con la recta l. El punto de corte es el punto C.

El punto D es el punto medio de B y C. Por lo que calculamos el simétrico de C respecto de D y obtenemos


el punto B.

Así queda determinado el triángulo ABC.

22 de septiembre de 2014 Curso 20132014.


SEC. 9. PROBLEMAS 65
b2

A
l0
c L b
E

l B D C
a

Si el ejercicio en lugar de dar la mediana que parte del vértice de A, diera la que parte de C, el ejercicio sería
mucho más fácil y lo podríamos poner como nivel local. Por lo que podemos encontrarlo en el capitulo de
Problemas de Olimpiadas a nivel local (7.8.).

9.2. Tercera Olimpíada Internacional de Matemáticas, 1961(Hungría)

La tercera Olimpíada Internacional de Matemáticas se celebró del 18 al 25 de Julio de 1961 en las ciudades
de Budapest y Veszprém.
Los países participantes fueron: Bulgaria, Checoslovaquia, Alemania Oriental, Hungría, Polonia y Rumania.

Ejercicio. 9.2. (Alemania oriental,[2, Enunciado, solución: pág 21,23, Ejercicio 3.4])
Sea P1 P2 P3 un triángulo y P un punto interior del mismo.
Denotamos a Q 1 , Q 2 y Q 3 los puntos de intersección de P1 P, P2 P, P3 P con los lados opuestos P2 P3 , P3 P1 , P1 P2 ,
respectivamente.

P1

Demuestra que entre estas tres relaciones:


Q3
Q2
P
P1 P P2 P P3 P
, , ,
PQ 1 PQ 2 PQ 3
existe una mayor o igual que dos y otra menor o
igual que dos.
P2 Q1 P3

TFM: OLIMPIADAS MATEMÁTICAS SOBRE EL TRIÁNGULO Elisabeth González


66 CAP. V. OLIMPIADAS INTERNACIONALES

SOLUCIÓN. Sean:

G el baricentro(1.4) del triángulo P1 P2 P3 .

R i el punto de intersección de Pi G con el lado opuesto respectivamente, i = 1, 2, 3.

Distinguimos dos casos:

1. P = G. Entonces Q i = R i , por lo que tenemos:

P1 G P G P3 G
= 2 = = 2, véase la propiedad de la mediana (1.4).
GR1 GR2 GR3

P1

R3 R2

P2 R1 P3

2. P 6= G. Entonces P es un punto interior de los seis triángulos que determinan las medianas, pertenece a
una de las medianas o es un punto del borde de dichos triángulos.
Suponemos que P ∈ int(P1 GR2 ). Entonces:

Sea S el punto de intersección de P1Q 1 y la paralela a P2 P3 que pasa por el punto G.

P1 P P1 S P1 G
< = = 2,
PQ 1 SQ 1 GR1
donde la primera igualdad se da por ser los triángulos P1 GS y P1 R1Q 1 semejantes por estar en
posición de Thales(5.1) y la última igualdad por una propiedad de la mediana(1.4) .

P1

P
R3 R2

S
i G

P2 R1 Q1 P3

22 de septiembre de 2014 Curso 20132014.


SEC. 9. PROBLEMAS 67

P2 P P G
> 2 = 2,
PQ 2 GR2
donde la igualdad vuelve a darse por la misma propiedad de la mediana(1.4) .

P1

Q2
P
R3 R2

P2 R1 P3

En el caso de que el punto P estuviera dentro de otro de los triángulos, la demostración sería análoga.

Supongamos que P esté en el borde del triángulo P1 GR2 (es decir, en la mediana del triángulo que
parte de P1 ). Entonces Q 1 = R1 , y por tanto:
P1 P P1 G
> = 2,
PR1 GR1
donde la igualdad vuelve a darse por la misma propiedad de la mediana.

P1

R3 R2

P2 R1 P3

9.3. Sexta Olimpíada Internacional de Matemáticas, 1964 (URSS, Unión de Repúblicas So-
cialistas Soviéticas)

La sexta Olimpíada Internacional de Matemáticas se celebró del 30 de Junio al 10 de Julio de 1964 en las
ciudades de Moscú.
Los países participantes fueron: Bulgaria, Checoslovaquia, Alemania Oriental, Hungría, Mongolia, Polonia,
Rumanía y Yugoslavia.

TFM: OLIMPIADAS MATEMÁTICAS SOBRE EL TRIÁNGULO Elisabeth González


68 CAP. V. OLIMPIADAS INTERNACIONALES

Ejercicio. 9.3. (Yugoslavia,[2, Enunciado, solución: pág 39,41, Ejercicio 6.3])


Sea ABC un triángulo y a, b, c las longitudes de sus lados. Las rectas tangentes a la circunferencia inscrita2,1 en
el triángulo, las cuáles son paralelas a los lados, dividen a ABC en tres pequeños triángulos. En cada triángulo
pequeño consideramos la circunferencia inscrita.
Calcula la suma de las áreas de las cuatro circunferencias inscritas.

SOLUCIÓN.
Sea K L la recta tangente a la circunferencia inscrita del triángulo A
ABC, la cuál es paralela al lado BC y sea rA el radio de la circunfe-
rencia inscrita en el triángulo AK L. IA
K L

De la misma forma, sean: c b


P I J
rB el radio de la circunferencia inscrita en el triángulo P BQ.
IC
rC el radio de la circunferencia inscrita en el triángulo J M C. IB
B Q a M C
Y sea r el inradio(2.1) del triángulo ABC.

Los triángulos ABC y AK L son semejantes porque están en posición de Thales(5.1) , por lo que la razón de
semejanza es:
r
. (V.2)
rA
Esta razón de semejanza coincide con la correspondiente a las alturas de los triángulos:

h r
= , (V.3)
h − 2r rA

donde:

h es la altura1,2 del triángulo ABC.

2r coincide con la altura del cuadrilátero BK LC.

Entonces:
r(h − 2r) rh − 2r 2 2r 2
rA = = =r− . (V.4)
h h h

Igualando las fórmulas:

Área(3) del triángulo.

Área(I.14) del triángulo en función del radio de la circunferencia inscrita.

22 de septiembre de 2014 Curso 20132014.


SEC. 9. PROBLEMAS 69

y despejando h tenemos:
2r p
h= . (V.5)
a

Entonces:
2r 2 2r 2 a rp − ra r
rA = r − =r− = = (p − a). (V.6)
2r p 2r p p p
a

De la misma manera:

r
rB = (p − b).
p

r
rC = (p − c).
p

La suma de las áreas de las cuatro circunferencias inscritas es:



T = π r 2 + rA2 + rB2 + rC2
 ‹2 
 ‹2  ‹2 
r r r
=π r + (p − a) +
2
(p − b) + (p − c)
p p p

(p − a)2 + (p − b)2 + (p − c)2


 
= πr 2
1+
p2

p2 − 2ap + a2 + p2 − 2a b + b2 + b2 + p2 − 2pc + c 2
 
= πr 2
1+
p2

4p2 + a2 + b2 + c 2 − 2p(a + b + c)
 
= πr 2
, usando(a + b + c) = 2p, (ver semiperímetro(I.12) )
p2

a2 + b2 + c 2
 
= πr 2 .
p2

Igualando las fórmulas:

Fórmula del Herón(II.44) .

Área(I.14) del triángulo en función del radio de la circunferencia inscrita.

y despejando r 2 tenemos:
r 2 = p(p − a)(p − b)(p − c), (V.7)

TFM: OLIMPIADAS MATEMÁTICAS SOBRE EL TRIÁNGULO Elisabeth González


70 CAP. V. OLIMPIADAS INTERNACIONALES

por tanto nos queda:

(a + b2 + c 2 )(p − a)(p − b)(p − c)


 2 
T =π , sustituyendo p por su valor, ver semiperímetro(I.12)
p3

−a + b + c a−b+c a+b−c
  ‹ ‹ ‹
(a + b + c )
2 2 2
2 2 2
= π
 
p 3

 
 (a2 + b2 + c 2 )(−a + b + c)(a − b + c)(a + b − c) 
= π )
a+b+c 3
  ‹ 
8
2

π(a2 + b2 + c 2 )(−a + b + c)(a − b + c)(a + b − c)


= .
(a + b + c)3
ƒ

9.4. Octava Olimpíada Internacional de Matemáticas, 1966 (Bulgaria)

La octava Olimpíada Internacional de Matemáticas se celebró del 3 al 13 Julio de 1966 en la ciudad de Sofía.
Los países participantes fueron: Bulgaria, Checoslovaquia, Alemania Oriental, Hungría, Mongolia, Polonia,
Rumania, la URSS y Yugoslavia.

Ejercicio. 9.4. (Polonia,[2, Enunciado, solución: pág 52,54, Ejercicio 8.6])


Sean:

ABC un triángulo.

M , K y L puntos interiores de los segmentos AB, BC y CA, respectivamente.

Demostrar que, entre los triángulos M AL, K BM y LC K, al menos el área de uno de ellos no es superior a la
cuarta parte del área del triángulo ABC.

22 de septiembre de 2014 Curso 20132014.


SEC. 9. PROBLEMAS 71

SOLUCIÓN. Sean a, b, c los lados del triángulo ABC y A


b, B
b, Cb sus respectivos ángulos.
Si denotamos:
A
AM = p1 c y BM = p2 c,
L
BK = m1 a y C K = m2 a,
c b
C L = n1 b y AL = n2 b. M

tenemos que:
B C
p1 , p2 , m1 , m2 , n1 , n2 son números reales positivos, a K

p1 + p2 = n1 + n2 = m1 + m2 = 1.

1. Por un lado aplicando la siguiente fórmula:

1
S= a b sen C,
b véase (I.16), (V.8)
2
tenemos:
1
n1 bm2 a sen Cb
SK LC 2
= = n1 m2 . (V.9)
SABC 1
ab
2
Análogamente:
SL M A
= p1 n2 ,
SABC
SK LC
= p2 m 1 .
SABC

Razonemos usando la reducción al absurdo, por lo que vamos a suponer que las áreas de los tres peque-
1
ños triángulos son mayores que area(ABC). Entonces multiplicando las tres áreas tenemos:
4
 ‹3
1 1
p1 p2 n1 n2 m1 m2 > = . (V.10)
4 64
p
2. Por otro lado aplicando la desigualdad de las medias aritmética y geométrica: a + b > 2 a b, tenemos:

1 = p1 + p2 > 2
p
p1 p2 (V.11)

1 = m1 + m2 > 2
p
m1 m2 (V.12)

1 = n1 + n2 > 2
p
n1 n2 (V.13)

TFM: OLIMPIADAS MATEMÁTICAS SOBRE EL TRIÁNGULO Elisabeth González


72 CAP. V. OLIMPIADAS INTERNACIONALES

Si multiplicamos estas tres desigualdades tenemos:

1>8
p p p
p1 p2 n1 n2 m1 m2 (V.14)

3
1> 8
p p p
p1 p2 n1 n2 m1 m2 . (V.15)

Entonces hemos llegado a una contradicción, por lo que al menos una de las tres áreas es mayor que
1
area(ABC).
4
ƒ

9.5. Duodécima Olimpíada Internacional de Matemáticas, 1970 (Hungría)

La segunda Olimpíada Internacional de Matemáticas se celebró del 8 al 22 Julio de 1970 en las ciudades de
Keszthely y Budapest.
Los países participantes fueron: Alemania, Austria, Bulgaria, Checoslovaquia, Francia, Alemania Oriental,
Hungría, Mongolia, Países Bajos, Polonia, Rumania, Suecia, Reino Unido, la URSS y Yugoslavia.

Ejercicio. 9.5. (Polonia,[2, Enunciado, solución: pág ,79,80 Ejercicio 12.1])


Sea ABC un triángulo y M un punto interior del lado AB.
Sean:

r1 , r2 , r los radios de las circunferencias inscritas2,1 a los triángulos AM C, BM C, ABC, respectivamente.

ρ1 , ρ2 , ρ los radios de las circunferencias exinscritas2,3 a los triángulos AM C, BM C, ABC, respectiva-


mente.

Probar la igualdad:
r1 r2 r
= . (V.16)
ρ1 ρ2 ρ

SOLUCIÓN.

Sean:

I el incentro(1.3) del triángulo ABC.


A
D el punto de tangencia de la circunferencia ins- Ec

crita en el triángulo ABC y el lado AB. M


D
E I
E el punto de tangencia de la circunferencia exins- C
crita en el triángulo ABC y el lado AB.
B

j
22 de septiembre de 2014 Curso 20132014.
SEC. 9. PROBLEMAS 73

Sabemos que:
AD = p − a, BD = p − b, (véase (2.1)), (V.17)

BE = p − a, AE = p − b, (véase (2.3)). (V.18)

i
Por tanto a partir de la tangente de los triángulos rectángulos I DA y I DB tenemos:

A r r
tan = = , aplicándo (IV.67),
2 AD p−a
B r r
tan = = , aplicándo (IV.67). A
2 BD p−b Ec

m
M
Entonces nos queda: D
E r I
C
A B
r = (p − a) tan = (p − b) tan . (V.19)
2 2 B

Análogamente a partir de la cotangente de los triángulos rectángulos Ec BE y EciAE tenemos:

A AE p−b
cotg = = , aplicándo (V.18),
2 ρ ρ
B BE p−a
cotg = = , aplicándo (V.18).
2 ρ ρ Ec
A

ρ M
Entonces nos queda: D
E I
A B C
ρ = (p − b) cotg = (p − a) cotg . (V.20)
2 2
B

Por lo tanto:
A A
(p − a) tan tan
r 2 = 2.
= (V.21)
ρ B B
(p − a) cotg tan
2 2

TFM: OLIMPIADAS MATEMÁTICAS SOBRE EL TRIÁNGULO Elisabeth González


74 CAP. V. OLIMPIADAS INTERNACIONALES

Por otro lado, aplicando todo este procedimiento a los triángulos CAM y C M B, tenemos:

r1 A ∠AM C
= tan tan ,
ρ1 2 2

r2 B ∠C M B
= tan tan .
ρ2 2 2

Además sabemos:
∠AM C = π − ∠C M B, (V.22)

por lo que:
∠AM C π ∠C M B
= − . (V.23)
2 2 2

Si aplicamos tangente en esta última igualdad tenemos:

π ∠C M B
 ‹  ‹  ‹
∠AM C ∠C M B
tan = tan − = cotg , aplicando la tangente del ángulo complementario1 .
2 2 2 2

Entonces tenemos:  ‹  ‹
∠AM C ∠C M B
tan = cotg . (V.24)
2 2

Por tanto:
r1 r2 A ∠AM C B ∠C M B
= tan tan tan tan
ρ1 ρ2 2 2 2 2

A ∠C M B B ∠C M B
= tan cotg tan tan , véase (V.24)
2 2 2 2
A B
= tan tan .
2 2

Por lo que queda demostrada la igualdad pedida. ƒ

9.6. Décima Olimpíada Internacional de Matemáticas, 1968 (URSS, Unión de Repúblicas So-
cialistas Soviéticas)

La décima Olimpíada Internacional de Matemáticas se celebró del 5 al 18 Julio de 1968 en las ciudades de
Moscú y Leningrado (San Petersburgo).
Los países participantes fueron: Bulgaria, Checoslovaquia, Alemania Oriental, Hungría, Italia, Mongolia, Po-
lonia, Rumania, Suecia, Reino Unido, la URSS y Yugoslavia.
π 
1
Tangente del ángulo complementario: tan − α = cotg α.
2

22 de septiembre de 2014 Curso 20132014.


SEC. 9. PROBLEMAS 75

Ejercicio. 9.6. (Rumania,[2, Enunciado, solución: pág ,63,64 Ejercicio 10.1])


Demuestra que solo existe un triángulo cuyos lados vienen dados por tres enteros positivos consecutivos y un
ángulo es dos veces otro ángulo.

SOLUCIÓN.

Sean:
A
b = 2A
B b.
α
BD la bisectriz interior del ángulo B
b.
c

Usando el Teorema de la bisectriz(4.7) tenemos:


b
D
AB AD
= (V.25) B
α
BC DC α

a
lo que es igual a:
C
c AD
= (V.26)
a DC

Expresando DC como b − AD y AD como b − DC, tenemos:


c AD
= ,
a b − AD
c b − DC
= .
a DC

Despejando AD y DC, respectivamente, tenemos:


cb
AD = ,
a+b
ab
DC = .
a+c

Además tenemos que el triángulo ABD es isósceles porque tiene dos ángulos iguales, por lo tanto tenemos que
AD = BD.
Por otro lado los triángulos ABC y BDC son semejantes por el siguiente razonamiento. Demostremoslo:

ABD Sabemos que A


ÕBD = Õ
B AD = α, por lo tanto:

B DA = 180o − 2α.
Õ (V.27)

TFM: OLIMPIADAS MATEMÁTICAS SOBRE EL TRIÁNGULO Elisabeth González


76 CAP. V. OLIMPIADAS INTERNACIONALES

BDC Sabemos que C


Õ BD = α y tenemos que BDC
Õ = 180o − (180o − 2α) = 2α por (V.27).

Ahora, si nos centramos en los dos triángulos ABC y BDC, tenemos:

b = α, B
ABC : A b = 2α.

DBC = α y BDC
BC D: Ö Õ = 2α.

Entonces aplicando el primer criterio de semejanza(5.1) , tenemos que los triángulos ABC y BDC son semejantes

Entonces tenemos que:


AB AC AB
= = . (V.28)
DB BC BD
Centrándonos en la primera igualdad y sustituyendo DC por su valor (véase (9.6)), tenemos:

a+c b
= . (V.29)
b a

Despejando obtenemos la relación clave para el ejercicio:

a(a + c) = b2 . (V.30)

Ahora consideramos tres casos:

A < B < C : Entonces b = a + 1 y c = a + 2. En este caso, sustituyendo en (V.30), tenemos que a = 1, por lo
que b = 2 y c = 3.
Entonces el triángulo no verifica la desigualdad triangular(4.8) .

A < C < B: Entonces c = a + 1 y b = a + 2. En este caso, sustituyendo en (V.30), tenemos que a = 4, por lo
que b = 6 y c = 5.

C < A < B: Entonces c = a − 1 y b = a + 1. En este caso, sustituyendo en (V.30), no tenemos soluciones


enteras.

Por tanto sólo existe un triángulo cuyos lados vienen dados por tres enteros positivos consecutivos y un ángulo
es dos veces otro ángulo y es el que cumple:

a=4
b=6
c=5
3
cos A = . Valor obtenido aplicando el Teorema del Seno(4.5) con los valores a,b y c anteriores.
4
ƒ

22 de septiembre de 2014 Curso 20132014.


SEC. 9. PROBLEMAS 77

9.7. Decimoséptima Olimpíada Internacional de Matemáticas, 1975 (Bulgaria)

La decimoséptima Olimpíada Internacional de Matemáticas se celebró del 3 al 16 Julio de 1975 en las ciudades
de Burgas y Sofía.
Los países participantes fueron: Austria, Bulgaria, Checoslovaquia, Francia, Alemania Oriental, Grecia, Hun-
gría, Mongolia, Países Bajos, Polonia, Rumania, Suecia, Reino Unido, Estados Unidos, la URSS, Vietnam y
Yugoslavia.

Ejercicio. 9.7. (Países Bajos,[2, Enunciado, solución: pág ,116,118 Ejercicio 17.3])
Sea ABC un triángulo. Los triángulos ABR, BC P y CAQ son dibujados externamente en los lados AB, BC y CA,
respectivamente, de forma que:

∠P BC = ∠CAQ = 45o ,

∠BC P = ∠QCA = 30o ,

∠RBA = ∠RAB = 15o .

Demuestra que ∠QRP = 90o y QR = RP.

SOLUCIÓN.
A 45◦
15◦

Q
Sea RS el segmento obtenido al rotar RB sobre R
S
un ángulo de 90o .
Como el triángulo ABR es isósceles porque tiene
G
dos ángulos iguales, tenemos que AR = BR y R

entonces RS = RA.
Por otro lado sabemos:

Ô=R
SRA b − 90o = (180o − 30o ) − 90o = 60o .

Entonces el triángulo RSA es equilátero por


15◦
tener dos lados iguales y el ángulo que los une B

tener valor 60o , por lo que tenemos:


45◦

AS = RA = AS. (V.31) 30◦

P 30◦
C

TFM: OLIMPIADAS MATEMÁTICAS SOBRE EL TRIÁNGULO Elisabeth González


78 CAP. V. OLIMPIADAS INTERNACIONALES

Por otro lado tenemos:


Ô =Õ
SAQ BAC. (V.32)

Para demostrar esta igualdad, si nos centraremos primero en el triángulo AGR, tenemos:

Ô = 180o − ARG
RGA Ô = 180o − 60o − 15o = 105o .
Ô − RAG

Fijándonos ahora en el triángulo AGS, tenemos:

Ô = 180o − RGA
AGS Ô = 75o y GAS
Ô = 180o − AGS Õ = 180o − 75o − 60o = 45o .
Ô − ASG

Ô =Õ
Entonces resulta GAS BAQ, y restándole a ambos ángulos el ángulo BAS
Ô nos queda la igualdad que buscá-
bamos.

Ahora vamos a usar el Teorema del Seno (4.5) en los triángulos AQC y ARB:

AQ AC
AQC : = , entonces:
sen 30o sen 105o

sen 30o
AQ = AC
sen 105o

sen (2 · 15o )
= AC
sen (90o + 15o )

2 sen 15o cos 15o


= AC , ver coseno de la suma y seno del ángulo doble2
cos 90o cos 15o + sen 90o sen 15o

2 sen 15o cos 15o


= AC
1 cos 15o + 0 sen 15o

= 2AC sen 15o .

AR AB
ARB: = , entonces:
sen 15o sen 150o

sen 15o
AR = AB
sen 150o

sen 15o
= AB
1
2

= 2AB sen 15o .

2
Coseno de la suma: cos(x + y) = cos x cos y − sen x sen y. Seno del ángulo doble: sen 2x = 2 sen x cos x.

22 de septiembre de 2014 Curso 20132014.


SEC. 9. PROBLEMAS 79

Teniendo en cuenta estas dos últimas igualdades tenemos que:


AQ AQ
= , por (V.31)
AS AR

2AC sen 15o


=
2AB sen 15o
AC
= ,
AB

lo que es igual que:


AQ AS
= . (V.33)
AC AB

Entonces con (V.32) y (V.33) tenemos que los triángulos CAB y QAS son semejantes(5.1) y, aplicando de nuevo
semejanza de triángulos, podemos decir que:
Õ = ASQ.
ABC Ô
AB BC
= de donde despejando y aplicando (V.31) tenemos:
AS SQ
SQ = 2BC sen 15o . (V.34)

Aplicando nuevamente el Teorema del Seno en el triángulo BP C, obtenemos:


BP = 2BC sen 15o (V.35)

Entonces tenemos que SQ = BP.

Sabemos:
R
Õ BP = ABC
Õ + 45o + 15o = ABC
Õ + 60o ,
Õ = ASQ
RSQ Ô + 60o , por ser el triángulo RSA equilátero.

Por (9.7) tenemos:


R
Õ BP = RSQ.
Õ (V.36)

Por lo que podemos concluir que la linea poligonal RSQ es obtenida por una rotación de 90o de centro R de
la linea poligonal RBP, entonces el segmento RQ es obtenido del segmento RP por la misma rotación. Esto
prueba:
PR = RQ
PRQ = 90o .
Õ

TFM: OLIMPIADAS MATEMÁTICAS SOBRE EL TRIÁNGULO Elisabeth González


Bibliografía

[1] N. E: Aguilera, Tópicos de geometría euclidiana plana, Olimpiada Matemática Argentina, 2012.

[2] M. Becheanu, International Mathematical Olympiads. 1959–2000, The Academic Distribution Center,
2001. 9.1., 9.2., 9.3., 9.4., 9.5., 9.6., 9.7.

[3] H. S. M. Coxeter, Introduction to geometry, John Wiley, 2nd. Ed., 1969.

[4] H. S. M. Coxeter y S. L. Greitzer, Geometry revisited, The Mathematical Association of America, 1967.

[5] A. Engel, Problem–solving strategies, Springer.

[6] Cristóbal Sánchez-Rubio and Manuel Ripollés Amela, Manual de matemáticas para preparación olímpica,
Universitat Jaume I. Castellón, 2000.

[7] Sessions de preparació per l’olimpiada matemàtica, Soc. Cat. Mat. Barcelona, 2000.
Refencias Web:

Sobre conceptos básicos del triángulo:

1. http://es.wikipedia.org/wiki/Tri%C3%A1ngulo

2. http://www.ditutor.com/geometria/perimetro_triangulo.html

3. http://www.geoka.net/triangulos/area_triangulo.html

4. http://www.vitutor.net/1/22.html

5. http://www.ditutor.com/geometria/area_triangulo.html

6. http://www.ditutor.com/geometria/triangulos_iguales.html

7. http://www.mathematicsdictionary.com/spanish/vmd/full/s/similartriangles.htm

8. http://jorge-fernandez.es/proyectos/angulo/temas/temad/index.html

9. http://www.vitutor.com/geo/eso/ss_3.html

10. http://jorge-fernandez.es/proyectos/angulo/temas/temad/index.html

11. http://www.disfrutalasmatematicas.com/definiciones/triangulo-acutangulo.html

Sobre los puntos notables de un triángulo:

12. http://www.vitutor.com/geo/eso/pl_5.html

13. http://gaussianos.com/inradio-y-semiperimetro/

14. http://es.wikipedia.org/wiki/Circunferencia_circunscrita

15. http://es.wikipedia.org/wiki/Circunferencia_inscrita

16. http://www.xtec.cat/~qcastell/ttw/ttwesp/definicions/d_exinscrites_c.html

17. Triángulos isósceles

Sobre relaciones métricas en el triángulo:

18. http://es.wikipedia.org/wiki/Teorema_de_Pit%C3%A1goras

19. http://siguiendoathales.blogspot.com.es/
84 BIBLIOGRAFÍA

20. http://es.wikipedia.org/wiki/Teorema_de_Tales

21. http://www.profesorenlinea.cl/geometria/Teorema_de_Tales.html

22. http://es.wikipedia.org/wiki/Teorema_del_seno

23. http://es.wikipedia.org/wiki/Teorema_del_coseno

24. http://es.wikipedia.org/wiki/Teorema_de_la_bisectriz

25. http://www.acm.ciens.ucv.ve/main/entrenamiento/guia-4.pdf

26. http://es.wikipedia.org/wiki/Teorema_de_Apolonio

27. https://www.youtube.com/watch?v=mKXU01rHWOs

Sobre trigonometria:

28. http://www.aritor.com/trigonometria/tangente.html

29. http://www.aritor.com/trigonometria/angulo_mitad.html

30. Tabla de fórmulas trigonométricas

31. http://www.vadenumeros.es/cuarto/relacion-razones-de-angulos.htm

Sobre los puntos notables de un triángulo:

32. http://www.jorge-fernandez.es/proyectos/angulo/temas/temab/

33. http://es.wikipedia.org/wiki/Arco_capaz

34. http://es.wikipedia.org/wiki/Puntos_coc%C3%ADclicos

35. http://perso.ya.com/jmreyes/lugaresgeometricos.html

36. http://www.matematicasvisuales.com/html/geometria/circunferencias/angcap.html

37. http://es.slideshare.net/iaespino/propiedades-angulares-de-la-circunferencia

38. http://www.matetam.com/glosario/definicion/angulos-alternos-internos

39. http://es.wikipedia.org/wiki/%C3%81ngulos_opuestos_por_el_v%C3%A9rtice

Sobre los ejercicios locales y nacionales de las olimpiadas matemáticas:

40. http://platea.pntic.mec.es/$\sim$csanchez/olimprab.htm

22 de septiembre de 2014 Curso 20132014.


Índice alfabético

E, 2 excentro, 2
G, 3 exincentro, 2
H, 2
I, 2 incentro, 2, 3
O, 1 inradio, 3
P, 6
lado
p, 6
de un triángulo, 1
S, 7
ángulo mediana, 3
exterior, 2 mediatriz, 1
inscrito, 6
interior, 1 ortocentro, 2
interno, 41, 50
semiinscrito, 21 perímetro de un triángulo, 6
ángulos potencia de un punto respecto de una circunferencia,
alternos internos, 36 20
opuestos por el vértice, 51 puntos cocíclicos, 21
suplementarios, 33
razón de semejanza, 19
área de un triángulo, 7
semiperímetro de un triángulo, 6
altura, 2
arco capaz, 6 triángulo, 1
triángulos
baricentro, 3
en posición de Thales, 20
bisectriz, 2
iguales, 18
exterior, 2
semejantes, 19
centros de un triángulo
baricentro, 3 vértice
circuncentro, 1, 4 de un triángulo, 1
excentro, 2
incentro, 2
ortocentro, 2
circuncentro, 1, 4
circunferencia
circunscrita, 4
inscrita, 3
circunferencias exinscritas, 5
circunradio, 4
.

PROBLEMAS SOBRE
CUADRILÁTEROS

ROCÍO LÓPEZ ANGUITA

Departamento de Álgebra

Universidad de Granada. 2014


Problemas de Olimpiadas
sobre Cuadriláteros

ROCÍO LÓPEZ ANGUITA

Dirigido por el Prof. Dr. D. Pascual Jara

Departamento de Álgebra
Universidad de Granada. 2014
Introducción

Como Trabajo Fin de Máster (TFM) en el Máster Interuniversitario en Matemáticas, se presenta


Problemas de Olimpiadas Matemáticas sobre Cuadriláteros.

Las primeras competiciones matemáticas nacionales fueron los concursos Eotvos en Hungría, que se
iniciaron en 1894, paralelamente al proceso iniciado por el Barón de Coubertin que desembocó en
las Olimpiadas de la época moderna (Atenas 1896). Pero no será hasta principio del S. XX cuando
este tipo de competiciones se extienda por el Centro y Este de Europa. La forma actual del concurso
es similar a la establecida en 1938 en las competiciones W. L. Putnam, organizadas en EEUU y
Canadá. El nombre de Olimpiadas data de 1958, año de celebración de las Olimpiadas Matemáticas
Internacionales por iniciativa de Rumania.

La elección de este modalidad de TFM tiene como objetivo, el análisis y autoevaluación personal
sobre las distintas materias cursadas en el mencionado Máster. A la vez, con este trabajo intento
estimular el estudio de las Matemáticas y el desarrollo de jóvenes talentos en esta ciencia. Filosofía
que comparto con la de las Olimpiadas Matemáticas, las cuales son algo más que un concurso,
orientas a promocionar las Matemáticas y dotarla de un contenido lúdico, ayudando así a aquellos
jóvenes para que dejen de enfrentarse a ellas como un obstáculo difícil de superar, y al mismo tiempo
descubrir a jóvenes talentos en esta materia.

¿Por qué sobre cuadriláteros?


El estudio de los cuadriláteros constituye un pilar fundamental para el aprendizaje de la geometría
plana y está muy presente en los problemas de Olimpiadas Matemáticas convocadas periódicamente.
Al mismo tiempo, están presente en nuestra vida cotidiana de diversas formas, por ejemplo a través
de distintos diseños arquitectónicos, dando origen al los poliedros (edificios), formando mosaicos,
además de algunos elementos naturales como los accidentes geográficos. En consecuencia, el estu-
dio de los cuadriláteros contribuye al desarrollo del pensamiento espacial y a la superación de los
fenómenos didácticos encontrados por los estudiantes que participan en este tipo de competiciones.

El presente trabajo va a estar dividido, fundamentalmente, en tres partes.


La primera de ellas se centra en repasar los conceptos fundamentales necesarios para la realización de
los problemas de Olimpiadas sobre cuadriláteros. En la segunda, se estudia los principales teoremas
necesarios para el desarrollo de las mismas, que sirven como base a esta ciencia. Y la tercera, está
dedicada al desarrollo de los propios Problemas de Olimpiadas, los cuales están agrupados según el
ii

grado de dificultad, en base a las distintas fases del concursos: local, nacional e internacional.

En última instancia, mencionar que por el grado de dificultad elegido en el desarrollo de este Trabajo
Fin de Máster, éste está orientado a alumnos de la Enseñanza Secundaria y Bachillerato.

22 de septiembre de 2014 Curso 20132014. TRABAJO FIN DE MÁSTER


Índice general

Introducción I

I Circunferencias y polígonos 1
1 Conceptos fundamentales . . . . . . . . . . . . . . . . . . . . . . . . . . . . . . . . . . . 1
2 Ángulos en la circunferencia . . . . . . . . . . . . . . . . . . . . . . . . . . . . . . . . . 5
3 Cuadriláteros . . . . . . . . . . . . . . . . . . . . . . . . . . . . . . . . . . . . . . . . . . . 8
4 Polígonos Regulares . . . . . . . . . . . . . . . . . . . . . . . . . . . . . . . . . . . . . . . 12

II Teoremas importantes y ejercicios 15


5 Teorema de Pitágoras . . . . . . . . . . . . . . . . . . . . . . . . . . . . . . . . . . . . . . 15
6 Teorema de Ptolomeo . . . . . . . . . . . . . . . . . . . . . . . . . . . . . . . . . . . . . . 20

III Problemas de Olimpiadas 25


7 Problemas de Olimpiadas: Fase Local . . . . . . . . . . . . . . . . . . . . . . . . . . . . 25
8 Problemas de Olimpiadas: Fase Nacional . . . . . . . . . . . . . . . . . . . . . . . . . . 35
9 Problemas de Olimpiadas: Fase Internacional . . . . . . . . . . . . . . . . . . . . . . . 44

Bibliografía 53

Bibliografía. Referencias Web 55


Capítulo I

Circunferencias y polígonos

Para comenzar a estudiar y desarrollar problemas de olimpiadas sobre cuadriláteros son imprescin-
dible las nociones básicas de geometría en general, y de triángulos, y circunferencias en particular,
que como tal, no formaría parte de este Trabajo de Fin de Máster, pero que me siento en la obligación
de recapitular, puesto que sin las mismas sería muy complicado resolver los mencionados problemas.

1. Conceptos fundamentales
Por su utilidad en los ejercicios, vamos a empezar recordando algunas definiciones básicas y funda-
mentales en geometría.

1.1. Triángulos:
Es una figura geométrica plana de tres lados y tres ángulos, los cuales siempre suman 1800 .
El triángulo está determinado por tres segmentos de recta (lados), o por tres puntos no alineados
llamados vértices.
Los triángulos lo podemos clasificar según los lados o los ángulos:

1. Clasificación de los triángulos según los lados:

a. Triángulo Equilátero: triángulo con los tres lados iguales, por lo que los ángulos también
son iguales y miden 600 .
b. Triángulo Isósceles: triángulo con dos lados iguales y uno desigual, como consecuencia
tiene dos ángulos iguales.
c. Triángulo Escaleno: triángulos con los tres lados y los tres ángulos distintos.
2 CAP. I. CIRCUNFERENCIAS Y POLÍGONOS

2. Clasificación de los triángulos según los ángulos:

a. Triángulo Acutángulo: triángulo con los tres ángulos agudos, menores de 900 .
b. Triángulo Obtusángulo: triángulo con un ángulo obtuso, mayor de 900 .
c. Triángulo Rectángulo: triángulo con un ángulo recto, igual a 900 .

Puntos Notables en el triángulo


Para poder saber cuales son los puntos notables en un triángulo, debemos tener presente las nocio-
nes de altura, bisectriz y mediatriz en esta figura geométrica.

Altura
La altura de un triángulo es cada una de las rectas perpendiculares, trazadas desde un vértice,
a la recta que contiene al lado opuesto (o su prolongación).

Mediana
Mediana es cada una de las rectas que une el punto medio de un lado con el vértice opuesto.

Bisectriz
La bisectriz de un triángulo es cada una de las rectas que divide a un ángulo en dos ángulos
iguales.

Mediatriz
La mediatriz de un triángulo es cada una de las rectas perpendiculares trazadas a un lado por
su punto medio.

Tras este breve repaso de concepto, ya podemos observar cuales son los puntos notables en un
triángulo, los cuales son:

Ortocentro
El ortocentro, H, es el punto de intersección de las tres alturas que tiene un triángulo.

Baricentro
El baricentro, B, es el punto de intersección de las tres medianas que posee un triángulo.

Circuncentro
El circuncentro, O, es el punto de intersección de las tres mediatrices que tiene un triángulo.
Dicho punto equidista de los tres vértices, por lo que es el centro de la circunferencia que pasa
por ellos.

22 de septiembre de 2014 Curso 20132014. TRABAJO FIN DE MÁSTER


SEC. 1. CONCEPTOS FUNDAMENTALES 3

Incentro
El incentro, I, es el punto de intersección de las tres bisectrices que se pueden trazar en un
triángulo. Dicho punto también es el centro de una circunferencia inscrita en el triángulo y
tangente a sus tres lados.

Por último recordad que el ortocentro, el baricentro y el circuncentro de un triángulo no equilátero


están alineados. Es decir, pertenecen a la misma recta, llamada recta de Euler.

Principio de Dirichlet o Principio del Palomar


El curioso nombre del Principio del Palomar viene derivado por el ejemplo trivial, generalmente usa-
do para presentarlo: Si hay un número de palomas mayor que el número de nidos, necesariamente hay
que colocar más de una paloma en algún nido.
Precisamente por la sencillez del enunciado es muy frecuente su aplicación en diversos planteamien-
tos y circunstancias.
De modo más general, podemos exponer el principio diciendo: si hay p palomas y n nidos (con
p > n), haciendo la división entera con resto de p entre n tendremos:

p = nk + r

siendo k el cociente y r el resto por defecto.


Podemos concluir que, si r es mayor que 0, entonces al menos en un nido el número de palomas es
mayor o igual que k + 1.

Desigualdades:
Desigualdad de Cauchy–Schwarz
‚ n
Œ2 ‚ n
Œ‚ n
Œ
X X X
∀a1 , a2 , a3 . . . , an , b1 , b2 , b3 , . . . , bn ∈ R : ai bi ≤ ai2 bi2 .
i=1 i=1 i=1

Se verifica la desigualdad si y sólo si existe λ tal que ai = λbi para todos i = 1, 2, . . . , n.

DEMOSTRACIÓN.
n
X n
X n
X n
X
∀λ ∈ R : (ai − λbi ) ≥ 0 ⇐⇒ λ
2 2 2
bi − 2λ ai bi + ai2 ≥ 0,
i=1 i=1 i=1 i=1

por tanto el discriminante ha de ser menor o igual a cero:


‚ n
Œ2 ‚ n
Œ‚ n
Œ
X X X
ai bi ≤ ai2 bi2 .
i=1 i=1 i=1

TFM: Problemas de Olimpiadas sobre Cuadriláteros Rocío López Anguita


4 CAP. I. CIRCUNFERENCIAS Y POLÍGONOS

La igualdad vale si y sólo si:

n
X
(ai − λbi )2 = 0 ⇐⇒ ai − λbi = 0, para todos i = 1, 2, . . . , n.
i=1

Extrayendo la raíz cuadrada de ambos miembros la desigualdad se puede escribir:


v v
n
X uXn uXn
2t
ai bi ≤ bi2 .
t
ai
i=1 i=1 i=1

Desigualdad Triangular

∀a1 , a2 . . . an ∈ R : |a1 + a2 + . . . an | ≤ |a1 | + |a2 | + . . . + |an |.

Se llama triangular porque en el caso n = 2, y si a1 , a2 representan las longitudes de dos lados de un


triángulo, la desigualdad anterior expresa la conocida propiedad de que un lado no puede exceder
a la suma de los otros dos.

DEMOSTRACIÓN.
La demostración es muy sencilla para n = 2, pues

−|a1 | ≤ a1 ≤ |a1 |

−|a2 | ≤ a2 ≤ |a2 |

Sumando

⇒ −(|a1 | + |a2 |) ≤ a1 + a2 ≤ a1 + a2 ≤ |a1 | + |a2 | ⇒ |a1 + a2 | ≤ |a1 | + |a2 |,

y después se extiende por inducción sobre n.


ƒ

En conclusión, cada lado es menor que la suma de los otros dos y mayor que su diferencia. Lo
podemos expresar:
|b − c| < a < b + c

|a − c| < b < a + c

|a − b| < c < a + b

22 de septiembre de 2014 Curso 20132014. TRABAJO FIN DE MÁSTER


SEC. 2. ÁNGULOS EN LA CIRCUNFERENCIA 5

2. Ángulos en la circunferencia
Podemos dibujar ángulos que se relacionen con la circunferencia. Dependiendo de la posición que
ocupen los mismos recibirán nombres acorde con su relación a esa posición. Cuando nos referimos a
los ángulos en la circunferencia siempre relacionamos a éstos con los arcos que forman. Así, podemos
diferenciar entre:
Ángulo central
Es el ángulo que tiene su vértice en el centro de la circunferencia y sus lados son el radio de ella.
Si el radio de la circunferencia es la unidad, la medida del arco corresponde con el valor del ángulo
expresada en radianes. Es decir
α = longitudAB.
A

Ángulo inscrito
Es el que tiene su vértice en la circunferencia y sus lados son dos rectas secantes.
Su valor es la mitad del central correspondiente
α
β= .
2
A

C B

Ángulo semiiscrito
Es el que tiene su vértice en la circunferencia, un lado secante y otro tangente.
Su valor es la mitad del central correspondiente
β = 2α.

TFM: Problemas de Olimpiadas sobre Cuadriláteros Rocío López Anguita


6 CAP. I. CIRCUNFERENCIAS Y POLÍGONOS

A
C
α

Ángulo exterior
Es el que tiene el vértice fuera de la circunferencia y los lados de su ángulo son secantes a ella.
Su valor es la diferencia de los ángulos interiores, o equivalentemente, semidiferencia de los dos
arcos centrales, ver dibujo. Este resultado sigue siendo válido si los lados del ángulo son tangentes
a la circunferencia.

α = β − γ.

Ángulo interior
Es el que tiene el vértice en el interior de la circunferencia, y sus lados son secantes. Su valor es la
semisuma de los arcos centrales.
α = β + γ.

22 de septiembre de 2014 Curso 20132014. TRABAJO FIN DE MÁSTER


SEC. 2. ÁNGULOS EN LA CIRCUNFERENCIA 7

D E

β α

γ
B

Arco capaz de un ángulo dado sobre un segmento

El lugar geométrico de los puntos, desde los cuales se ve un segmento dado (una cuerda) bajo ángulo
constante, es un arco de circunferencia, llamado arco capaz del ángulo dado sobre el segmento
constituido por la cuerda.

Este resultado es muy importante, tanto por su aplicación a numerosos problemas, como por su uso
para otros teoremas. Por ello, vamos a desarrollar los pasos para su construcción geométrica con
regla y compás.
El problema está definido como sigue a continuación.

Dados un ángulo α y un segmento AB, construir el arco capaz de α sobre AB.

Para la construcción del arco capaz de α sobre AB seguimos los siguientes pasos:

1. Se traza la mediatriz M N de AB.


2. Se traza una recta r que pasa por A y forma ángulo α con AB.
3. Se construye el punto M intersección de la mediatriz y la recta r.
4. Se traza la perpendicular por A a r. La intersección de esta recta con la mediatriz determina
un punto O.
5. Los puntos del arco de la circunferencia con centro O y que pasa por A y B, en el interior del
semiplano que no contiene a M es el lugar geométrico buscado.

TFM: Problemas de Olimpiadas sobre Cuadriláteros Rocío López Anguita


8 CAP. I. CIRCUNFERENCIAS Y POLÍGONOS

A B

Puede comprobarse que el arco en el otro semiplano corresponde al arco capaz del ángulo α0 =
1800 − α, suplementario de α sobre el mismo segmento AB dado. La verificación de que el arco
construido es el solicitado, es evidente observando la figura y por la igualdad de los dos ángulos
marcados con α en ella.

3. Cuadriláteros

Los cuadriláteros son polígonos que tiene cuatro lados.


Estas figuras geométricas tienen distintas formas, pero todas tienen: cuatro lados, cuatro vértices,
cuatro ángulos interiores y dos diagonales. Además, la suma de los ángulos interiores de un cuadri-
látero es 3600 . Básicamente, existen dos tipos de cuadriláteros: convexos y cóncavos.

Una diagonal de un cuadrilátero es un segmento cuyos extremos son dos vértices no consecutivos.

CUADRILÁTEROS CONVEXOS
Son aquellos tales que, si se toman dos puntos interiores A y B cualesquiera del mismo, todos los
puntos del segmento AB que determinan están dentro del cuadrilátero. En particular todas las dia-
gonales son interiores.

22 de septiembre de 2014 Curso 20132014. TRABAJO FIN DE MÁSTER


SEC. 3. CUADRILÁTEROS 9

CUADRILÁTEROS CÓNCAVOS O NO CONVEXOS


Son aquellos cuadriláteros en los que se pueden encontrar dos puntos interiores A y B del mismo,
tales que algunos de los puntos del segmento AB que determinan están fuera del cuadrilátero. O
equivalentemente, una de las dos diagonales es exterior.

A
B

Cuadrilátero Cíclico

Un hecho expuesto en geometría es el que podemos presentar indicando que por cualquiera tres
puntos no alineados pasa exactamente una circunferencia.
¿Pero qué podemos decir si en vez de considerar tres puntos, consideramos cuatro? Como es de es-
perar, no siempre va a existir una circunferencia que pase por los cuatro puntos dados.
Por ejemplo, consideramos una circunferencia que pase por tres puntos dados (la cuál es única)
A, B, C y consideramos un punto D que no esté sobre circunferencia. Claramente, no existe una cir-
cunferencia que pase por estos cuatro puntos.

TFM: Problemas de Olimpiadas sobre Cuadriláteros Rocío López Anguita


10 CAP. I. CIRCUNFERENCIAS Y POLÍGONOS

B D

A C

Por lo tanto, llegamos a la conclusión que un cuadrilátero que posean una circunferencia que pase por
sus vértices deben ser en cierta forma especiales. A dichos cuadriláteros se le llaman cuadriláteros
cíclicos.

Definición. 3.1.
Un cuadrilátero que está inscrito en una circunferencia, es decir que sus vértices están sobre una
circunferencia se dice que es un cuadrilátero cíclico ó inscriptible.

Proposición. 3.2.
Un cuadrilátero es inscriptible ⇐⇒ cada dos ángulos opuestos son suplementario.

C
B

A1

DEMOSTRACIÓN.

DAB =
Para probar esto, primero vamos a suponer que el cuadrilátero ABC D es cíclico. Tenemos que Õ
2 y BC D = 2 y como Bd + D B = 360 (midiendo los ángulos en grados). Entonces tenemos que:
BD Ö DB Ó Ó 0

DAB + Ö
Õ BC D = α + β = 1800

22 de septiembre de 2014 Curso 20132014. TRABAJO FIN DE MÁSTER


SEC. 3. CUADRILÁTEROS 11

A
α B

D β
C

Ahora supongamos que Õ DAB + ÖBC D = α + β = 1800 . Trazamos la circunferencia que pasa por
los vértices D, A, B y supongamos que está no pasa por el vértice C. Prolonguemos DC hasta que
intersecte a la circunferencia en C 0 .
Como el cuadrilátero ABC 0 D es cíclico tenemos que Õ DAB + × BC 0 D = α + β = 1800 , esto quiere decir
que × BC 0 D = Ö
BC D = β y entonces DC sería paralelo a DC 0 , lo cuál es una contradicción por que
las líneas paralelas no se intersecan. Entonces C y C’ coinciden, por lo que ABC D es un cuadrilátero
cíclico.

A D
α

β β0
B C C0

Definición. 3.3.
Un cuadrilátero se llama circunscriptible si existe una circunferencia tangente a sus cuatro lados.

Proposición. 3.4.
Un cuadrilátero es circunscriptible ⇐⇒ los pares de lados opuestos suman igual.

TFM: Problemas de Olimpiadas sobre Cuadriláteros Rocío López Anguita


12 CAP. I. CIRCUNFERENCIAS Y POLÍGONOS

C E D

H O F

A B
G

Problema. 3.5.
Las circunferencias C1 y C2 se intersecan en los puntos A y B. Por el punto A se traza una recta que
corta a la circunferencia C1 y C2 en los puntos C y D, respectivamente. Por los puntos C y D se trazan
tangentes a las circunferencias, las cuales se intersecan en el punto M . Demuestra que el cuadrilátero
M C BD es cíclico.

SOLUCIÓN.
Queremos probar que C× MD + ÖDBC = 1800 . Tracemos la cuerda común AB. Tenemos que M Ö CA =
C BA = α ya que uno es ángulo semiinscrito y el otro es ángulo inscrito, ambos en la circunferencia
Õ
C1 .
Análogamente se demuestra que M ÖDA = Õ D BA = β en C2 .
Tenemos que α + β + θ = 180 , por lo que los ángulos internos del triángulo AM C D, pero como
0

C
Õ BD = α + β tenemos que C×MD +Ö DBC = 1800 . Por lo tanto el cuadrilátero M C BD es cíclico.
ƒ

4. Polígonos Regulares
Un polígono regular, de n lados (n-ángulo), es el que tiene todos sus lados iguales, y en consecuencia
todos sus ángulos también son iguales. Todo polígono regular se puede inscribir en una circunferen-
cia y tiene una circunferencia inscrita. Los centros de estas dos circunferencias coinciden; este punto
se llama el centro del polígono.

Se llama radio del polígono (y lo denotaremos por r), al de su circunferencia circunscrita, es


decir, el segmento que une el centro del polígono con cualquier vértice.

22 de septiembre de 2014 Curso 20132014. TRABAJO FIN DE MÁSTER


SEC. 4. POLÍGONOS REGULARES 13

Se llama apotema del polígono (y la denotaremos a), al segmento que une el centro de la
circunferencia circunscrita, con el punto medio de cualquier lado.

Por lo tanto, la construcción de un polígono regular de n lados, equivale a la división de una cir-
cunferencia en n partes iguales. Supuesto que se ha construido por bisección del arco, a través de la
cuerda, es inmediato duplicar el número de lados, por lo que se pueden construir polígonos regula-
res de n · 2m lados.
Existen métodos para algunos valores como 3, 4 ó 5, pero no es posible inscribir con regla y compás
polígonos de 7 ó 9 lados. Aunque sí es posible para 17 lados (célebre problema resuelto por Gauss).
Gauss demostró que una circunferencia se puede dividir, con regla y compás, en n partes iguales si,
y sólo si, n admite una descomposición en factores primos de la forma:
a b c
n = 2 p (22 − 1)(22 − 1) . . . (22 − 1)

con a, b, c, . . . distintos.
Por su incidencia en muchos problemas, vamos a estudiar los polígonos regulares de 3, 4, 5, 6 y 10
lados y algunos elementos.

Relaciones métricas en cualquier polígono regular


Sea r el radio, a la apotema y l el lado, para construir polígonos regulares. Considerando el triángulo
isósceles con base l y lados r, la apotema a es la altura. Tenemos dos triángulos rectángulos de
hipotenusa r y catetos a y l/2. Por el teorema de Pitágoras (que veremos más adelante) tenemos:

l2
r2 = + a2 ,
4
relación que permite, conociendo dos de estos elementos, hallar el tercero.

El ángulo central α es el formado por dos radios consecutivos, para un n-ángulo los valores son:

3600 (n − 2)1800
α= β=
n n
Los ángulos entre lados con vértices comunes son inscritos en arcos múltiplos de α. Se tiene así una
relación que permite calcular l y a en función de r. En efecto, se tiene
α r 2r
sen = , esto es, l = , (I.1)
2 l/2 sen α2
α r r
cos = , esto es, a = . (I.2)
2 a cos α2
Si consideramos el polígono regular circunscrito a la circunferencia que pasa por los vértices de un
polígono regular dado, sus lados serán tangentes a esta circunferencia; luego la apotema del polí-
gono circunscrito coincide con el radio del inscrito.

TFM: Problemas de Olimpiadas sobre Cuadriláteros Rocío López Anguita


14 CAP. I. CIRCUNFERENCIAS Y POLÍGONOS

Como dos polígonos regulares del mismo número de lados son semejantes, la razón de semejanza
entre ambos polígonos es la misma que hay entre el radio y la apotema del polígono.
Todas las medidas lineales, como lado o diagonales del polígono circunscrito, se hallan multiplicando
la medida correspondiente del inscrito por la razón de semejanza k = r/a.
En particular, el área del polígono inscrito es: A = P·a
2 , siendo A el área, P el perímetro y a la apotema;
entonces el área del polígono circunscrito es: A = A · k2 .
0

En virtud de las relaciones expuestas, basta conocer, en función del radio r, el lado del n-ágono regu-
lar inscrito para poder deducir los demás elementos: apotema, perímetro y área, tanto del polígono
inscrito como del circunscrito.
Sólo nos queda obtener el valor del lado del n-ágono regular inscrito en una circunferencia de radio
r, ver la ecuación (I.1), y dar el modo de construcción con regla y compás para n = 3, 4, 5, 6, 10.

NÚMEROS DE LADOS VALOR DEL LADO


p
3 l = 3r
p
4 l = 2r
p p
10−2 5
5 l= 2 r
6 l=r
p
5−1
10 l= 2 r

22 de septiembre de 2014 Curso 20132014. TRABAJO FIN DE MÁSTER


Capítulo II

Teoremas importantes y ejercicios

Una vez introducido los conceptos fundamentales de la geometría que necesitamos para comprender
el estudio de esta materia, pasamos a desarrollar algunos teoremas que serán de gran utilidad para
resolver los Problemas de Olimpiadas sobre Cuadriláteros.

5. Teorema de Pitágoras
El teorema de Pitágoras nos indica que la suma de los cuadrados de los catetos de un triángulo
rectángulo es igual al cuadrado de la hipotenusa.

h2 = c 2 + C 2 .
D

h
c

α
B
A
C

A lo largo de la historia han sido muchas las demostraciones y pruebas que matemáticos y amantes
de estas ciencias han dado sobre este teorema.
La demostración más simple es la atribuida a Lagrange, la cuál vamos a ver a continuación:

DEMOSTRACIÓN.
Trazando la perpendicular C D a AB, obtenemos así tres triángulos semejantes, como observamos en
la figura.
16 CAP. II. TEOREMAS IMPORTANTES Y EJERCICIOS

b
a
h

A B
D c

Considerando y, el segmento AD, obtenemos como resultado que el segmento DB es c − y. Por lo


tanto, tenemos las siguientes ecuaciones:

y b
= =⇒ y c = b2
b c

c− y a
= =⇒ c(c − y) = a2
a c

Sumando las dos igualdades resulta que:

a2 + b2 = c 2

Ejercicio. 5.1. (Ref. Web (5))


Sea ABC D un paralelogramo y supongamos que AC es su diagonal mayor. Desde C se trazan
perpendiculares a las rectas AB y AD, con pies en E y F respectivamente. Demostrar que:

AB · AE + AD · AF = AC 2

SOLUCIÓN.
Para resolver este ejercicio aplicamos el teorema de Pitágoras a los triángulos rectángulos AC E y BC E.

22 de septiembre de 2014 Curso 20132014. TRABAJO FIN DE MÁSTER


SEC. 5. TEOREMA DE PITÁGORAS 17

B
E

F
C

Usando también que BC = AD al ser un paralelogramo, obtenemos que:

AC 2 = AE 2 + EC 2 = AE 2 + BC 2 − BE 2 = (AE + BE)(AE − BE) + AD2 = AB · AE + AB · BE + AD2

Ahora bien, vemos que los triángulos BC E y C DF son semejante, ya que tienen dos ángulos iguales.
Entonces tenemos que BCBE
= CDFD , y teniendo en cuenta que C D = AB, deducimos que BE ·AB = AD· DF
Sustituyendo esto último en la igualdad anterior obtenida para AC 2 , queda demostrada que:

AB · AE + AD · AF = AC 2

Ejercicio. 5.2. (Ref. Web (5))


En una circunferencia de centro O y radio 1, tomemos una cuerda de longitud a y la circunferencia
C que tiene a dicha cuerda por diámetro. Si P es el punto de C más alejada de O, ¿cuál es el valor
máximo de la distancia entre O y P cuando variamos a?

SOLUCIÓN.
Para realizar este ejercicios, en primer lugar vamos a calcular la longitud de OP en términos de a,
en segundo, calcularemos su valor máximo. Denotaremos M al punto de corte de la cuerda con el
segmento OP, y A a uno de los puntos de corte de la circunferencia.
Por el teorema de Pitágoras tenemos que OM 2 = OA2 − AM 2 = 1 − ( 2a )2 , y por otro lado, como M P
es un radio de C, tendremos que M P = 2a .

TFM: Problemas de Olimpiadas sobre Cuadriláteros Rocío López Anguita


18 CAP. II. TEOREMAS IMPORTANTES Y EJERCICIOS

Siendo OP = OM + M P y usando la desigualdad entre las medias aritméticas y cuadrática1 , obtene-


mos que: v
u a2 2
t 4 + 1 − a4
v
a t a2 p
OP = OM + M P − + 1 − ≤2 = 2
2 4 2
q 2 p
Como dicha cota se alcanza cuando 2a = 1 − a4 , es decir, para a = 2, el valor máximo de OP es
p p
2 y se alcanza ⇐⇒ a = 2.
ƒ

Ejercicio. 5.3. (Ref. Web (5))


Calcular la razón entre la diagonal y el lado de un pentágono regular.

SOLUCIÓN.
Para resolver este ejercicio vamos a intentar buscar triángulos semejantes, una vez que tracemos las
diagonales en el pentágono.
A1

A2 P A5

A3 A4

Como podemos observar en la figura, consideramos A1 , A2 , A3 , A4 , A5 a los vértices, l al lado y d a la


diagonal del pentágono. Además, sea P el punto de corte de las diagonales A1 A3 y A2 A5 .
Sabemos que cada diagonal es paralela a un lado del pentágono, por lo que el cuadrilátero PA3 A4 A5
es un paralelogramo y, por tanto, PA5 = l y PA2 = d − l.
Podemos ver que los triángulos A2 PA3 y A3 A4 A1 son semejantes, al tener los lados paralelos, por lo
que se verifica que
A2 P A2 A3
= .
A3 A4 A1 A3
Al escribir la igualdad en función de d y l tenemos: d−ll = d.
l

Por lo que deducimos que ( dl )2 − dl − 1 = 0 y a resolver, la ecuación de segundo grado, obtenemos


1
La raíz cuadrada de la media de los cuadrados.

22 de septiembre de 2014 Curso 20132014. TRABAJO FIN DE MÁSTER


SEC. 5. TEOREMA DE PITÁGORAS 19

que: p
d 1+ 5
=
l 2
es decir, la razón entre la diagonal de un pentágono regular y su lado es la razón áurea.
ƒ
Una vez visto el teorema de Pitágoras, es fácil probar el siguiente teorema conocido como la ley del
paralelogramo.

Teorema. 5.4. (Ley del Paralelogramo)


La suma de los cuadrados de las diagonales de un paralelogramo es igual a la suma de los cuadrados
de los lados

DEMOSTRACIÓN.
Sea ABC D el paralelogramo y sean AB = C D = a y BC = DA = b. También sea AC = c y BD = d.
A D

d
a c
h

B M b C k N

Tracemos perpendiculares a BC desde A y D, las cuales intersectan a BC en M y N . Sean AM =


DN = h y BM = C N = k.
Aplicando el teorema de Pitágoras a los triángulos DC N , DBN y AM C tenemos las siguientes igual-
dades:
h2 + (b + k)2 = d 2
h2 + (b − k)2 = c 2
h2 + k 2 = a 2
Sumando las dos igualdades primeras obtenemos:
2h2 + 2b2 + 2k2 = d 2 + c 2
Ahora utilizando la última igualdad, de las tres obtenidas anteriormente, tenemos que:
2a2 + 2b2 = d 2 + c 2
ƒ
A continuación vamos a mencionar algunas generalizaciones del teorema de Pitágoras que se utilizan
con frecuencia en los ejercicios.
En primer lugar tenemos el teorema del coseno que es un generalización del teorema de Pitágoras

TFM: Problemas de Olimpiadas sobre Cuadriláteros Rocío López Anguita


20 CAP. II. TEOREMAS IMPORTANTES Y EJERCICIOS

5.1. Teorema del Coseno


Dado un triángulo ABC, siendo α, β, γ, los ángulos, y a, b, c, los lados respectivamente opuestos a
estos ángulos,
c 2 = a2 + b2 − 2ab cos γ
O lo que es lo mismo, en un triángulo el cuadrado de cada lado es igual a la suma de los cuadrados
de los otros dos menos el doble del producto de ambos por el coseno del ángulo que forman.
C

b a

α β
A B
c

5.2. Teorema del Seno


Los lados de un triángulos es directamente proporcional al seno de los ángulos opuestos:
a b c
= =
senα senβ senγ

6. Teorema de Ptolomeo
El teorema de Ptolomeo nos indica que un cuadrilátero es convexo e inscriptible si y solo si el pro-
ducto de sus diagonales es igual a la suma de los productos de los lados opuestos.
O lo que es lo mismo:

Proposición. 6.1.
Un cuadrilátero ABC D es cíclico si y solo si AB · C D + AD · BC = AC · BD.

DEMOSTRACIÓN.

Empezaremos demostrando que si el cuadrilátero es cíclico, se cumple la expresión dada en el


enunciado.
Consideramos un punto P sobre la diagonal AC, de tal manera que P ÕBC = A ÕBD = α.

22 de septiembre de 2014 Curso 20132014. TRABAJO FIN DE MÁSTER


SEC. 6. TEOREMA DE PTOLOMEO 21

C
D

Dado que ABC D es cíclico, también tenemos que PÕ CB = A


Õ DB = β. De aquí deducimos que los
triángulos P BC y ABC son semejantes, por lo que:
BC · AD
PC =
BD
Como también los triángulos BAP y BDC son semejantes, tenemos que:
AB · C D
AP =
BD
Sumando las dos expresiones obtenidas anteriormente tenemos:
AB · C D BC · AD
AP + P C = AC = +
BD BD
por tanto,
AC · BD = AB · C D + BC · AD
Demostraremos la otra implicación, es decir, si cumple AC · BD = AB · C D + BC · AD es un cua-
drilátero cíclico. Consideramos el cuadrilátero de la figura de vértices ABC D, lados m, n, p, q y
diagonales d, d 0 .

n
m

d0
H
C
d
A

p
q

TFM: Problemas de Olimpiadas sobre Cuadriláteros Rocío López Anguita


22 CAP. II. TEOREMAS IMPORTANTES Y EJERCICIOS

Construimos un triángulo AH B semejante al DC B, de forma que el lado q, se corresponda con


d 0 , y los ángulos con vértice B marcados en la figura, sean iguales.
Por la semejanza se cumple:
BH q AH
= 0= ⇐⇒ q · n = AH · d 0
p d n
La primera de las igualdades de la proporción junto a la igualdad de los ángulos marcados
establece que los triángulos H BC y ABD también sean semejantes (un ángulo igual y propor-
cionales los lados que lo forman), por tanto:
HC p
= 0 ⇐⇒ p · m = H C · d 0
m d
Y sumando las dos igualdades nos queda:

q · n + p · m = AH · d 0 + H C · d 0 ≥ AC · d 0 = d · d 0

Desigualdad válida para cualquier cuadrilátero.


El teorema estará demostrado cuando veamos la condición para que se de la igualdad:

q · n + p · m = d · d0

⇐⇒ H está alineado con A y C ⇐⇒

BAC = B
⇐⇒ Õ Õ AH = BDC
Õ ⇐⇒

⇐⇒ ABC D es inscriptible.

Es decir, es un cuadrilátero cíclico,ya que A y D están en el arco capaz de un mismo ángulo


sobre el segmento BC.

Problema. 6.2. (Ref. Web (6))


Dado un heptágono ABC DE F G de lado 1, demuestra que las diagonales AC y AD verifican:

1 1
+ =1
AC AD

SOLUCIÓN.
Sea x la longitud de los segmento AC y BD e y, las de los segmentos AD y AE.

22 de septiembre de 2014 Curso 20132014. TRABAJO FIN DE MÁSTER


SEC. 6. TEOREMA DE PTOLOMEO 23

F
B

G C

E D

Si consideramos el cuadrilátero AC DE por el teorema de Ptolomeo tenemos que x y = y + x. Si dicha


y
ecuación la dividimos por x obtenemos: y = x + 1 y si ahora los dividimos por y, el resultado es:
1 = 1x + 1y , o lo que es lo mismo
1 1
+ =1
AC AD
ƒ

Ejercicio. 6.3. (Ref. Web (5))


Sea ABC DE F un hexágono convexo tal que AB = BC = C D, DE = E F = AF y Ö BC D = E
Ô FA = π3 .
Sean P y Q dos puntos interiores al hexágono de forma que los ángulos A
Ô Ó valen ambos 2·π .
P B y QE 3
Demostrar que:
AP + P B + PQ + DQ + QE ¾ C F

SOLUCIÓN.
Sabemos que la longitud de una poligonal es siempre mayor o igual que la distancia que une sus
extremos por la desigualdad triangular. Vamos a intentar relacionar AP + P B + PQ + DQ + QE con la
longitud de una poligonal.
Consideramos M y N puntos exteriores del hexágono tales que, ABM y DEN sean triángulos equi-
láteros. En consecuencia, la propiedad de arco capaz nos asegura que P está en la circunferencia
circunscrita al triángulo ABM .
Si aplicamos el teorema de Ptolomeo al cuadrilátero AM BP, AP + P B = M P. De igual modo, obte-
nemos que DQ + QE = NQ y se cumple que:
AP + P B + PQ + DQ + QE = M P + PQ + QN ≥ M N

TFM: Problemas de Olimpiadas sobre Cuadriláteros Rocío López Anguita


24 CAP. II. TEOREMAS IMPORTANTES Y EJERCICIOS

ya que M PQN es una poligonal que une M y N , su longitud siempre es mayor que la del segmento
MN.
Ahora bien, el octógono AM BC DN E F es simétrico respecto de la recta BE, por las condiciones del
enunciado. Luego se tiene que M N = C F ,por lo tanto demostramos que:

AP + P B + PQ + DQ + QE ¾ C F

22 de septiembre de 2014 Curso 20132014. TRABAJO FIN DE MÁSTER


Capítulo III

Problemas de Olimpiadas

7. Problemas de Olimpiadas: Fase Local

Una vez recordada la teoría necesaria para la realización de Problemas de Olimpiadas relacionados
con nuestra temática, los cuadriláteros. Comenzamos con los problemas propuestos en la fase local,
los mismos conllevan un menor grado de dificultad, y han sido extraídos de la página oficial de las
Olimpiadas Matemáticas Española.

Ejercicio. 7.1. (Ref. Web (7), 2008)


En un cuadrilátero convexo se trazan las perpendiculares desde cada vértice a la diagonal que no
pasa por él. Demuestra que los cuatro puntos de intersección de cada perpendicular con su corres-
pondiente diagonal forman un cuadrilátero semejante al dado.

SOLUCIÓN.
En primer lugar, veamos como es el dibujo para comprender mejor lo que nos están pidiendo:
Si observamos la figura, nos damos cuenta que lo que tenemos que demostrar es que ambos cuadri-
láteros tienen todos los ángulos iguales.
26 CAP. III. PROBLEMAS DE OLIMPIADAS

B
D0
C0

A0

B0
D

Consideramos el cuadrilátero ADA0 D0 . Sabemos por construcción, que dicho cuadrilátero es circuns-
criptible y de ahí que B 0 D 0 A0 = A
Ø Õ DA0 , al ser ambos suplementarios del mismo ángulo A ×D0 A0 .
0 0
Por otro lado, si consideramos el cuadrilátero DC C D que es circunscriptible, vemos que los ángulos
C
Ù 0 D0 B 0 y A
Ö 0 DC son iguales por estar inscrito en el mismo arco.

Ahora si sumamos los ángulos, obtenemos ADC Õ=Ù C 0 D0 A0 . De igual forma, demostramos que se dan
las siguientes igualdades:
BC D = D
Ö Ù Õ=C
0 C 0 B 0 , ABC Ø B AD = B
0 B 0 A0 , Õ Ø 0 A0 D 0 .

Luego, vemos que efectivamente los dos cuadriláteros tienen los mismo ángulos. Por lo que son se-
mejantes.
ƒ

Ejercicio. 7.2. (Ref. Web (7), 2008)


Las longitudes de los lados y de las diagonales de un cuadrilátero convexo plano ABC D son racio-
nales. Si las diagonales AC y BD se cortan en el punto O, demuestra que la longitud OA es también
racional.

SOLUCIÓN.
Consideramos la siguiente figura:

22 de septiembre de 2014 Curso 20132014. TRABAJO FIN DE MÁSTER


SEC. 7. PROBLEMAS DE OLIMPIADAS: FASE LOCAL 27

B α O D
γβ

donde α = AÕ BD, β = Õ C BA, γ = C


Õ BD.
Para resolver este ejercicio vamos a utilizar el teorema del coseno y del seno.
En primer lugar, aplicamos el teorema del coseno en el triángulo ABC:

AC 2 = BC 2 + BA2 − 2BC · BAcos β

despejando, tenemos:
BC 2 + BA2 − AC 2
cos β =
2BC · BA
(que por hipótesis) es un número racional.
De igual modo obtenemos el cos α y el cos γ.
Para obtener el coseno(α) consideramos el triángulo ABD

AD2 = BA2 + BD2 − 2BA · BD cos α

despejando, tenemos:
BA2 + BD2 − AD2
cos α =
2BA · BD
(que por hipótesis) es un número racional.
Para obtener el coseno(γ) consideramos el triángulo ABD

C D2 = BC 2 + BD2 − 2BC · BD cos γ

despejando, tenemos:
BC 2 + BD2 − C D2
cos γ =
2BC · BD
(que por hipótesis) es un número racional.
Por otra parte, cos β = cos(α + γ) = cos α cos γ − senαsenγ, por lo que senαsenγ es un número
racional.

TFM: Problemas de Olimpiadas sobre Cuadriláteros Rocío López Anguita


28 CAP. III. PROBLEMAS DE OLIMPIADAS

Sabemos, que también es racional sen2 γ = 1 − cos γ


senαsenγ
Por tanto sen α
senγ = sen2 γ es racional.
En segundo lugar, aplicamos el teorema del seno a los triángulos OAB y obtenemos que senABÕ
BOA
=
AO BC
senα y si lo aplicamos en el triángulo OC B tenemos que senBOC
Õ
= senγ .
OC

AB senα
Por lo que deducimos que OC = BC · senγ = r, es un número racional. Luego
OA

AC
AC = OA + OC = (1 + r)OA =⇒ OA =
1+r
y OA es racional.
ƒ

Ejercicio. 7.3. (Ref. Web (7), 2008)


Si un cuadrilátero convexo tiene la propiedad que cada una de sus dos diagonales biseca su área,
demuestra que es un paralelogramo

SOLUCIÓN.
Partimos del cuadrilátero ABC D dado.
T

A
X

P
B

C
Y

Sabemos que si trazamos paralelas a cada diagonal por los extremos de la otra, se forma un para-
lelogramo X Y Z T , como observamos en la figura. El cuál tiene el doble de área del cuadrilátero de
partida. (Esto es muy sencillo comprobarlo con GeoGebra)
A continuación lo demostramos:

22 de septiembre de 2014 Curso 20132014. TRABAJO FIN DE MÁSTER


SEC. 7. PROBLEMAS DE OLIMPIADAS: FASE LOCAL 29

Si tomamos el segmento AC por construcción, vemos que biseca a ABC D y también a X Y Z T , pero
siendo X Y Z T un paralelogramo, y AC paralela a los lados X Y y T Z, llegamos a que P es el punto
medio de BD.
De igual forma comprobamos que P es el punto medio de AC, ya que el segmento BD por cons-
trucción, vemos que biseca a ABC D y también a X Y Z T , pero siendo X Y Z T un paralelogramo y BD
paralela a los lados X T y Y Z llegamos a la conclusión que P es el punto medio de AC.
Luego, los triángulos AP D y BP C son iguales porque tienen dos lados iguales y también un ángulo.
Del mismo modo los triángulos AP B y C P D también son iguales.
Entonces llegamos a la conclusión de que el cuadrilátero inicial tiene iguales los lados opuestos, por
lo que es un paralelogramo.
ƒ

Ejercicio. 7.4. (Ref. Web (7), 2010)


Se considera un tetraedro regular, como el de la figura. Si el punto E recorre la arista AB. ¿Cuándo
el ángulo CÕE D es máximo?

B D

SOLUCIÓN.
En primer lugar, como vemos en el dibujo un tetraedro regular es un poliedro formado por cuatro
caras que son triángulos equiláteros, y cuatro vértices en cada uno de los cuales concurren tres caras.
(Es uno de los cinco poliedros perfectos llamados sólidos platónicos.)
Para simplificar los cálculos, vamos a suponer que el tetraedro tiene arista de longitud 1. Llamamos
α al ángulo C E D, y x, a la longitud del segmento AE Aplicamos el teorema del coseno al triángulo
AEC obteniendo:
EC 2 = x 2 + 1 − 2x cos 600 = x 2 + 1 − x

TFM: Problemas de Olimpiadas sobre Cuadriláteros Rocío López Anguita


30 CAP. III. PROBLEMAS DE OLIMPIADAS

Por otra parte,se tiene que la longitud EC es igual a la longitud E D, por simetría. De nuevo, aplicando
el teorema del coseno, ahora para p el triángulo E DC, resultando que:
1 = (x − x + 1) + (x − x + 1) − 2( x 2 − x + 1)2 cos α =⇒ 1 = 2(x 2 − x + 1) · (1 − cos α) Despejando
2 2
1
se tiene que: cos α = 1 − 2(x 2 −x+1)
La función coseno es decreciente en el primer cuadrante, por lo que para encontrar el valor máximo
de α tenemos que buscar el valor de x ∈ [0, 1] que haga mínima la función,es decir, que haga mínimo
el denominador.
Vamos a sacar el mínimo del denominador, para ello hacemos la derivada de 2(x 2 − x + 1) que es
4x − 2, seguidamente la igualamos a 0 por lo que el mínimo se alcanza cuando x = 12 .
Luego, el C ÕE D es máximo cuando E es el punto medio del lado AB. En ese caso tenemos: α =
1
arc cos 3 .
ƒ

Finalizado el ejercicio, me planteo lo siguiente:

Pregunta. 7.5.
¿Y cuándo el ángulo C
Õ E D es mínimo?

SOLUCIÓN.
Para ver cuándo el CÕ E D es mínimo tenemos que sacar el máximo del denominador.
En este caso, como solo nos ha salido un punto crítico, el cuál era mínimo, el máximo se alcanza en
los extremos, es decir en 0 y en 1, por lo que α vale para x0, α = arc cos 21 y para x = 1, α = arc cos 12 .
Luego, el ángulo CÕE D es mínimo cuando el punto E = A ó E = B.
ƒ

Ejercicio. 7.6. (Ref. Web (7), 2012)


Los puntos A1 , A2 , A3 , . . . A2n+1 son los vértices de un polígono regular de 2n + 1 lados. Hallar el
número de ternas Ai , A j , Ak tales que el triángulo Ai A j Ak es obtusángulo.

SOLUCIÓN.
En primer lugar, sabemos que al tener un número impar de vértices (2n + 1), no es posible construir
triángulos rectángulos.

22 de septiembre de 2014 Curso 20132014. TRABAJO FIN DE MÁSTER


SEC. 7. PROBLEMAS DE OLIMPIADAS: FASE LOCAL 31
A1
A2n+1 A2

A2n A3

An+2 An+1

Observando la figura, voy a intentar resolver el problema dejando un vértice agudo fijo y viendo
cuántos triángulos podemos formar.
Vemos que cualquier triángulo obtusángulo va a dejar el centro de la circunferencia ,O, fuera de él.
Por lo que si lo giramos en sentido directo o inverso alrededor de O, podemos conseguir que uno de
los vértices agudos esté en A1 . Los otros dos ángulosestarán en el conjunto {A2 , A3 , . . . , An+1 }, ó bien
n
en {An+2 , An+3 , . . . , A2n+1 }. El número buscado es 2 2 .
n

Como esto lo podemos hacer con cada uno de los 2n + 1 vértices, quedarán 2(2n + 1) 2 triángulos.
Antes de finalizar, tenemos que ser consciente de que cada triángulo lo hemos contado 2 veces, uno
para cada vértice agudo. En conclusión, la solución buscada es
 ‹
n
(2n + 1)
2

Como todo problema matemático, podemos intentar resolverlo por otro método.
Así teniendo la misma idea de fijar un ángulo agudo, en este caso el A1 , los tres lados del triángu-
lo abarcarán respectivamente x, y, y z lados del polígono de 2n + 1 lados. Entonces, tenemos que
x + y + z = 2n + 1. El lado opuesto al ángulo obtuso , digamos z, deberá cumplir z ≥ n + 1.
Por lo que tenemos que calcular el número de soluciones enteras positivas de la ecuación x + y + z =
2n + 1, para saber cuántos triángulos obtusángulo hay.
Para resolver dicha ecuación tenemos que considerar una condición, es decir, fijar una variable, por
ejemplo z, la cuál sabemos que puede tomar 2n + 1 valor.

Si z=n+1, queda x + y = n que tiene n − 1 soluciones.


Si z=n+2, queda x + y = n − 1 que tiene n − 2 soluciones.
...
Si z=2n-1, queda x + y = 1 que tiene 1 solución.

Luego, en total hay:

n(n − 1)
 ‹
n
(n − 1)(n − 2)(n − 3) . . . + 2 + 1 = = .
2 2

TFM: Problemas de Olimpiadas sobre Cuadriláteros Rocío López Anguita


32 CAP. III. PROBLEMAS DE OLIMPIADAS

Soluciones con el ángulo obtuso en A1 . Si consideramos todas las posibilidades para dicho ángulo,
en total queda  ‹
n
(2n + 1) .
2
ƒ

Ejercicio. 7.7. (Ref. Web (7), 2012)


Los puntos A1 , A2 , A3 , . . . A2n+1 son los vértices de un polígono regular de 2n lados. Hallar el número
de ternas Ai , A j , Ak tales que el triángulo Ai A j Ak es rectángulo y el número de ternas tales que el
triángulo es acutángulo.

SOLUCIÓN.
En este ejercicio tenemos un número par de vértices 2n, por lo que a diferencia del ejercicio anterior,
aquí si podemos formar triángulos rectángulos. Cuyas hipotenusas estarán sobre los n diámetros del
polígono.
Para cada diámetro fijado, el ángulo recto del triángulo puede ser cualquiera de los (2n − 2) vértices
sobrantes.
Luego, en total, los triángulos rectángulo que podemos obtener son

Rn = n(2n − 2) = 2n(n − 1)

A1
A2n A2

A2n−1
A3

An+2 An
An+1

Ahora vamos a calcular los triángulos acutángulos. Para ello se me ocurre calcular los triángulos
obtusángulos (del mismo modo que el ejercicio anterior) y restar al total los triángulos obtusángulos
y los triángulos rectángulos (ya calculados).
Sabemos que cualquier triángulo obtusángulo va a dejar el centro de la circunferencia, O, fuera de
él.
Por lo que si lo giramos en sentido directo, o inverso, alrededor de O, podemos conseguir que uno de

22 de septiembre de 2014 Curso 20132014. TRABAJO FIN DE MÁSTER


SEC. 7. PROBLEMAS DE OLIMPIADAS: FASE LOCAL 33

los vértices agudos esté en A1 . Los otros dos ángulos estarán en el conjunto {A2 , A3 , . . . , An }, ó bien
en {An+2 , An+3 , . . . , A2 n}
n+1

Luego, el número buscado será 2 2 . Como esto lo podemos hacer con cada uno de los 2n vér-
n−1

tices, quedarán 2(2n) 2 triángulos. Tenemos que darnos cuenta de que cada triángulo lo hemos
contado 2 veces,  uno para cada vértice agudo. Luego, el número de triángulos obtusángulo será
n−1
On = (2n) 2
2n

En conclusión, el número de triángulos acutángulos An será el número total de triángulos 3 menos
los triángulos rectángulos Rn y los obtusángulo On.

2n 2n n−1 2n! (n − 1)!


 ‹  ‹  ‹
An = − Rn − On = − 2n(n − 1) − 2n = − 2n(n − 1) − 2n =
3 3 2 (2n − 3)!3! (n − 3)!2!
2n(2n − 1)(2n − 2)(2n − 3)! 2n(n − 1)(n − 2)(n − 3)! n(n − 1)(n − 2)
 ‹
n
= −2n(n−1)− = ... = =2
(2n − 3)!3! (n − 3)2! 3 3
Luego,
 ‹
n
An = 2
3
ƒ

Ejercicio. 7.8. (Ref. Web (7), 2012)


Sea ABC D un cuadrilátero convexo y P un punto interior. Determinar qué condiciones deben cumplir
el cuadrilátero y el punto P para que los cuatro triángulos PAB, P BC, P C D y P DA tengan la misma
área.

SOLUCIÓN.
Consideramos los triángulos P C D y P C B

C
Z
P
Y
Q X

TFM: Problemas de Olimpiadas sobre Cuadriláteros Rocío López Anguita


34 CAP. III. PROBLEMAS DE OLIMPIADAS

Observando la figura, vemos que tienen la base común P C, y las alturas correspondiente son DX y
BY .
Por lo tanto si queremos que tengan la misma área, tienen que tener la misma altura, es decir DX =
BY .
Para que se de esto, el punto Q tiene que ser el punto medio de la diagonal BD y Q tiene que estar
en la recta P C.
Del igual forma, consideramos los triángulos P BA y P DA, en este caso la base común es PA y las
alturas son DZ y BZ, las cuales tienen que ser iguales para que tengan la misma área, por lo que Z
tiene que ser el punto medio de la diagonal BD y pasar por la recta PA. Luego Z = Q.
Entonces, AP tiene que pasar por Q. Luego AP y P C tienen dos puntos comunes, P y Q. Sin más
remedio AP y P C están alineados. Es decir, son las diagonales AC
De modo que es necesario que las dos diagonales se corten en el punto medio de una de ellas. Pero
mirando los triángulos P DA y P DC, que tienen la misma área, resulta que P tiene que ser el punto
medio de AC.
Luego, llegamos a la conclusión de que la condición que nos piden consiste en que las diagonales
del cuadrilátero se corten en el punto medio de una de ellas, y el punto P sea el punto medio de la
otra.
ƒ

Una vez que hemos resuelto este ejercicio, me resulta interesante plantear la siguiente cuestión:

Pregunta. 7.9.
¿Cómo debería ser el cuadrilátero cíclico para que el centro de la circunferencia fuese el punto P y
que los cuatro triángulos de un cuadrilátero tengan la misma área?

SOLUCIÓN.
Jugando un poco con Geogebra, nos resulta muy sencillo ver, que realmente el centro de la circun-
ferencia es el punto medio de una diagonal (AC), también tiene que ser el punto medio de la otra
diagonal (BD), ya que estamos en una circunferencia y las diagonales miden lo mismo. Por lo tanto,
el cuadrilátero cíclico tiene que tener paralelos sus lados opuestos, y por tanto es necesariamente un
rectángulo.

22 de septiembre de 2014 Curso 20132014. TRABAJO FIN DE MÁSTER


SEC. 8. PROBLEMAS DE OLIMPIADAS: FASE NACIONAL 35

A B

C
D

8. Problemas de Olimpiadas: Fase Nacional

En esta sección vamos a analizar algunos problemas con mayor grado de dificultad con respecto
la sección anterior ya que son propuestos en la fase nacional, relacionados siempre con las figuras
geométricas. Los problemas se han extraídos de la página oficial de las Olimpiadas Matemáticas
Española.

Ejercicio. 8.1. (Ref. Web (7), 2000)


Tomemos cuatro puntos situados en el interior o en el borde de un cuadrado de lado 1. Demuestra
que al menos dos de ellos están a distancia menor o igual que 1.

SOLUCIÓN.
Este ejercicio vamos a intentar resolverlo por reducción al absurdo.
Supongamos que distribuimos 4 puntos en el cuadrado de manera que cada una de las seis distancias
sea mayor que 1.
Entonces tenemos dos posibilidades, la cuales vamos a estudiar:

1. Los cuatro puntos forman un cuadrilátero convexo.


En este caso tendríamos la siguiente figura:

TFM: Problemas de Olimpiadas sobre Cuadriláteros Rocío López Anguita


36 CAP. III. PROBLEMAS DE OLIMPIADAS

D C
P1

P4
P2 γ
α

A B
P3

Consideramos α, β, γ, δ los ángulos del cuadrilátero convexo. Por lo que α + β + γ + δ = 3600 .


Además
p cualquier pareja de puntos del interior,(o p frontera) del cuadrado están a una distancia
d ≤ 2 ya que el diámetro del cuadrado mide 2
De la condición α + β + γ + δ = 3600 , se deduce que necesariamente uno de los ángulos ha de
ser mayor o igual que 900 , digamos por ejemplo α = 900 .
Entonces, tenemos: Pi P j > 1, i 6= j, i, j = 1, 2, 3, 4.
2 2 2 2 2
Luego, P1 P3 = P1 P2 + P2 P3 − 2P1 P2 · P2 P3 · cos α
como el cuadrilátero es convexo, 900 ≤ α ≤ 1800 y por tanto cos α ≤ 0 y en consecuencia:
2 2 2 p
P1 P3 ≥ P1 P2 + P2 P3 =⇒ P1 P2 > 2.
Por lo que llegamos a una contradicción. Entonces nos damos cuenta de que el error ha estado
en tener que Pi P j > 1, i 6= j, i, j = 1, 2, 3, 4.
En conclusión al menos dos de ellos están a una distancia menor o igual que 1.
2. Los cuatro puntos forman un cuadrilátero no convexo.
Si se forma un cuadrilátero no convexo podemos elegir tres puntos de los cuatro puntos for-
mando un triángulo de modo que el cuarto punto sea interior, como podemos ver en la figura:

D P1 C1

P4
P2 C

P3

A B

22 de septiembre de 2014 Curso 20132014. TRABAJO FIN DE MÁSTER


SEC. 8. PROBLEMAS DE OLIMPIADAS: FASE NACIONAL 37

Vamos a suponer que p el punto interior sea P4 . Sabemos que cada lado de dicho triángulo es
menor o igual que 2 (diámetro del cuadrado) yp por tanto q estará contenido en el triángulo
p p
equilátero de lado 2, y su circunradio es 2 = 2 · 3 = 23 < 1. Por lo tanto, si su centro es
3 2

C, P4 estará en el interior y la distancia


q de P4 a uno de los vértices será menor o igual que el
circunradio , es decir menor que 23 < 1 y por tanto menor que 1. Luego, hemos encontrado
un par de puntos a distancias menor o igual que 1. Por último, si tres puntos están alineados
se reduce al caso (1),es decir, si llamamosx 1 , x 2 , x 3 a las distancias entre puntos consecutivos,
p p
tenemos: x 1 , x 2 , x 3 ≤ 2 y por el principio del palomar, uno de ellos, digamos x 1 ≤ 32 < 1.
Luego queda demostrado el ejercicio para los posibles casos.
ƒ

Ejercicio. 8.2. (Ref. Web (7), 2001)


ABC D es un cuadrilátero inscrito en una circunferencia de radio 1 de modo que AB es un diámetro
y el cuadrilátero tiene una circunferencia inscrita. Probar que:
p
CD ≤ 2 5 − 4

SOLUCIÓN.
Para resolver el ejercicio, primero vamos a ver la figura que tenemos:

D
C

A B
O

Observando la figura, tenemos que O es el centro de la circunferencia, y vamos a llamar


2α = BOC,
Õ 2β = AOD,Õ 2γ = COD.Ö Para que el cuadrilátero ABC D admita una circunferencia inscrita
tiene que cumplir que la suma de los lados opuestos sean iguales, es decir

C D + 2 = BC + AD

Por otra parte, 2α + 2β + 2γ = 1800 =⇒ β = 90 − (α + γ) y además, por trigonometría, tenemos


que: BC = 2senα, DC = 2senγ, AD = 2senβ = 2 cos(α + γ) = 2 cos α cos γ − 2senαsenγ

TFM: Problemas de Olimpiadas sobre Cuadriláteros Rocío López Anguita


38 CAP. III. PROBLEMAS DE OLIMPIADAS

A continuación, expresamos la condición C D + 2 = BC + AD en función de α y el segmento DC que


determina por completo el cuadrilátero.
v p
t DC 2 4 − DC 2
cos γ = 1 − =
4 2
p
donde AD = 4 − DC 2 cos α − DCsenα sustituyendo nos queda:
p
DC + 2 = 2senα + 4 − DC 2 cos α − DCsenα

o lo que es lo mismo:
p
4 − DC 2 cos α + (2 − DC)senα = DC + 2

Por lo que, existirá circunferencia inscrita para los valores de DC que hagan compatible la última
ecuación con la incógnita α. En este momento se me ocurren dos caminos para seguir: Uno expresar
el seno en función del coseno y estudiar el discriminante de la ecuación de segundo grado que se
obtiene; Y otro, el que voy a desarrollar interpretando la ecuación como producto
p escalar de vectores.

→ −

Los vectores que tenemos son u = (cos α, sec α) de módulo 1 y v = ( 4 − DC 2 , 2 − DC). Por lo
tanto, la condición nos queda: |− →
v | cos δ = DC + 2 siendo δ = −Ø→
u ,−
→v.
Para que dicha condición sea compatible debe cumplirse DC + 2 ≤ |− → p
v | = 4 − DC 2 + (2 − DC 2 ),
elevando al cuadrado y operando, nos p queda: DC 2 + 8DC − 4 ≤ 0.
Las raíces de esta ecuación es DC = ±2 5−4, teniendo en cuenta que DC es un segmento no puede
ser negativo, por lo tanto la condición final es:
p
0 ≤ DC ≤ 2 5 − 4

Ejercicio. 8.3. (Ref. Web (7), 2004)


ABC D es un cuadrilátero cualquiera, P y Q los puntos medios de las diagonales BD y AC respectiva-
mente. Las paralelas por P y Q a la otra diagonal se cortan en O. Si unimos O con los cuatro puntos
medios de los lados X , Y, Z, T se forman cuatro cuadriláteros OX BY , OY C Z, OZ DT y OTAX . Probar
que los cuatro cuadriláteros tienen la misma área.

Comparar con el Ejercicio (7.8.).

SOLUCIÓN.
En primer lugar, hacemos un dibujo plasmando los datos de los que partimos:

22 de septiembre de 2014 Curso 20132014. TRABAJO FIN DE MÁSTER


SEC. 8. PROBLEMAS DE OLIMPIADAS: FASE NACIONAL 39

T
X
D
P
B

Q O
Z
Y

Observando la figura, para demostrar que los cuatro cuadriláteros, de distintos colores, tienen la
misma área, podemos probar que el área de cada cuadrilátero es la cuarta parte del área total. Por
un lado, tenemos que la quebrada AP C divide al cuadrilátero en dos partes de igual área, ya que AP
es la mediana de ABD y P C es la mediana de C BD.
Por otro parte, la quebrada T P Z divide al cuadrilátero AP C B en dos partes de igual área, pues P T
es la mediana de AP D y P Z es la mediana de C P D.
Luego tenemos ya probado que el área del cuadrilátero T P Z D, es la cuarta parte del área total del
cuadrilátero inicial.
Finalmente T Z es paralela a OP por serlo amabas a AC, luego los triángulos T P Z y T OZ tienen la
misma área y lo mismo les ocurre a los cuadriláteros T P Z D y T OZ D.
Del mismo modo se probaría para los otros tres cuadriláteros.
Para finalizar el ejercicio lo podemos comprobar fácilmente con GeoGebra.
Como todo ejercicio de matemáticas hay otra forma de realizarlo,la cual vamos a ver a continuación:
Sabemos que la superficie de un cuadrilátero se calcula como el semiproducto de las diagonales por
el seno del ángulo que forman, es decir: S = AC·BD2senα donde α = AC,
Ú BD
Además, Z T = X Y = 2 al ser Z T la paralela media del triángulo AC D y X Y la paralela media del
AC

triángulo ABC
Igualmente: X T = Z Y = BD 2 .
Para probar el enunciado bastará probar que:

AC · BD
senα = 4X T · AO · senβ
2

AC · BDsenα = 2X T · AOsenβ

ACsenα = 2AOsenβ

AQsenα = AOsenβ

AQ AO
=
senβ senα

TFM: Problemas de Olimpiadas sobre Cuadriláteros Rocío López Anguita


40 CAP. III. PROBLEMAS DE OLIMPIADAS

hemos llegado al teorema del seno en el triángulo AQO


Queda probado el enunciado por extensión de la demostración a los 4 cuadrilátero pequeños que
resulta ser una cuarta parte del grande.
ƒ

Ejercicio. 8.4. (Ref. Web (7), 2006)


Las diagonales AC y BD de un cuadrilátero convexo ABC D se cortan en E. Denotamos por S1 y
S2 a laspáreas p
de los triángulos ABE, C DE y del cuadrilátero ABC D respectivamente. Prueba que
S1 + S2 ≤ S. ¿Cuándo se alcanza la igualdad?
p

SOLUCIÓN.

A
D
S4

L
E S2
S1 K

S3
C

Como podemos ver en la figura, llamamos al área p


p S3 p del triángulo BEC, y S4 a la del triángulo AE D.
Entonces tenemos que demostrar que: S1 + S2 ≤ S,es decir

S1 +S2 ≤ S1 + S2 + S3 + S4
p p p

Elevando al cuadrado obtenemos: S1 +S2 +2 S1 S2 ≤ S1 +S2 +S3 +S4 , operando 2 S1 S2 ≤ S3 +S4


p p p p

Consideramos ahora K y L los pies de las perpendiculares en la diagonal AC trazadas desde D y B


respectivamente.
Entonces tenemos que: S1 = 12 AE · B L, S2 = 12 C E · DK, S3 = 21 C E · BK, S4 = 12 AE · DL Sustituyendo
esta expresión en la desigualdad anterior se llega a :
p 1
AE · B L − AE · DK ≤ (C E · BK + AE · DL)
2
que es precisamente la desigualdad entre la media aritmética y la media geométrica de los dos
productos C E · BK y AE · DL.
Esta última desigualdad se alcanza si y solo si C E · BK = AE · DL ⇐⇒ BK
DL = C E Las rectas BK y DL
AE

22 de septiembre de 2014 Curso 20132014. TRABAJO FIN DE MÁSTER


SEC. 8. PROBLEMAS DE OLIMPIADAS: FASE NACIONAL 41

son paralelas.
D L = DE , por la semejanza entre los triángulos BK E y DLE, por lo tanto
Así BK BE

BK AE
C E · BK = AE · DL ⇐⇒ =
DL CE
se convierte en
BE AE
=
DE CE
Y recíprocamente por la semejanza de triángulos, DE BE
= CAEE se verifica ⇐⇒ AB y C D son paralelas, es
decir el cuadrilátero dado es un trapecio con los lados paralelos AB y C D
ƒ

Ejercicio. 8.5. (Ref. Web (7), 2008)


Dada una circunferencia y en ella dos puntos fijos A y B, otro variable P y una recta r. Se trazan las
rectas PA y P B que cortan a r en C y D respectivamente. Determina dos puntos fijos M y N de r,
tales que el producto C M · DN sea constante al variar P.

SOLUCIÓN.
Empezamos trazando una paralela a r que pase por A y corte a la circunferencia en A0 . Y seguidamente
trazamos otra paralela que pase por B y corte a la circunferencia en B 0 , de modo que tenemos un
trapecio isósceles formado por AA0 BB 0 .

A A0

D
M C N
r

B
B0

Como vemos en la figura los puntos de intersección de AB 0 y BA0 con r, van a determinar los puntos
fijos M y N que buscamos.
Por lo tanto, los triángulos que vemos sombreado en el dibujo, AM C y el N BP son semejantes, ya
que tienen dos ángulos iguales (como podemos observar), M Ö AC = BÖ0 BP por ser ángulos inscrito en

TFM: Problemas de Olimpiadas sobre Cuadriláteros Rocío López Anguita


42 CAP. III. PROBLEMAS DE OLIMPIADAS

el mismo arco y B
Ö 0 BP = N
Ö DB por seerBB 0 paralela a r, por lo tanto M
Ö AC = N
Ö DB. Con los mismo
argumentos tenemos que AM C = AB B = D N B.
Ö Õ 0 Ö
Si establecemos ahora la proporcionalidad de los lados obtenemos que:
AM ND
= ⇐⇒ M C · N D = AM · BN
MC BN
resultado que no depende de P.
En resumen, si la recta r pasa por el punto A, entonces M = A = C, por lo que no se forma en
triángulo AM C. En este caso C M = 0 y el producto C M · DN = 0 es constante. De igual modo, este
producto es cero si la recta r pasa por el punto B o por los puntos A y B en cuyo caso tendríamos
C M = DN = 0. Luego queda demostrado el ejercicio.
ƒ

Ejercicio. 8.6. (Ref. Web (7), 2010)


Sea ABC D un cuadrilátero conexo. Sea P la intersección de AC y BC. El ángulo < AP D = 600 . Sean
E, F , G y H los puntos medios de los lados AB, BC, C D y DA respectivamente. Halla el mayor número
real positivo K tal que
EG + 3H F ¾ kd + (1 − k)s
siendo s el semiperímetro del cuadrilátero ABC D y d la suma de las longitudes de sus diagonales.
¿Cuándo se alcanza la igualdad?

SOLUCIÓN. p
En primer lugar vamos a demostrar que k = 1 + 3 y que la igualdad se da sí y solo si, ABC D es un
rectángulo.

D
Z
H

A
Y

P G
W
E

B C
F

22 de septiembre de 2014 Curso 20132014. TRABAJO FIN DE MÁSTER


SEC. 8. PROBLEMAS DE OLIMPIADAS: FASE NACIONAL 43

En primer lugar, vamos a considerar cuatro puntos exteriores a ABC D que van a ser W ,X , Y , Z.
Elegiremos los puntos de tal forma que los triángulos ABW y DC Y son equiláteros, el triángulo BC X
sea isósceles en X , y el triángulo AZ D sea isósceles en Z y BX
Õ C =A
Õ Z D = 1200 .
Los cuadriláteros WAP B, X BP C, Y C P D Y Z DPA son cíclicos.
Luego, aplicando el teorema de Ptolomeo, se obtiene que:
p p
W P = PA + P B, X P 3 = P B + P C, Y P = P C + P D, Z P 3 − 0P D + PA

Por otro lado,


W
× PY = W
Ö P B + 600 + C
Õ PY = W
Ö AB + 600 + Ö
C DY = 1800

Luego, W, P, Y están alineados y, de igual modo Z, P, X también lo están.


Por lo que
W Y = W P + PY = PA + P B + P C + P D = AC + BC
1 1
X Z = X P + P Z = (P B + P C + P D + PA) = (AC + BD)
3 3
Por la desigualdad triangular, obtenemos:

W Y ≤ W E + EG + GY, X Z ≤ X F + F H + H Z

Luego:
1 1 1 BC AC
AC + BD ≤ AB + EG + DC , (AC + BD) ≤ + FH + p
3 3 3 2 2 3
Si sumamos las dos desigualdades tenemos:
p p
(1 + 3)(AC + BC) ≤ EG + 3F H + s 3

lo que es lo mismo:
p p
EG + 3F H ≥ (1 + 3)d − s 3
p
Luego, si k = 1 + 3 =⇒ EG + 3F H ≥ kd + (1 − k)s
Entonces la igualdad se da, si y sólo si, tenemos por un lado que los puntos W, E, G, Y están aliena-
dos, y por otro lado X , F, H, Z también lo están.
Como W E es perpendicular a AB, y GY es perpendicular DC, por tanto AB y DC deben de ser parale-
los. De igual forma, AD y BC también deben de ser paralelos.Luego ABC D debe ser un paralelogramo.
Además, la recta EG es perpendicular a DC, lo que implica que ABC D es un rectángulo.
Por lo tanto, llegamos a la conclusión,que EG + 3F H = kd + (1 − k)s se da si ABC D es un rectángulo.
Ahora bien, sea un número real positivo l tal que EG + 3F H ≥ ld + (1 − l)s. En consecuencia, si
ABC D es un rectángulo tenemos: kd + (1 − k)s ≥ ld + (1 − l)s, es decir k(d − s) ≥ l(d − s)
Pero la desigualdad
p triangular, implica que d > s, por lo que k ≥ l. Luego el número real buscado es
k = 1 + 3 y la igualdad se da s, y sólo si, ABC D es un rectángulo.
ƒ

TFM: Problemas de Olimpiadas sobre Cuadriláteros Rocío López Anguita


44 CAP. III. PROBLEMAS DE OLIMPIADAS

9. Problemas de Olimpiadas: Fase Internacional


En última instancia,vamos a desarrollar los problemas con mayor grado de dificultad, los cuales son
propuesto en la fase internacional. Debemos de tener en cuenta que los mismos versan sobre la
temática de cuadriláteros.

Ejercicio. 9.1. (IMO[1, pag. 44], 1965)


Sea ABC D un tetraedro, no regular, y AB = a, C D = b. La distancia entre los segmentos AB y DC es
d y su ángulo es ω. Sea P el plano paralelo a AB y a DC tal que el radio entre las distancias de P y
a AB y DC es k. El plano P divide al tetraedro en dos sólidos geométricos.
Encontrar el radio entre los volúmenes de esos dos sólidos.

SOLUCIÓN.
En primer lugar vamos hacer el dibujo plasmando los datos facilitados.

C N
M

L K

B
A

Como vemos en la figura hemos consideramos K, L, M , N los puntos de intersección entre el plano
P y los segmentos BC, AC, AD, BD respectivamente.
Tenemos que K L es paralelo a AB, LM es paralelo a C D, M N es paralelo a AB, y N K es paralelo a
C D. Luego el cuadrilátero K LM N es un paralelogramo, ya que LM es paralelo a C D y LK es paralelo
a AB. También tenemos que el M Ö LK = ω.
Luego el área del paralelogramo es:
S = K L · LM senω
Por otro lado, consideramos x la distancia entre el plano P y el segmento C D, teniendo:

d−x d
= k =⇒ = k + 1
x x
De igual forma tenemos:
d AB d CD
= y =
x KL d − x LM

22 de septiembre de 2014 Curso 20132014. TRABAJO FIN DE MÁSTER


SEC. 9. PROBLEMAS DE OLIMPIADAS: FASE INTERNACIONAL 45

Luego , obtenemos que:

ax b(d − x) ax b(d − x)
KL = , LM = yS = S(x) = · · senω
d d d d
Entonces tenemos que el volumen es:
Z x
abx 2 abx 3
Vol(K LM N C D) = S(t)d t = ( − )senω
0
2d 3d 2
y
Z d−x
ab(d − x)2 ab(d − x)3
Vol(K LM N AB) = S(t)d t = ( − )senω
0
2d 3d 2
Luego, el radio entre los volúmenes de los dos sólidos es:

Vol(K LM N AB) 3k + 3
= k2
Vol(K LM N C D) 3k + 1
ƒ

Ejercicio. 9.2. (IMO[1], 1967)


Sea un tetraedro, no regular, tal que solo uno de sus lados tiene una longitud mayor que 1. Demostrar
que el volumen del tetraedro no es mayor que 81 .

SOLUCIÓN.
En primer lugar, vamos a fijar el segmento que va a ser mas largo que 1, el cuál será C D = a y
veamos como queda el dibujo.
A

L H
a

K
C

TFM: Problemas de Olimpiadas sobre Cuadriláteros Rocío López Anguita


46 CAP. III. PROBLEMAS DE OLIMPIADAS

Tenemos que los lados de los triángulos ADC y BDC son menores que 1. Tenemos que tener cuidado,
y no confundir los lados,que hemos fijado con a, con la base.
Ahora, consideramos elqtriángulo BDC cuya altura es BK, es máxima cuando DB = C B = 1 y en este
a2
caso obtenemos que: 1− 4 . Luego
v
t a2
BK < 1−
4
Del mismo modo, considerando el triángulo DAC cuya altura es AL, la cual es máxima cuando AD =
AC = 1 y por lo que obtenemos que: v
t a2
AL < 1 −
4
Análogamente, considerando el triángulo ADB cuya altura es AH, la cual es máxima cuando AD =
AB = 1 obteniendo en este caso que: v
t a2
AH < 1 −
4
En conclusión el volumen del tetraedro es:
1 1 1 a2 1
V= · SBC D · AH < · a(1 − ) = a(4 − a2 )
3 3 2 4 24
Finalmente, solo queda ver que a(4 − a2 ) ≤ 3
Para ello se me ocurre estudiar la función. p p
Sacamos las raíces de a(4 − a2 ) = 3, las cuales son a1 = 1, a2 = 1+ 2 13 , a3 = 1− 2 13 . Como vemos la
solución es a3 < 0, por lo que la descartamos,
p
ya que a es un segmento y tiene que ser positivo.
1+ 13
Ahora estudiamos la función entre [0, 2 ]. Es fácil ver (comprobando) que el valor máximo que
toma a(4 − a2 ) es cuando a = 1 que vale 3.
En conclusión,
1
V≤
8
ƒ

Ejercicio. 9.3. (IMO[1, pag. 56], 1967)


Sea ABC D un paralelogramo tal que AB = a, AD = 1 y Õ B AD = α y ABD es un triángulo agudo.
Probar que es posible cubrir el paralelogramo con 4 círculos KA, KB , KC , K D de centros A, B, C, D,
respectivamente, si y sólo si p
a ≤ cos α + 3senα

SOLUCIÓN.
Como partimos de un paralelogramo, si trazamos una de las diagonales, por ejemplo BD tenemos
dos triángulos simétricos ABD y BDC, como vemos en la figura.

22 de septiembre de 2014 Curso 20132014. TRABAJO FIN DE MÁSTER


SEC. 9. PROBLEMAS DE OLIMPIADAS: FASE INTERNACIONAL 47

Por lo tanto, para resolver el ejercicio, solo tenemos que buscar una solución necesaria y suficiente
para que el triángulo ABD esté cubierto por KA, KB , K D .
Sea T la circunferencia circunscrita del triángulo ABD y de centro O, sabemos que por ser el triángulo
agudo O está dentro de él.
Sabemos que: OA = OB = OD = R, y para cualquier punto P interior, al menos una de las distancias
PA, P B, P D es menor que R.
Esto es fácil de ver, considerando los seis triángulos interiores de ABD, determinados por los seg-
mentos OA, OB, OD y las mediatrices de los segmentos AB, BD, DA
Luego, ABD está cubierto por KA, KB , K D ⇐⇒ R ≤ 1.
En el triángulo ABD, el radio R viene determinado por

AB · BD · DA
R=
4S
donde S es el área.
Tenemos BD2 = a2 + 1 − 2a cos α y S = 21 asenα. Luego:

R < 1 ⇐⇒ a2 (a2 + 1 − 2a cos α) ≤ 4a2 sen2 α ⇐⇒

⇐⇒ a2 + 1 − 2a cos α ≤ 4 − 4 cos2 α ⇐⇒
⇐⇒ a2 − 2a cos α + cos2 α ≤ 3sen3 α ⇐⇒
p
⇐⇒ |a − cos α| ≤ 3senα
En consecuencia, en el triángulo ADB, cos α es la longitud de la proyección del segmento AD en AB
y luego a > cos α. Sustituyendo esto, en la condición que hemos obtenido anteriormente, tenemos:
p
a ≤ cos α + 3senα

Demostrando así el ejercicio, por la simetría del problema.


ƒ

TFM: Problemas de Olimpiadas sobre Cuadriláteros Rocío López Anguita


48 CAP. III. PROBLEMAS DE OLIMPIADAS

Ejercicio. 9.4. (IMO[1, pag. 63], 1968)


Demostrar que en cualquier tetraedro, no regular, existe un vértice tal que los lados que salen de él
son los lados de un triángulo.

SOLUCIÓN.
Sea ABC D un tetraedro y considerando AB = a, su lado más largo. Para mayor comodidad tomamos
AC = c, AD = d, BC = d, BD = f , C D = g.
Vamos a demostrar que, el vértice A, o el vértice B, se puede elegir como respuesta al problema.

c
d

C g D

b f

Supongamos que A no cumple la condición: c + d ≤ a.


De lo contrario, A cumple: a + c > d y a + d > c. De los triángulos ABC y ABD obtenemos: c + d > a
yd+f >a
Sumando las dos desigualdades obtenemos: 2a < b + c + d + f
Si unimos ahora estas dos desigualdades 2a < b + c + d + f y c + d ≤ a conseguimos: b + f > a.
Luego, los lados que salen del vértice B son los lados de un triángulo.
ƒ

Ejercicio. 9.5. (IMO[1, pag. 80], 1970)


Sea ABC D un tetraedro, no regular, tal que BD es perpendicular a DC y el pie de las perpendicular
desde D en la cara ABC es el ortocentro del triángulo ABC. Demostrar que

(AB + BC + CA)2 ≤ 6(AD2 + BD2 + C D2 )

¿Cuándo se da la igualdad?

22 de septiembre de 2014 Curso 20132014. TRABAJO FIN DE MÁSTER


SEC. 9. PROBLEMAS DE OLIMPIADAS: FASE INTERNACIONAL 49

SOLUCIÓN.
Consideramos H el ortocentro del triángulo ABC. Sabemos que un tetraedro, donde H es el pie de
la altura desde D, tiene los lados opuestos perpendiculares.

A
L
C

M H
K

Para demostrar esto, vamos a ver el plano generado por las líneas de AH y DH y observaremos que
BC es perpendicular a ella.
De este modo, BC es perpendicular a AD. Las otras perpendiculares se obtiene de las misma forma.
Tenemos que DC es perpendicular a BD, y DC es perpendicular a AB, por lo tanto se deduce que DC
es ortogonal en el plano ABC.
Luego, DC es perpendicular a AD. Del mismo modo, obtenemos que BD es perpendicular AD, por lo
tanto, cada par de aristas que derivan del vértice D son perpendiculares.
En conclusión tenemos:

(AB + BC + CA)2 ≤ 3(AD2 + BD2 + BD2 + C D2 + C D2 + AD2 ) = 3(AB 2 + BC 2 + AC 2 )

Esta desigualdad es obvia. Para facilitar las operaciones vamos a hacer el siguiente cambio: BC = a,
CA = b, AB = c, y utilizar la desigualdad de Cauchy–Schwarz:
p p
a + b + c ≤ 1 + 1 + 1 a2 + b2 + c 2 =⇒ (a + b + c)2 ≤ 3(a2 + b2 + c 2 )

Teniendo esta desigualdad, podemos ver que se da (a + b + c)2 = 3(a2 + b2 + c 2 ) ⇐⇒ a = b = c, es


decir, el triángulo ABC tiene que se equilátero para obtener la igualdad.
ƒ

Ejercicio. 9.6. (IMO[1, pag. 95], 1972)


Demostrar que cualquier cuadrilátero cíclico se puede descomponer en n, (n ≥ 4), cuadriláteros
cíclicos.

TFM: Problemas de Olimpiadas sobre Cuadriláteros Rocío López Anguita


50 CAP. III. PROBLEMAS DE OLIMPIADAS

SOLUCIÓN.
Sabemos que cualquier cuadrilátero cíclico se puede descomponer en cuatro cuadrilátero cíclico„
dos de los cuales son trapecios isósceles.
Considerando B el mínimo ángulo de ABC D, tomamos un punto M en el interior del cuadrilátero
ABC D cerca del vértice B. Trazamos dos líneas paralelas a BC, y a BA, desde el punto M . Dichas
líneas cortan a C D en el punto F , y a DA en el punto E, como podemos ver en la figura.

C
F
D

L O
M E

B A
K

Por otro lado, consideramos dos puntos interiores lo segmentos AB y BC, los cuales son K y L res-
pectivamente.Si unimos los puntos K, L y M obtenemos dos cuadriláteros AE M K y M LC F tales que
son trapecios isósceles.
Es fácil ver que K B LM es un cuadrilátero cíclico, por la forma de construirlo, ya que tenemos:
L BK = FÖ
Õ BK M = ×
M E, × M F D, K
Ö ML = EÕDF y BÖLM = ×
M E D, por lo tanto es semejante al cuadrilátero
cíclico ABC D, (esto lo podemos observar perfectamente con el dibujo)
Finalmente, cada trapecio isósceles puede descomponerse en un número arbitrario de trapecios isós-
celes, mediante líneas paralelas a las bases.
En consecuencia, el enunciado del ejercicio queda demostrado.
ƒ

Ejercicio. 9.7. (IMO[1, pag. 96], 1972)


Dado cuatro planos paralelos. Demuestra que existe un tetraedro regular que tiene sus vértices en
esos planos.

SOLUCIÓN.
Consideramos los cuatro planos π1 , π2 , π3 , π4 paralelos, y sea A1 , A2 , a3 , A4 los cuatro puntos del
tetraedro tal que Ai ∈ πi , con i = 1, 2, 3, 4.

22 de septiembre de 2014 Curso 20132014. TRABAJO FIN DE MÁSTER


SEC. 9. PROBLEMAS DE OLIMPIADAS: FASE INTERNACIONAL 51

El plano π2 interseca al tetraedro formando el triángulo A2 BC donde B ∈ [A1 , A3 ] y C ∈ [A1 , A4 ]


El plano π3 interseca al tetraedro formando el triángulo A3 DE donde D ∈ [A1 , A4 ] y E ∈ [A2 , A4 ]

A1

A2
B C

A3
E
D

A4

Sea α1 , α2 y α3 las distancias entre los planos π1 yπ2 , π2 yπ3 , π3 yπ4 respectivamente y sea a la
longitud del segmento A1 , A2 .
Luego tenemos las siguientes relaciones:

A1 B A1 C α1
= =
BA3 CD α2

CD A E α
= 2 = 2
DA4 EA4 α3

A1 B + BA3 = A1 C + C D + DA4 = A2 E + EA4 = a

A1 D α1 + α2
=
DA4 α3

Tenemos un sistema de ocho ecuaciones lineales, donde lo único que tenemos fijos son los planos
π1 , π2 , π3 , π4 , y las distancias a los planos α1 , α2 , α3 , por lo que nuestras incógnitas son A1 , A2 , A3 , A4 , B, C, D
y a.
Luego tenemos un sistema de ocho ecuaciones lineales con ocho incógnitas.
Estudiando el sistema tenemos que:

TFM: Problemas de Olimpiadas sobre Cuadriláteros Rocío López Anguita


52 CAP. III. PROBLEMAS DE OLIMPIADAS

BA3 A1 C
α2 = α1 , α1 = α2 ,
A1 B CD

DA4 A2 E
α3 = α2 , α2 = α3 ,
CD EA4

DA4
α3 = (α1 + α2 ),
A1 D
o lo que es lo mismo:
α1 α3
A1 B = BA3 , EA4 = A2 E ,
α2 α2
α3 α1
DA4 = C D , A1 C = C D ,
α2 α2

α1 + α2 α1 + α2
A1 D = DA4 = CD ,
α3 α2

Además se verifica:

A1 B + BA3 = a, A1 C + C D + DA4 = a, A2 E + EA4 = a.

Llegándose a que
2(α1 + α2 )
 ‹
a = A1 C + C D + CA4 = C D
α2

α1
 ‹
a = BA3 +1
α2

α3
 ‹
a = A2 E 1 + .
α2
Por lo tanto el sistema, parametrizado en a, tiene solución, la cual podemos expresar en función,
por ejemplo, de C D. ƒ

22 de septiembre de 2014 Curso 20132014. TRABAJO FIN DE MÁSTER


Bibliografía

[1] Dr. Mircea Becheanu, International Mathematical Olympiads, 1959-2000, University of Bucha-
rest, 2001. 9.1., 9.2., 9.3., 9.4., 9.5., 9.6., 9.7.

[2] H. S. M. Coxeter, Introduction to geometry, John Wiley, 2nd. Ed., 1969.

[3] H. S. M. Coxeter y S. L. Greitzer, Geometry revisited, The Mathematical Association of America,


1967.

[4] A. Engel, Problem–solving strategies, Springer, 1998.

[5] K. Kedlaya, Notes on euclidean geometry MIT, 1999.

[6] Cristóbal Sánchez-Rubio and Manuel Ripollés Amela, Manual de matemáticas para preparación
olímpica, Universitat Jaume I. Castellón, 2000.

[7] Sessions de preparació per l’olimpiada matemàtica, Soc. Cat. Mat. Barcelona, 2000.
Refencias Web:

Teoría.

1. www.acm.ciens.ucv.ve/material.php

2. www.ual.es/eventos/OMERSMALMERIA/GEOMETRIA/Preparacion.pdf

3. www.ual.es/eventos/OMERSMALMERIA/GEOMETRIA/Apuntes.pdf

4. http://cepre.uni.edu.pe/pdf/cuadrilateros.pdf

Problemas para aplicar teoría.

5. http://wdb.ugr.es/~jmmanzano/preparacion/

6. http://lya.fciencias.unam.mx/omdf/material/GeometriaChuy.pdf

Problemas de Olimpiadas.

7. http://platea.pntic.mec.es/~csanchez/olimprab.htm

Vous aimerez peut-être aussi